Physical Examination and Health Assessment- Remembering (Knowledge)

अब Quizwiz के साथ अपने होमवर्क और परीक्षाओं को एस करें!

Hepatitis B Virus (HBV)

Transmission through all fluids of an infected source Considerations Hepatitis B vaccination, public education, blood screening, use of gloves when handling secretions, proper sterilization of equipment, reporting of all known cases

Hepatitis C Virus (HCV)

Transmission through contaminated blood, plasma, and needles Considerations See Hepatitis B.

Chlamydia

Transmission through sexual contact Considerations Public education; use of latex condoms

Which of the cranial nerves are functionally classified as mixed? The Nervous System

Trigeminal (V) facial (VII) glossopharyngeal (IX) vagus (X) PERIPHERAL NERVOUS SYSTEM

What bones comprise the hard palate? The Digestive System

Two maxillae and two palatines

Abdominal Distention

Tympanites, or enlargement of the abdomen with gas or air as a result of excessive swallowing of air, eating gas-producing foods, or an inability to expel gas. Causes Constipation, fecal impaction, or postoperative conditions Remedies Rectal tube can be used to expel air; increase ambulation, and change position in bed..

Name the four fat-soluble vitamins that can be stored in the liver Nutrition and Metabolism

Vitamins A, D, E, and K

Caloric Increase Needed For Select Injury Factors

"Minor surgery 10 Mild infection 20 Moderate infection 40 Severe infection 60 CHF. 30 Cancer therapy 30 Pulmonary disease 30 Wound healing 20-60 Long bone fracture 30-50%

Asepsis

"Prevention of the transfer of microorganisms and pathogens"

Calculating Percent Weight Change

% weight change = (usual body weight -current body weight) /usual body weight x 100

Personal distance (proxemics)

(1 to 4 ft.) will be used for the physical assessment. (18 in to 4 ft) personal space between friends

Measles

(Red, Hard, Morbilli, Rubeola) Transmission through airborne droplets or direct contact with lesions Considerations Public education about vaccine; avoidance of contact with infected persons

Coagulation disorders

(e.g., thrombosis, embolus). Blood clot causes mechanical obstruction to blood flow.

attention-deficit/hyperactivity disorder (ADHD)

a psychological disorder marked by the appearance by age 7 of one or more of three key symptoms: extreme inattention, hyperactivity, and impulsivity (restless, fidgeting, excess talking).

lordosis

abnormal anterior curvature of the lumbar spine (sway-back condition)

acute hepatitis symptoms

1-4 months • Anorexia • Nausea, vomiting • Right upper quadrant discomfort • Constipation or diarrhea • Decreased sense of taste and smell • Malaise • Headache • Fever • Arthralgias • Urticaria • Hepatomegaly • Splenomegaly • Weight loss • Jaundice • Pruritus • Dark urine • Bilirubinuria • Light stools • Fatigue • Continued hepatomegaly with tenderness • Weight loss Laboratory panel- IgM anti-HAV, IgM anti-HBc, HBsAg, and anti-HCV

soft diet

1. Description -Soft consistency and mild spice 2. Patient Complaints - Difficulty swallowing

full liquid diet

1.Description - Clear liquids plus milk products, eggs 2.Patient Complaints -Transition from clear to regular diet

Clear liquid diet

1.Description -Broth, tea, clear soda, strained juices, gelatin 2.Patient Complaints -Recovery from surgery or very ill

How many pairs of spinal nerves are connected to the spinal cord? The Nervous System

31 pairs

Social distance

4 to 12 feet, is usually the distance category for most of the interview.

What is the correct sequence of steps when performing a wound irrigation? 1. Use slow continuous pressure to irrigate wound. 2. Attach angio catheter to syringe 3. Fill syringe with irrigation fluid 4. Place water proof bag near bed 5. Position angio catheter over wound

4, 3, 2, 5, 1

What is the normal pH for freshly voided urine?

4.6-8.0 (average 6.0)

grading muscle strength (1-5 scale)

5/5: Normal strength 100 percent Normal interpretation 4/5: Movement possible against some resistance *note-this category can be subdivided 75 percent Full movement but not against gravity 3/5: Movement possible against gravity *note- does not test again resistance 50 percent Normal movement against gravity 2/5: Movement possible but not against gravity *note- test the joint in its horizontal plane 25 percent Movement if gravity eliminated 1/5: Muscle flicker but no movement 10 percent No movement 0/5 No contraction 0 percent Contraction

moderate malnutrition

70% to 80% of ideal weight

Interpreting BMI

<18.5 underweight 18.5-24.9 healthy weight 25-29.9 overweight 30-34.9 obesity class 1 35-39.9 obesity class 2 >40 obesity class 3

Calculating Estimated Needs

A "rule-of-thumb" method of estimating calorie requirements Multiply weight in kg by 30 cal/kg for most healthy adults 25 cal/kg for elderly adults 20-25 cal/kg for obese adults protein requirements Healthy adults need 0.8 g protein/kg

Alcoholic psychosis

A confused, disoriented state after intoxication

Spirituality

A connection with something larger than oneself and belief in transcendence

polar covalent bond

A covalent bond between atoms that differ in electronegativity. The shared electrons are pulled closer to the more electronegative atom, making it slightly negative and the other atom slightly positive.

antifibrinolytic

A drug that prevents the lysis of fibrin thereby promoting clot formation coagulation modifier drugs

What are triglycerides composed of? Nutrition and Metabolism

A glycerol subunit to which are attached three fatty acids

International Normalized Ratio (INR)

A standardized measure of the degree of coagulation achieved by drug therapy with warfarin sodium coagulation modifier drugs

Aura

A warning of a seizure. Actually, the aura is an early part of the seizure itself.

The nurse knows that the newborns of mothers with diabetes often exhibit tremors, periods of apnea, cyanosis, and poor suckling ability. With which complication are these signs associated? Hypoglycemia Hypercalcemia Central nervous system edema Congenital depression of the islets of Langerhans

A. Hypoglycemia

Which actions describe the beneficial effects produced by sulfonylurea oral hypoglycemics? (Select all that apply.) Stimulate insulin secretion from beta cells Increase hepatic glucose production Enhance action of insulin in various tissues Inhibit breakdown of insulin by liver

A. Stimulate insulin secretion from beta cells C. Enhance action of insulin in various tissues D. Inhibit breakdown of insulin by liver

An 89-year old client has been diagnosed with a urinary tract infection. Which best describes a psychological effect of this type of infection? A. Agitation B. Lethargy C. Depression D. Paranoia

ANS B * Agitation This is not a common symptom of a UTI in the older adult population. * Depression This is not a common symptom of a UTI in the older adult population. * ✅Lethargy A urinary tract infection can cause symptoms of urinary frequency and pelvic pain for many clients. In many older adults, altered mental status and lethargy are the only symptoms of a urinary tract infection. The nurse should look for signs of cognitive changes in older adult clients and assess for possible infection when they occur . * Paranoia This is not a common symptom of a UTI in the older adult population.

A 66-year-old client who has just been diagnosed with Parkinson's disease has many questions for the nurse. Which information regarding cognitive changes is associated with early Parkinson's disease? Select all that apply. * A. Dementia that interferes with activities of daily living B. Slowness in memory recall C. Feeling distracted D. Difficulties with planning E. A significant decline in physical abilities

ANS B,C,D * ✅Slowness in memory recall Having trouble with memory is an early noticeable change in a client with Parkinson's. * * ✅Difficulties with planning The early stages of Parkinson's disease may cause mild cognitive changes that are noticeable to the client. Examples include difficulties with planning, feeling distracted, or having trouble with memory. A significant decline in physical abilities This does not occur in the early stages of Parkinson's. Dementia that interferes with activities of daily living The client typically does not have cognitive changes significant enough to cause interference with daily activities. * ✅Feeling distracted Distractibility is an early noticeable change in a client with Parkinson's.

A client presents to the emergency room with c/o nausea, vomiting, and diarrhea. Which of the following medications is appropriate to treat this condition? * Indomethacin * Loperamide * Ibuprofen * Propanolol

ANS C * Indomethacin This is an antirheumatic medication used most often for clients with rheumatoid arthritis. * * ✅Loperamide This is an antidiarrheal medication. * Ibuprofen This is a non-steroidal anti-inflammatory agent that treats mild to moderate pain and inflammation. * Propanolol This is a beta blocker that is used to treat hypertension, angina, arrhythmias, and heart attacks.

A client with severe, uncontrolled hypertension is being evaluated for chronic kidney disease. The provider checks the glomerular filtration rate and determines that it is 22 mL/min. Which stage of chronic kidney disease does this GFR place the client? * A. Stage III * B. Stage I * C. Stage II * D. Stage IV

ANS D ✅Stage IV The glomerular filtration rate (GFR) is a measure of kidney function that determines how well the kidney is able to filter waste products. A normal GFR is approximately 125 mL/minute. A client who has a GFR of 22 mL/min would be categorized as having severe kidney disease and would be classified at stage IV. * Stage I Stage I GFR is >90 mL/minute. * Stage II Stage II GFR is 60-89 mL/min. * Stage III Stage III is 30-59 mL/min. Stage IV is 15-29 mL/min. Stage V is <15 mL/min.

* Question 9 of 10 Which of the following medications lower blood pressure by decreasing the volume of blood in the body? A. Calcium channel blockers B. Beta-adrenoceptors C. Diuretics D. ACE inhibitors

ANS:

TRUE or FALSE A person can be malnourished without being underweight. Weight does not provide qualitative information about body composition.

ANS: True

What does the review of systems provide the nurse? a. Physical findings r/t each system b. Information regarding health promotion practices c. An opportunity to teach the patient medical terms d. Information necessary for the nurse to diagnose the patient's medical problem

ANS: B The purposes of the review of systems are to: (1) evaluate the past and current health state of each body system, (2) double-check facts in case any significant data were omitted in the present illness section, and (3) evaluate health promotion practices. The review of systems does not provide the physical findings of each body system, an opportunity to teach the patient medical terms, or the information necessary for the nurse to diagnose the patient's medical problem. DIF: Cognitive Level: Remembering (Knowledge) MSC: Client Needs: Health Promotion and Maintenance

A client presents to the emergency department with complaints of chest pain. The EKG shows normal sinus rhythm, but the client's troponin comes back at 3.4 ng/mL. The nurse knows this client is most likely experiencing an N-STEMI and prepares which medication to give to the client? A.Hydrochlorothiazide B.Hydromorphone C.Heparin D.Humalog

ANS: C Hydrochlorothiazide This is a thiazide diuretic that treats conditions such as hypertension, congestive heart failure, and cirrhosis. ✅Heparin Heparin is an anticoagulant that is commonly used to prevent blood clots, but is also used as anticoagulant therapy for myocardial infarctions. Hydromorphone This is an opioid analgesic used to alleviate pain. Humalog This is a fast-acting insulin that is used to control blood sugar levels.

* Question 7 of 10 The nurse is caring for a client who has tachycardia and is hypertensive. The provider has prescibed metoprolol. The nurse knows that this drug works in which of the following ways? A. Binds to alpha receptors B. Binds to cholinergic receptors C. Binds to calcium channel receptors D. Binds to beta receptors

ANS: D Metoprolol is a beta blocker, which helps slow the heart rate. Reduces ventricular rate

How much weight would a 200-pound adult need to lose in a month to be considered significant? a. It depends on the patient's BMI. b. More than 5 pounds c. More than 7.5 pounds d. More than 10 pounds

ANS: D Weight loss is judged as significant if there is a 5% loss over the course of 1 month.

In using verbal responses to assist the patient's narrative, some responses focus on the patient's frame of reference and some focus on the health care provider's perspective. Which is an example of a verbal response that focuses on the health care provider's perspective? a. Empathy b. Reflection c. Facilitation d. Confrontation

ANS: D When the health care provider uses the response of confrontation, the frame of reference shifts from the patient's perspective to the perspective of the health care provider, and the health care provider starts to express his or her own thoughts and feelings. Empathy, reflection, and facilitation responses focus on the patient's frame of reference. Empathy, reflection, and facilitation responses focus on the patient's frame of reference. DIF: Cognitive Level: Remembering (Knowledge) MSC: Client Needs: Psychosocial Integrity

The flow of blood through the kidney tissue follows what path? The Urinary System

Abdominal aorta, renal artery, segmental arteries, lobar arteries, interlobar arteries, arcuate arteries, interlobular artery, afferent arteriole, glomerulus (glomerular capillaries), efferent arteriole, peritubular capillaries (vasa recta), interlobular veins, arcuate veins, interlobar veins, lobar veins, segmental veins, renal vein, and inferior vena cava.

Virulence

Ability of a microorganism to produce disease"

Communicable

Ability of a microorganism to spread disease

Morphine

Adult: 0.1 mg/kg to max dose of 10 mg IV, IM, SC, IO Pediatric: 0.1 - 0.2 mg/kg does IV, IO, IM, SC every 5 mins titrated to max does of 5 mg

Midazolam (Versed)

Adult: 0.5 - 2.5 mg slow IV push; may be repeated to max 0.1 mg/kg Pediatric dose: to facilitate intubation medical control may order: 6 months- 5 yr : 0.05- 0.1 mg/kg Iv max dose of 5 mg 6-12yr : 0.1 mg/kg IV max dose of 8 mg

Glucagon

Adult: 0.5-1 mg IM, SC, or slow IV; may repeat q 20 mintues PRN Pediatric dose: 0.03- 0.1 mg / kg/ dose +note do not exceed 1 mg) q 20 min IM,IO,SC, slow IV

Sodium Bicarbonate

Adult: 1 mEq/kg IV, may repeat with 0.5 mEq/kg every 10 mins, *side adult infusion 1-4 amps in 1 Liter D5W or NS, rate determined by sending physician Pediatric: 1 mEq/kg IV, may repeat with 0.5 mEq/kg every 10 mins, *side adult infusion 1-4 amps in 1 Liter D5W or NS, rate determined by sending physician

Disease

Alteration of normal tissues, body processes, or functions

Anorexia nervosa

An eating disorder; loss of appetite for food not explainable by disease

A nurse is discussing restorative health care with a newly licensed nurse. Which of the following examples should Th nurse include in the teaching? (Select all that apply) A. Home health care B. Rehabilitation facilities C. Diagnostic center D. Skilled nursing facilities E. Oncology centers

And: A,B,D Restorative health care involves intermediate follow-up care for restoring health and promoting self-care.

The nurse is caring for a client with multiple kidney stones in the renal pelvis. The urologist has determined that the client can pass the stones without surgical intervention. The nurse anticipates which medication to assist with discomfort? A. Propranolol B. Phenytoin C. Phenazopyridine D. Paroxetine

Answer C * ✅Phenazopyridine This is a non-opioid analgesic that targets the urinary tract system. * Phenytoin This is an antiseizure medication used for clients with grand mal siezures. * Propranolol This is a beta blocker used to treat arrhythmias, and hypertension. * Paroxetine This is an SSRI for treatment of depression and anxiety.

amino acid diseases

amino acid diseases include PKU, cystinosis, and cystic fibrosis.

Diabetes Ketoacidosis (DKA)

an acute complication of diabetes. The serum values indicate metabolic acidosis with some accompanying respiratory compensation. Elevated glycosylated hemoglobin shows that he has been hyperglycemic for several months. This diabetes is type 1 and will require insulin administration. Patient education must be provided with instructions regarding recognition of future signs and symptoms of hyperglycemia and hypoglycemia, blood glucose self-monitoring, insulin therapy, diet, and exercise.

A follow-up database is used: A) to monitor progress of short-term or chronic health concerns. B) for a limited or short-term problem usually consisting of one problem, one cue complex, or one body system. C) to evaluate the cause of disease. D) to perform a thorough or comprehensive health history and physical examination.

ans: a A follow-up database is performed to follow up, or evaluate changes, on short-term and chronic health problems, but would be collected at appropriate intervals after a complete database was collected at the initial visit. is most appropriate when a nurse evaluates identified problems at regular intervals used in all settings to monitor progress of short-term or chronic health problems

Broca's aphasia

aphasia the person can understand language but cannot express himself using words or language.

Palpation

applies the sense of touch to assess texture, temperature, moisture, organ location and size, as well as any swelling, vibration or pulsation, rigidity or spasticity, crepitation, presence of lumps or masses, and the presence of tenderness or pain.

What is the term that describes the distinctive pattern of internal white matter within the cerebellum? The Nervous System

Arbor vitae

Name the four sections of the colon. The Digestive System

Ascending, transverse, descending, and sigmoid

Alert

Awake and aware, responds appropriately, begins conversation (A&O × 3: alert and oriented to person, place, time) Levels of Consciousness

water soluble vitamins

B vitamins vitamin C - Try orange and citrus fruits, tomatoes, kiwi, red and green peppers, broccoli, Brussels sprouts, cantaloupe, and strawberries

During a diabetes mellitus campaign, the community nurse is assessing different clients. Which client should be treated first? Client A - A1C% 5.6; Fasting Plasma GL 100 mg/dL; Two Hour Plasma GL 150 mg/dL Client B - A1C% 6.8; Fasting Plasma GL 130 mg/dL; Two Hour Plasma GL 200 mg/dL Client C - A1C% 6.0; Fasting Plasma GL 120 mg/dL; Two Hour Plasma GL 130 mg/dL Client D - A1C% 6.1; Fasting Plasma GL 100 mg/dL; Two Hour Plasma GL 140 mg/dL

B. Client B - A1C% 6.8; Fasting Plasma GL 130 mg/dL; Two Hour Plasma GL 200 mg/dL

Which information should the nurse include in a teaching plan for patients taking oral hypoglycemic drugs? (Select all that apply.) Take your medication only as needed. Report symptoms of anorexia and fatigue. Explain dietary changes are not necessary. Advise to avoid smoking and alcohol consumption. Instruct that it is okay to skip breakfast 1 to 2 times per week.

B. Report symptoms of anorexia and fatigue. D. Advise to avoid smoking and alcohol consumption

alteplase (Activase) adverse effects

Bleeding Transient dysrhythmias (desired) Angioedema (rare) Intracranial bleeding (rare)

Ideal Body Weight (IBW)

By dividing current weight by her usual weight and then multiplying by 100, 100 pounds for first 5 feet then five pounds for every inch

What minerals are required to build strong structural components of the skeleton? Nutrition and Metabolism

Calcium, phosphorus, and magnesium

Although we eat six main types of chemical substances, which three have to be chemically digested to be absorbed? The Digestive System

Carbohydrates, proteins, and fats

Etiology

Cause of a disease

Name the three stages of gastric secretion. The digestive System

Cephalic, gastric, and intestinal

Neurologic Function

Cerebral Includes mental status, thought processes, emotions, level of consciousness, orientation, memory language, appropriateness, intelligence, and developmental age Cranial Nerves Olfactory Optic Oculomotor Trochlear Trigeminal Cerebellar Includes coordination and balance; muscle size, strength, and tone (see p. 190); evaluation of reflexes"

Insomnia

Chronic difficulty with sleep patterns

Demulcent -Antidiarrheal Medications

Coats and protect

In general, what controls the release of digestive juices in such a way that they appear in proper amounts when and for as long as needed? The Digestive System

Complicated nerves and hormonal reflex mechanisms

Medicated Enemas

Contains a therapeutic agent (e.g., Kayexalate to treat high potassium levels)

Explain the difference between a somatic reflex and an autonomic (visceral) reflex. The Nervous System

Contraction of a skeletal muscle is a part of the somatic reflex. Impulse conduction over somatic reflex arcs produces somatic reflexes. Autonomic (visceral) reflexes consist of contractions of smooth or cardiac muscle or secretion by glands. They are mediated by impulse conduction over autonomic reflex arcs.

Nutrition therapy recommendations

are usually general suggestions to increase/ decrease, limit/avoid, reduce/encourage, or modify/maintain aspects of the diet because exact nutrient requirements are determined on an individual basis.

What are the three main parts of a typical tooth? The Digestive System

Crown, neck, and root

Which insulin can be administered by continuous intravenous (IV) infusion? A. Insulin glargine (Lantus) B. Insulin aspart (Novolog) C. Insulin detemir (Levemir) D. Regular insulin (Humulin R)

D. Regular insulin (Humulin R)

Medications that May Discolor Urine

Dark Yellow • Vitamin B2 Orange • Sulfonamide • Phenazopyridine HCl (Pyridium) • Warfarin (Coumadin) Pink or Red • Thorazine • Ex-Lax • Phenytoin (Dilantin) Green or Blue • Amitriptyline • Methylene blue • Triamterene (Dyrenium) Brown or Black • Iron • Levodopa • Nitrofurantoin • Metronidazole (Flagyl)"

Korsakoff syndrome

Delirium or hallucinations often caused by chronic alcohol use

Paranoia

Delusions of persecution or of grandeur

regular diet

Description -Has all essentials; no restrictions Patient Complaints No special diet needed

Obtunded

Difficult to arouse, slow to respond, and returns to sleep quickly Levels of Consciousness

Initial insomnia

Difficulty falling asleep

Terminal insomnia

Difficulty going back to sleep

Narcolepsy

Difficulty in regulating between sleep and awake states; person may fall asleep without warning

Active ROM (AROM)

Done unassisted & challenging build strength & increase muscle fiber (restorative/continuous ) Mechanics - increase fluids, decrease dehydration Electrolyte imbalance and replacement, increase water, sweat, respiration. Early, small frequent, and continued after

Device used to check for pulsation:

Doppler used to augment pulse or blood pressure measurements

convalescence phase

During the recovery period as symptoms subside"

EASI

Elder Abuse Suspicion Index

Magnesium Sulfate

Epsom salt Adult: asthma 1-2 gm IV, seizure activity associated with pregnancy 1-4 gm IV over 10 min, Torsades de Pointes or Refractory VF/VT 1-2 gm IV over 1-2 min Pediatric: asthma/bronchospasm, severe 25 mg/kg over 10 min IV, usually mixed in 50-100 CC of NS to be given IV

Diarrhea -Altered Bowel Elimination Patterns

Expulsion of fecal matter that contains too much water Causes Infection, anxiety, stress, medications, too many laxatives at one time, or food or drug allergies or reactions Remedies Add bulk or fiber to diet, maintain fluids and electrolytes, eat smaller amounts of food at one time, add cheese or bananas to diet, and rest after eating" .

Streptokinase (Streptase)

Fibrinolysis is enhanced ■ Drug class: Thrombolytic ■ Peak time: 30-60 min ■ Duration: 4-12 hr ■ Half-life: 4 hr Drugs affecting Hemostasis

Urokinase (Abbokinase)

Fibrinolysis is enhanced ■ Drug class: Thrombolytic ■ Peak time: end of infusion ■ Duration: 12 hr ■ Half-life: 20 min

TWEAK questionnaire

For women with alcohol problems, assesses tolerance, worry, eye-opener, amnesia, and "kut" down

Where is cerebrospinal fluid formed? The Nervous system

Formation of cerebrospinal fluid occurs by separation of fluid from blood in the choroid plexuses.

Hyperextension

Function Straighten joint beyond limits Examples Head tilted back, fingers pointed up

(Gastrin, Gastric inhibitory peptide, Secretin, or Cholecystokinin-pancreozymin) stimulates the secretion of gastric juice rich in pepsin and hydrochloric acid. The Digestive System

Gastrin

What is the term used to describe the backward flow of stomach acid up through the lower esophageal sphincter and into the lower esophagus? The Digestive System

Gastroesophageal reflux disease (GERD)

S4 (Atrial gallop) Tennessee

Heart abnormal sound Cause- Atrial contraction into noncompliant ventricle Location- Mitral if LV Tricuspid if RV

S2 -CARDIAC AUSCULTATION

Heart sound Cause: Closure of semilunar valves (aortic and pulmonic) Location:Base

S1 -CARDIAC AUSCULTATION

Heart sounds Cause - Closure of AV valves (mitral and tricuspid) Location- Apex

What is the name of the concave notch on the medial surface of each kidney? The Urinary System

Hilum medial depression for blood vessels and ureter to enter kidney chamber

Ports

How microorganisms exit and enter a system

What is the process in which a compound unites with water and then splits into simpler compounds? The Digestive System

Hydrolysis

Identify the three pairs of peduncles where fibers of the longer tracts enter or leave the cerebellum The Nervous System

Inferior cerebellar peduncles, middle cerebellar peduncles, superior cerebellar peduncles CENTRAL NERVOUS SYSTEM

What sugar-regulating hormone is secreted by the beta cells of the pancreatic islets? Nutrition and Metabolism

Insulin

Sleep apnea

Intermittent periods of cessation of breathing during sleep

Low residue diet

It limits high-fiber foods. That will ease symptoms like diarrhea, bloating, gas, and stomach cramping. The goal of the diet is to have fewer, smaller bowel movements each day.

Body Alignments

Ldeal alignment, Kyphosis, Lordosis

Emotions involve the functioning of what part of the cerebrum? The Nervous System

Limbic system neural system (including the hippocampus, amygdala, and hypothalamus) located below the cerebral hemispheres CENTRAL NERVOUS SYSTEM

Dense organs

Liver, Sleen

Duodenal Ulcers - digestive facts

Located in the first 1 to 2 cm of the duodenum Generally occurs in people ages 40 to 60 years Most common ulcer in people younger than 65 years old More common in men Lower mortality rate than gastric ulcers Four times more prevalent than gastric ulcers Risk factors are chronic obstructive pulmonary disease, alcohol, cirrhosis, pancreatitis, smoking, renal failure, stress Pain occurs after eating and at night Pain in midepigastric area Pain is described in the back Pain relieved by milk or antacids May cause weight gain Recurs seasonally (spring and fall) Rarely malignant High risk of perforation"

What is the term for the chewing movements in the mouth? The Digestive System

Mastication

What is the term for the fan-shaped projection of the parietal peritoneum from the lumbar region of the posterior abdominal wall? The Digestive System

Mesentery

Manic-depressive

Mood swing of very high to very low

To propel food from the pharynx into the esophagus, what three openings must be blocked? The Digestive System

Mouth, nasopharynx, and larynx

Blood Pressure Normal Averages (Systolic/Diastolic)

Newborn: 65-90/30-60 mm Hg Infant: (1 year) 65-125/40-90 mm Hg (2 years) 75-100/40-90 mm Hg Child: (4 years) 80-120/45-85 mm Hg (6 years) 85-115/50-60 mm Hg Adolescent: (12 years) 95-135/50-70 mm Hg (16 years) 100-140/50-70 mm Hg Adult: (18-60 years) 110-140/60-90 mm Hg (60+ years) 120-140/80-90 mm Hg" .

Name the nitrogenous waste products that are found in urine.

Nitrogenous waste products include urea, uric acid, ammonia, and creatinine.

Name five endocrine glands that play key roles in the glucose homeostatic mechanism Nutrition and Metabolism

Pancreatic islets, anterior pituitary gland, adrenal cortex, adrenal medulla, and the thyroid gland

Name the three pairs of salivary glands. The Digestive System

Parotids, submandibulars, and sublinguals

Chain

Path of infection; the components of the infectious disease process

Fowler's

Patient is partly sitting with knees slightly bent. The head of the bed can be at semi-Fowler's (45 degrees) or high Fowler's (90 degrees).

Prone

Patient lies on abdomen with arms at sides.

Dissociative disorder

Person escapes stress through memory or identity changes

Psychosomatic

Person is limited in coping skills, which produces physical effects

Hypertonic Enemas

Phosphates irritate bowel and draw fluid into bowel through osmosis (90-120 mL; hold for 10-15 min)

Source

Point that initiates chain of infection

Colonization

Presence of a potentially infectious organism in or on a host but not causing disease

Contamination

Presence of an infectious agent on a surface

Clean

Presence of few microorganisms or pathogens with no visible debris"

Absence seizure

Previously called petit mal. This is a generalized seizure without shaking.

Identify the three basic food types that provide balanced nutrition. Nutrition and Metabolism

Proteins, fats, and carbohydrates

What are the two bulges of white matter located on the ventral surface of the medulla The Nervous System

Pyramids CENTRAL NERVOUS SYSTEM

Epilepsy

Recurrent unprovoked seizures

Personality disorder

Repetitive, irresponsible, and manipulative behaviors

What is the term that describes the folds lining the inner wall of the stomach? The Digestive System

Rugae

Conversion disorder

Sensory or motor impairment in the absence of organic cause

Alcohol Withdrawal

Signs and Symptoms. Confusion, sweating, pallor, palpitations, hypotension, seizures, coma. Protocols may vary by facility. Withdrawal protocols may include seizure precautions, keeping the side rails up and padded, taking vital signs frequently (every 30-60 minutes or per hospital protocol), and close observation. Provide a safe environment. Perform neurologic, memory, and orientation checks. Document any withdrawal activity and actions taken.

Astringent-Antidiarrheal Medications

Skinks inflamed tissues

(STaT) questions

Slapped, Threatened, and Throw. If a woman answers "yes" to any of the Abuse Assessment Screen questions, then the nurse should ask questions designed to assess how recent and how serious the abuse was. Asking the woman an open-ended question, such as "tell me about this abuse in your relationship" is a good way to start.

lethargic (somnolent)

Sleeps but easily aroused, speaks and responds slowly but appropriately Levels of Consciousness

Somnambulism

Sleepwalking, night terrors, or nightmares"

Why are submandibular glands called mixed or compound glands? The Digestive System

Submandibular glands are called mixed or compound glands because they contain both serous (enzyme) and mucus-producing elements.

Repolarization of the heart muscle

T wave represents The various components of each heartbeat on the ECG represent important functions of the heart and its conduction system. The T wave, toward the end of each heartbeat, represents repolarization of the heart muscle, in which the impulse has traveled throughout the heart and it is preparing to start the cycle over again.

Hypotonic Enemas

Tap water (1 L; hold for 15 min); avoid with cardiac patients

Identify the two main structures that make up the diencephalon. The Nervous System

Thalamus and hypothalamus CENTRAL NERVOUS SYSTEM

How are the functions of the brainstem similar to the functions of the spinal cord? The Nervous System

The brainstem, like the spinal cord, performs sensory, motor, and reflex functions.

Distinguish between the cortical nephrons and the juxtamedullary nephrons. The Urinary System

The cortical nephrons are located almost entirely in the renal cortex. The juxtamedullary nephrons lie near the junction of the cortical and medullary layers but have Henle loops that dip far into the medulla. The specialized role of the juxtamedullary nephrons is to concentrate urine.

thrombolytic drug

The general term for a drug that dissolves thrombi coagulation modifier drugs

intimate zone of personal space

The intimate zone (1-1 ft.)

What is the lingual frenulum? The Digestive System

The lingual frenulum is a fold of mucous membrane in the midline of the undersurface of the tongue that helps anchor the tongue to the floor of the mouth.

simple partial seizure

The patient remains alert and is behaving appropriately.

Describe the purpose of the villi and microvilli in the walls of the small intestine. The Digestive System

The presence of the villi and microvilli increases the surface area of the small intestine hundreds of times, thus making this organ the main site of digestion and absorption.

Public distanceterm-175

The public zone (12+ ft.)

What are the two major functions of the urinary bladder? The Urinary System

The urinary bladder serves as a reservoir for urine before it leaves the body; aided by the urethra, it expels urine from the body.

Water soluble vitamins include

Thiamin Riboflavin Niacin Vitamin B6 Folate Vitamin B12 Pantothenic acid Biotin Choline Vitamin C - Try orange and citrus fruits, tomatoes, kiwi, red and green peppers, broccoli, Brussels sprouts, cantaloupe, and strawberries

disulfiram

This disulfiram-type reaction includes vomiting and hypertension. Used in tx of alcohol cessation. increased acetaldehyde when drinking (toxic)-->N/V-

Bruxism

Tooth grinding during sleep

How does total metabolic rate differ from basal metabolic rate? Nutrition and Metabolism

Total metabolic rate is the amount of energy used or expended by the body in a given time. It is often expressed in kilocalories per hour or per day. The basal metabolic rate is the energy used to do the work of maintaining life under basal conditions. Basal metabolic rate usually constitutes about 55% to 60% of the total metabolic rate.

coagulation system

coagulation system was divided into two initiating pathways, the tissue factor (extrinsic) and the contact factor (intrinsic) pathway, which met in a final common pathway to: 1. make a fibrous network to trap 2. stop bleeding 3. provides framework for repair to begin

Otoscope

directs light into the ear canal and onto the tympanic membrane that divides the external and middle ear.

Reflection

echoes the patient's words, repeating part of what the person has just said, and it also can help express the feelings behind a person's words.

linoleic acid (omega 6)

found in vegetable and nut oils is the predominant n-6 polyunsaturated fatty acid (PUFA) in the Western diet and we can obtain it from vegetable oils such as sunflower, safflower, soybean, corn, and canola oils as well as nuts and seeds

herbs with anticoagulant properties.

garlic ginger, ginseng, and ginkgo should be avoided taking clopidogrel (Plavix). drug blocks reduction in aggregation

types of peptic ulcers

gastric and duodenal

Retention Oil

given to soften stool (hold for 1 hr) "Types of Enemas

culture

is a complex phenomenon that includes attitude, beliefs, self-definitions, norms, roles, and values learned from birth through the processes of language acquisition and socialization. Culture has four basic characteristics, one of which is that it is adapted to specific conditions r/t environmental and technical factors and to the availability of natural resources. The other three characteristics are: (1) learned from birth through the processes of language acquisition and socialization; (2) shared by all members of the cultural group; and (3) dynamic and ever changing. It is not biologically or genetically determined and is learned by the individual. It is a universal phenomenon and is important because a person's culture defines health and illness, identifies when treatment is needed and which treatment is acceptable, and informs a person of how symptoms are expressed and which symptoms are important.

Confusion

is a disturbance of consciousness characterized by inability to engage in orderly thought or by lack of power to distinguish, choose, or act decisively.

evidence-based practice (EBP)

is a systematic approach to practice that emphasizes the use of research evidence in combination with the clinician's expertise and clinical knowledge (physical (physical assessment), as well as patient values and preferences, when making decisions about care and treatment. EBP is more than simply using the best practice techniques to treat patients, and questioning tradition is important when no compelling and supportive research evidence exists.

A 24-hour recall

is either an interview or a questionnaire that asks a person to recall everything eaten with the last 24 hours.

P wave is represented by:

is represented by the p wave The cardiac electrical impulse transmission from the SA node to the AV node .

mononucleosis (mono)

large lymph nodes, sore throat, fatigue, enlarged spleen most common in young adults: "Shaggy" white-purple exudate Transmission through saliva Considerations Public education; good hygiene

high-fiber foods

like whole-grain breads and cereals, nuts, seeds, raw or dried fruits, and vegetables.

Glipizide (Glucotrol)

short-term elevation in blood glucose level that occurs after he eats.works best if given 30 minutes before meals.

PTSD (Post Traumatic Stress Disorder)

the person relieves the trauma many times, intrusively and unwillingly. The same feelings of helplessness, fear, or horror recur. Avoidance of any trigger associated with the trauma occurs, and the person has hypervigilance, sleep problems, and difficulty concentrating, leading to feelings of being permanently damaged.

alteplase (activase, tPA)

treatment for acute myocardial infarction. Because of its short half-life, it is given along with heparin to prevent reocclusion of the infarcted blood vessel. Fibrinolysis is enhanced A pharmaceutically available tissue plasminogen activator (tPA) that is created through recombinant DNA techniques coagulation modifier drugs ■ Drug class: Thrombolytic ■ Peak time: 5-10 min ■ Duration: 2-3 hr ■ Half-life: 5 min . readministered because it has a very short half-life of 5 minutes. Because of its short half-life, it is given along with heparin to prevent reocclusion of the infarcted blood vessel. pharmaceutically available tissue plasminogen activator (tPA) that is created through recombinant DNA techniques coagulation modifier drugs

Clarification

used when the person's word choice is ambiguous or confusing. Clarification is also used to summarize the person's words or to simplify the words to make them clearer; the nurse should then ask if he or she is on the right track.

intimate partner violence (IPV)

violence that occurs between individuals who maintain a romantic or sexual relationship

insoluble fiber

whole wheat bread, bran and whole-grain cereals and dried peas and beans. whole wheat bagels or tortillas made with whole wheat flour for sandwiches and wraps.

Massage techniques.

• Assess if massage is contraindicated. • Start with the patient lying flat or on his or her side. • Begin with the forehead and work down the body. • Use a gentle but firm touch. • Always stroke toward the heart. • Rub downward on the chest and back. • Stroke upward on the arms. • Use a light lotion or oil." .

To thicken stool, a person should eat:

• Bananas, rice, bread, potatoes • Creamy peanut butter, applesauce • Cheese, yogurt, pasta, pretzels • Tapioca, marshmallows Foods and Their Effect on Fecal Output

Chronic Hepatitis symptoms

• Malaise • Easy fatigability • Hepatomegaly • Myalgias and/or arthralgias • Elevated liver enzymes (aspartate aminotransferase [AST] and alanine aminotransferase [ALT])

Products with no fiber

■ Animal products ■Milk

Dipyridamole (Persantine)

■Drug Class -Antiplatelet ■Peak time - 75 min (PO) 6.5 min (IV) ■ Duration - 3/4 hr 30 min ■Half/life - 10 hr

Ibuprofen (Motrin)

■Drug Class: NSAID Antiplatelet ■Peak time: 1-2 hr ■Duration: 4-6 hr ■Half/life: 1.8 -2 hr

Metroprolol (Lopressor)

beta blocker adult: HTN 1.25-5 mg every 6-12 hr , myocardial infarction (acute) IV 5 mg every 5-10 min max of 3 doses in early treatment of MI Pediatric: not recommened

What is the mechanism of action of - pramlintide (Symlin)

blood glucose levels are reduced. mimicking the action of the natural pancreatic hormone amylin. Amylin is secreted along with insulin in response to food intake and influences postmeal glucose levels by slowing gastric emptying, suppressing glucagon secretion (which reduces the liver's glucose output), and increases the sense of satiety. an incretin mimetic, given by subcutaneous injection

Heparin-induced thrombocytopenia (HIT)

can result from prolonged therapy due to immune-mediated platelet aggregation. A complication of prolonged heparin therapy (typically 5 days or more) is heparin-induced thrombocytopenia. This occurs more frequently when bovine lung heparin is used than with porcine heparin. In type 1 heparin-induced platelet deficiency there is a transient self-limiting fall in platelet count to 50 × 109 cells/L, probably as a result of direct heparin-induced platelet agglutination. Type 2 deficiency is more severe and overlaps with heparin-associated thrombosis. It occurs in around 6% of patients. Platelet count falls to 10 × 109 cells/L and is associated with thromboembolic phenomena. Thrombocytopenia results from immune-mediated platelet aggregation triggered by immunoglobulins G and M antibodies against platelets. The con- dition generally resolves rapidly on discontinuation of heparin, although occasionally it may persist for 1-2 months; in this case, intravenous immunoglobulin therapy is used. Reintroduction of heparin can be catastrophic.

Meningitis (Bacterial)

classic triad of headache, neck stiffness and fever -photophobia, vomiting, altered mental status Transmission through airborne droplets or direct contact Considerations Public education, vaccination; early prophylaxis of exposed contacts

enoxaparin (Lovenox)-contraindication coagulation modifier drugs

contraindicated in patients with an indwelling epidural catheter. The medication can be given 2 hours after the epidural is removed. This is very important for nurses to remember, because giv- ing an LMWH with an epidural has been associated with epidural hematoma.

Body Mass Index (BMI)

determine total body fat, calculated by using height and weight measurements, is a practical marker of optimal weight for height and an indicator of obesity.

Salmonellosis

diarrhea, abdominal cramps, vomiting, fever Transmission through ingestion of contaminated food Considerations Proper cooking and storage of food; good handwashing before food preparation

the mini-mental state examination (MMSE)

easy test of 11 questions, evaluates cognitive functioning, good screening tool to detect dementia and delirium and to differentiate these from psychiatric mental illness.

Warfarin (Coumadin)

effect of these drugs is related to their ability to interfere with the production of biologically active vitamin K-dependent clotting factorse. ■Drug Class -anticoagulant ■Peak time 0.5-3 days ■ Duration 2-5 days ■Half/life 0.5-3 days Drugs affecting Hemostasis

First-level priority problems

emergent, life-threatening, and immediate, such as establishing an airway or supporting breathing monitoring Severely abnormal vital signs

Waist-to-hip ratio (WHR)

evaluating patients for obesity-related diseases waist or abdominal circumference divided by the hip or gluteal circumference; method for assessing fat distribution

A complete database is: A. used to collect data rapidly and is often compiled concurrently with lifesaving measures. B. used for a limited or short-term problem usually consisting of one problem, one cue complex, or one body system. C. used to evaluate the cause or etiology of disease. D. used to perform a thorough or comprehensive health history and physical examination.

focus concerned with an individual's total health; collected in primary care settings, such as a pediatric or family practice clinic, independent or group private practice, college health service, women's health care agency, visiting nurse agency, or community health agency. conducted for a patient being admitted to a long-term care facility or being admitted for a scheduled surgery.

Hand Washing Procedure

hands before and after every physical patient encounter; after contact with blood, body fluids, secretions, and excretions; after contact with any equipment contaminated with body fluids; and after removing gloves. 1.) Have soap, paper towels, orange stick or nail file, and a wastebasket. 2.) Push watch, and sleeves up arm 4-5 inches 3.) Stand away from sink so soap & faucet are easy to reach. DO NOT let clothes touch sink! 4.) Turn on and adjust water until warm 5.) Wet wrists & hands, keeping hands BELOW elbows 6.) Apply 1 teaspoon of soap 7.) Rub palms together and wash for 15-20 seconds 8.) Wash each hand & wrist thoroughly, cleaning well between fingers 9.) Clean under fingernails rubbing against palms 10.) Clean under nails with file or orange stick first washing of day or when hand are highly soiled 11.) Rinse wrists and hands well letting water flow DOWN from arms to hands 12.) Pat dry with paper towels starting from fingertips working up 13.) Discard paper towels 14.) Turn off faucets with clean paper towels 15.) Discard paper towels into wastebasket

low blood sugar

headache, hunger, weakness, sweating, confusion, irritability, dizziness, fast heart rate, or feeling jittery;

Apraxia

impaired ability to carry out motor activities despite intact motor function impairment in the ability to program movements of the tongue, lips, and throat required to produce the proper sequence of speech sounds occur with dementia.

apraxia of speech

impairment in the ability to program movements of the tongue, lips, and throat required to produce the proper sequence of speech sounds

Culturally Competent

implies that the caregiver understands and attends to the total context of the individual's situation. This competency includes awareness of immigration status, stress factors, other social factors, and cultural similarities and differences. -Care must be sensitive to needs of individuals, families, or groups from diverse cultures -The healthcare system is a culture with customs, rules, values, and a language of its own -Nursing is the largest subculture of the healthcare system

Bulimia Disorder

in which vomiting is self-induced after eating large amount of food

Protein-calorie malnutrition (PCM)

inadequate consumption of protein and energy, resulting in a gradual body wasting and increased susceptibility to infection Marasmus, disease is due to prolonged starvation.

Symptoms of nicotine withdrawal

include vasodilation, headaches, anger, irritability, frustration, anxiety, nervousness, awakening at night, difficulty concentrating, depression, hunger, impatience, and the desire to smoke.

mobility status history

increased pressure, shearing, and friction can lead to breakdown ADL

A calorie count

involves calculating the calories of all foods consumed for a period of time and is often performed for hospitalized patients.

neologism

involves coining a new word which is inventing or making up words that have no real meaning except for the person. new word or expression

Colonic enema

irrigation Used to expel flatus

Reason for seeking care (chief complaint)

is a brief spontaneous statement in the person's own words that describes the reason for the visit. It states one (possibly two) signs or symptoms and their duration. It is enclosed in quotation marks to indicate the person's exact words.

Hepatitis

is a broad term that means inflammation of the liver. Hepatitis A acute RNA only vaccination found in feces. Hepatitis B acute and chronic DNA vaccination found in blood/fluids. Hepatitis C RNA is the most common cause of chronic liver disease

Aphasia

is a disorder of language comprehension. To assess the mental status of a patient with aphasia, the nurse should ask questions to assess her comprehension (asking the individual to point to articles in the room or parts of the body), reading (asking the person to read available print), and writing (asking the person to make up and write a sentence).

Delirium

is a disturbance of consciousness that develops over a short period and may be attributable to a medical condition.

A delusion

is a firm, fixed, false belief that is irrational and that a person clings to despite objective evidence to the contrary.

Dietary Reference Intakes

is a group name that includes four separate reference values that are based on the concepts of probability and risk ■ Updated RDAs ■ Estimated Average Requirement (EAR) ■ Adequate Intake (AI) ■ Tolerable Upper Intake Level (UL)

Hematoma

is a localized collection of extravasated blood. Blunt force often causes a hematoma.

Depersonalization

is a loss of identity, feeling of being estranged, or perplexed about one's own identity and meaning of existence.

pulse

is a measurement of the force at which blood is ejected against the arterial wall. Some examiners use a scale rating from 0 to 4+ for the strength of a pulse

The Mini-Cog

is a newer instrument that screens for cognitive impairment, often found with dementia

A social phobia

is a persistent and irrational fear of being in social situations.

A hypochondriac

is a person who is morbidly worried about his/her own health and/or feels sick with no actual basis for that assumption. An obsession is an unwanted, persistent thought or impulse in which logic will not purge him/her from his/her consciousness and is intrusive and senseless.

Circumlocution

is a roundabout expression, substituting a phrase when one cannot think of the name of the object.

A phobia

is a strong, persistent, irrational fear of an object or situation; the person feels driven to avoid it.

glycemic response

is actually influenced the amounts of fat, fiber, and acid in the food; the degree of processing; the method of preparation; the amount eaten; the degree of ripeness (for fruits and vegetables); and whether other foods are eaten at the same time.

AUDIT questionnaire

is an acronym for Alcohol Use Disorders Identification Test. It is a 10-item questionnaire that organizes the patient history of alcohol use.

Thrombomodulin

is an endothelial cell receptor that binds thrombin. When thrombomodulin and thrombin form a complex, the conformation of the thrombin molecule is changed so that it readily activates protein C while losing platelet-activating and protease activity. On binding to thrombomodulin, thrombin switches role from a potent procoagulant into an anticoagulant. In the normal physiological state this is important because endothelial cells produce thrombomodulin that binds any circulating thrombin, preventing clot formation in undamaged vessels.

Patterned injury

is an injury caused by an object that leaves a distinct pattern on the skin or organ.

Coagulation Cascade:

is balanced by the fibrinolytic cascade, which regulates the breakdown of fibrin and fibrinogen and prevents excessive formation of thrombi. Normal clotting and hemostasis represents a dynamic interaction between the coagulation cascade, the fibrinolytic cascade, and platelet function.

hot/cold theory of health and illness

is based on the four humors of the body: blood, phlegm, black bile, and yellow bile. These humors regulate the basic bodily functions, described in terms of temperature, dryness, and moisture. The treatment of disease consists of adding or subtracting cold, heat, dryness, or wetness to restore the balance of the humors. explanatory model with origins in the ancient greek humoral theory. the four humors of the body regulate bodily functions in terms of temperature dryness and moisture. treatment is adding or subtracting heat dryness cold or wetness to restore balance or humors. Foods and illnesses are classified as hot or cold by their perceived effects on the body not on physical characteristics

Diaphragm of stethoscope

is best for listening to high-pitched sounds such as breath, bowel, and normal heart sounds

Bell of the stethoscope

is best for soft, low-pitched sounds such as extra heart sounds or murmurs.

Abrasion

is caused by the rubbing of the skin or mucous membrane. A friction abrasion

Psychological abuse

is defined as behaviors that result in mental anguish.

Financial neglect

is defined as the failure to use the assets of the older person to provide services needed by him or her.

"Healthy" or "normal" BMI

is defined numerically as 18.5 to 24.9. Values above and below this range are associated with increased health risks.

flight of ideas

is demonstrated by an abrupt change, rapid skipping from topic to topic, and practically continuous flow of accelerated speech. symptom of mania that involves an abruptly switching in conversation from one topic to another

Preschoolers' communication

is direct, concrete, literal, and set in the present. Therefore, health care providers should use short, simple sentences with a concrete explanation for any unfamiliar equipment that will be used on the child.

attention span.

is evaluated by assessing the individual's ability to concentrate and complete a thought or task without wandering. Giving a series of directions to follow is one method used to assess attention span.

Mental status functioning

is inferred through the assessment of an individual's behaviors. It cannot be directly assessed like the characteristics of the skin or heart sounds.

Contusion

is injury to tissues without breakage of skin

Bleeding time

is one of several measures of platelet plug formation in vivo and may detect abnormalities of platelet function where blood count and platelet numbers are normal. The commonly used method uses the Ivy template. Bleeding should stop spontaneously after 3-8 minutes. Bleeding time is prolonged with thrombocytopenia or platelet dysfunction.

Ambivalence

is the existence of opposing emotions toward an idea, object, or person.

Pathology

is the investigation of structural alterations in cells, tissues, and organs, which can help identify the cause of a particular disease.

Perseveration

is the persistent repeating of verbal or motor response, even with varied stimuli.

pathophysiology

is the study of the underlying changes in body physiology (molecular, cellular, and organ systems) that result from disease or injury

Residue

is undigested food, including fiber, that makes up stool

broad nasal speculum

is used to visualize the nares is short

Fetoscope

is used to auscultate fetal heart tones.

Sulfonylureas

is used to deter alcohol ingestion in people with chronic alcoholism. This reaction includes vomiting and hypertension. In addition, may enhance the action of the antidiabetic drug, causing increased hypoglycemia.

Ophthalmoscope

is used to examine the internal eye structures. It can compensate for nearsightedness or farsightedness, but it will not correct astigmatism. Rotating the lens selector dial brings the object into focus.

diet "nutrition therapy"

is viewed as a short-term punishment to endure until a normal pattern of eating can resume

objective data

is what the health professional observes by inspecting, percussing, palpating, and auscultating during the physical examination. information that is seen, heard, felt, or smelled by an observer; signs

Blocking

is when a person experiences sudden interruption in train of thought and unable to complete sentences which seems r/t strong emotion. a failure to retrieve information that is available in memory even though you are trying to produce it

Circumstantiality

is when a person talks excessively with unnecessary detail and delays reaching the point. Their sentences have a meaningful connection but are irrelevant. Speech that is delayed in reaching the point and contains excessive or irrelevant details

The osteocyte

is within a space in the hardened bone matrix called a lacuna. contains long, thin cytoplasmic extensions, canaliculi, providing communication with osteoblasts lying on the bone surface. Another form of extracellular communication used by osteocytes is through transmembrane channels called gap junctions, which connect the cytoplasm of adjacent cells. Osteocytes are the most abundant cells found in bone and have numerous functions, including acting as mechanoreceptors and synthesizing certain matrix molecules, playing a major role in controlling osteoblast differentiation and production of growth factors, and maintaining bone homeostasis.5 As the major source of sclerostin, RANKL, and osteoprotegerin (OPG), osteocytes are thought to be key regulators of both bone formation and bone resorption.6-8 They also help concentrate nutrients in the matrix. Osteocytes obtain nutrients from capillaries in the canaliculi, which contain nutrient-rich fluids. Through exchanges among these cells, hormone catalysts, and minerals, optimal levels of calcium, phosphorus, and other minerals are maintained in blood plasma. One of the osteocyte's primary functions is to act as a mechanoreceptor

Clanging

is word choice based on sound, not meaning, and includes nonsense rhymes and puns.

Lymphoid organs

links the hematologic to the immune system primary and secondary

secondary lymphoid organs

lymph nodes, spleen, tonsils, aggregated lymphoid nodules, appendix

Unintentional physical neglect

may occur, despite good intentions, and is the failure of a family member or caregiver to provide basic goods or services.

American Indians

may seek assistance from a medicine man or shaman

Goniometers

measure joint range of motion.

Mobility & Exercise -Focused assessment

mobility status history Elements: 1. Daily Activity Level- describe the activities you normally carry out and types, frequency, duration of physical exercise 2. Endurance- describe how much and what type of activity make you tired, Hx of dizziness, dyspnea, frequent pauses, increase respiratory rate when exercising 3 Exercise/ Fit goals - what exercises/fit goals are you currently working on?. Knowledge of the benefits, attitude, motivation 4. Physical/Mental health alteration- are there any physical or mental health problems affecting you carry out exercises. Decrease strength or endurance (MI, CHF, COPD, GI disorder, cancer) Neuromuscular impairment (multiple sclerosis, spinal injury), pain, depression, musculoskeletal, cognitive

Ideal alignment

natural curve in spine through pelvis

Percussion child's lung

notes that are loud in amplitude, low in pitch, of a booming quality, and long in duration

caloric excess

obesity, refers to weight more than 20% above ideal body weight.

Echolalia

occurs when a person imitates or repeats phrases occurs when a person imitates or repeats another's words or phrases, often with a mumbling, mocking, or a mechanical tone.

mild malnutrition

of 80% to 90% of ideal weight suggests

Naturalistic Theory

or holistic, perspective holds that the forces of nature must be kept in natural balance. The naturalistic theory believes that illness is caused by an imbalance or disharmony in the forces of nature.

caloric excess,

overnutrition is caused by the consumption of nutrients, especially calories, sodium, and fat, in excess of body needs. can lead to obesity, refers to weight more than 20% above ideal body weight. is a risk factor for heart disease, type II diabetes, hypertension, dyslipidemia, carotid artery atherosclerosis, stroke, gallbladder disease, sleep apnea, certain cancers, and osteoarthritis.

Which serous membrane lines the abdominopelvic cavity and covers its organs? The Digestive System

peritoneum Membrane surrounding the organs in the abdomen. (Abdominal cavity)

magicoreligious theory

perspective holds that supernatural forces dominate and cause illness or health. The magicoreligious theory believes that illness is caused by supernatural forces such as God or other supernatural powers.

Exercise

planned, structured, repetitive movement intended to improve or maintain physical fitness. Improves mood, enhances memory, slowed neurodegenerative decline. Enhance (1) cardio.pulmonary ENDURANCE (2) muscle.skeleton FLEXIBILITY (3) bone integrity WEIGHT Nutritionally, the most important factor for increasing muscle mass is calories. If calorie intake meets or exceeds need, muscle growth can occur over a range or protein intakes (Tipton and Witard, 2007). When calorie intake is inadequate, an increase in muscle mass is not likely (ADA, 2009a). A high calorie intake needed to support intense training generally provides adequate protein for muscle anabolism. Exercise in the form of resistance training, also called weight or strength training, is necessary to increase muscle mass. To improve muscle strength and power in most healthy adults, the American College of Sports Medicine recommends two to four sets of 8 to 10 repetitions each of strength-training exercises of all major muscle groups at least twice a week (Garber et al., 2011). When an increase in muscular endurance is the goal, 15 to 20 repetitions are recommended. Athletes and body builders do more. Protein and amino acids are commonly believed to be the most important factor in building muscle mass because muscle is composed of protein

authority responses

promotes dependency and inferiority.

Vasodilator prostaglandins

prostacyclin, has an important role in limiting clot formation and in preventing adhesion of platelets to normal vascular endothelium.

protein malnutrition

protein deficiency with appropriate caloric intake Kwashiorkor disease is due to diets that may be high in calories but contain little or no protein (e.g., low-protein liquid diets, fad diets, and long-term use of dextrose-containing intravenous fluids). The serum albumin would be less than 3.5 g/dL.

grains (carbs)

provides antioxidants, iron, zinc, copper, magnesium, B vitamins, E vitamins, fiber, and phytochemicals.

primary lymphoid organs

red bone marrow and thymus gland

Ethnicity

refers to a social group that may possess shared traits, such as common geographic origin, migratory status, religion, language, values, traditions, or symbols and food preferences.

Assimilation

refers to taking on the characteristics of the dominant culture.

Etiology

refers to the study of the cause of disease.

stabbing

result in a penetrating, sharp, cutting injury that is deeper than it is wide.

Ventricular hypertrophy

results in the capillaries of the heart becoming more extensive as well as numerous This is shown when the QRS complex is altered.

Four glycoproteins are present in bone:

romotes calcification, osteoblast formationLaminin -Stabilizes basement membranes in bone Albumin- Transports essential elements to matrix; maintains osmotic pressure of bone fluid α-Glycoproteins - plays a significant role in calcification and also may facilitate bone resorption by activating osteoclasts Osteocalcin-which binds preferentially to crystallized calcium

The Clinical Institute Withdrawal Assessment of Alcohol Scale, Revised (CIWA-Ar)

screening tool use to assess for symptoms of alcohol withdrawal. The tool is quantified to measure the progress of withdrawal.

Global aphasia

spontaneous speech is absent or reduced to a few stereotyped words or sounds and comprehension is absent or reduced to only a person's own name and a few select words.

symptoms of uncomplicated alcohol withdrawal.

start shortly after the cessation of drinking, peak at the second day, and improve by the fourth or fifth day. include coarse tremors of the hands, tongue, and eyelids; anorexia; nausea and vomiting; autonomic hyperactivity (e.g., tachycardia, sweating, elevated blood pressure); and transient hallucinations, among other symptoms (see Table 6-7).

Hirudin

the anticoagulant secreted by leeches into the wound they create to prevent clotting. alternative anticoagulant use in patients with heparin-induced thrombocytopenia. acts by irreversible binding to the active site of thrombin.

Greatest risk for cardiac disease and death

the lifesyle factors ■ High BP ■ Increased waist circumference ■ Hyperglycemia ■ Low high-density lipoprotein (HDL) cholesterol

Heart - Capillaries

type of heart vessel Also called exchange vessels Exchange materials between blood and tissues Materials include dissolved gases, nutrients, waste product

A myocardial infarction can usually be diagnosed:

with an ECG and blood studies Insert Large Bore IV and draw initial Cardiac Enzymes RATIONALE IV access is important for administration of medications, possible interventions if angina worsens, and any scans that may be needed to rule out thrombosis. Cardiac enzymes further serve to rule out Myocardial Infarction and can give an indication to the extent of myocardial damage. * Troponin I * CK * CK-MB * Myoglobin Monitor Cardiac Enzymes: * Troponin I * Creatine Kinase-MB (CKMB) RATIONALE The values of these enzymes are based on your institutional laboratory technique. If they are elevated it indicates that the cardiac muscle is stressed out or injured. * Troponin I is an enzyme that helps the interaction of myosin and actin in the cardiac muscle. When necrosis of the myocyte happens, the contents of the cell eventually will be released into the bloodstream. * Troponin can become elevated 2-4 hours after in ischemic cardiac event and can stay elevated for up to 14 days. * Creatine Kinase MB: This enzyme is found in the cardiac muscle cells and catalyses the conversion of ATP into ADP giving your cells energy to contract. When the cardiac muscle cells are damaged the enzyme is eventually released into the bloodstream. * CKMB levels should be checked at admission, and then every 8 hours afterwards.

To decrease gas, a person should avoid:

• Beans, beer, sodas • Cucumbers, cabbage, onions, spinach • Brussels sprouts, broccoli, cauliflower • Most dairy products, corn, radishes"

To loosen stool, a person should eat:

• Chocolate, raw fruits and vegetables • Spiced foods, greasy or fried foods • Prunes, grapes, leafy green vegetables

A Patient's Sleep History

• Have the patient describe his or her specific problem. • Have the patient describe his or her symptoms and alleviating factors. • Assess the patient's normal sleep pattern. • Assess the patient's normal bedtime rituals. • Assess for current or recent physical illnesses. • Assess for current or recent emotional stress. • Assess for possible sleep disorders. • Assess the patient's current medications and their possible effects on sleep."

Warning Signs of Impending Stroke

• Numbness of face, arm, or leg • Weakness of face, arm, or leg • Difficult speaking or understanding • Sudden decreased or blurred vision • Loss of balance • Dizziness when accompanied by any of the above signs"

Interviewing Strategies

• Open-ended questions: "How can I help you?" • Summarizing statements: "Coughing and wheezing seem to be your concerns." • Reflective statements: "You seem out of breath." • Leading questions: "Is your sputum green?" • Focused questions: "Are you short of breath now?" • Clarification: "When did you last take you inhaler?" • Restating: "So you said your shortness of breath became worse yesterday?" • Encouraging: "Tell me more about your breathing."

Timed Urine Tests

• Quantitative albumin (24 hr) Determines albumin lost in urine as a result of kidney disease, hypertension, or heart failure • Amino acid (24 hr) Determines the presence of congenital kidney disease • Amylase (2, 12, and 24 hr) Determines the presence of disease of the pancreas • Chloride (24 hr) Determines loss of chloride in cardiac patients on low-salt or no-salt diets • Concentration and dilution Determines the presence of diseases of the kidney tubules • Creatinine clearance (12 and 24 hr) Determines the ability of the kidneys to clear creatinine • Estriol (24 hr) Measures this hormone in women with high-risk pregnancies because of diabetes • Glucose tolerance (12 and 24 hr) Determines malfunctions of the liver and pancreas • 17-Hydroxycorticosteroid (24 hr) Determines functioning ability of the adrenal cortex • Urinalysis (random times) Determines levels of bacteria, white blood cell count, red blood cell count, pH, specific gravity, protein, and bilirubin • Urine culture (random times) Determines the amount and type of bacteria in the urine • Urine sensitivity (random times) Determines the antibiotics to which the microorganisms will be sensitive or resistant • Urobilinogen (random times) Determines the presence of obstruction of the biliary tract"

Facts about Influenza

• Symptoms include fever, headache, dry mouth, fatigue, sore throat, and muscle aches. • Up to 20% of Americans get the flu each year. • Influenza in conjunction with pneumonia is the sixth leading cause of death in the United States among older adults. • A person cannot catch influenza from a vaccine. • Because influenza viruses can change from year to year, an annual influenza shot is needed each fall. • The best time to receive an influenza vaccine is October through December. • Influenza vaccine will not protect from other illnesses, such as colds, bronchitis, and the stomach influenza or gastritis. • Vaccinations can prevent up to 50% of the 140,000 hospitalizations and 80% of the 300,000 deaths that occur each year. • Influenza can worsen heart and lung diseases and diabetes. • Influenza can lead to pneumonia

Preventing Urinary Catheter Infections

• Use good handwashing techniques before handling. • Avoid raising the drainage bag above the bladder. • Allow urine to drain freely into the bag. • Perform good perineal care on the patient. • Secure the catheter per procedure. • Empty the drainage bag at least every 8 hours. • Avoid kinking the tubing. • Clean the spigot thoroughly before and after use. • Avoid dragging the drainage bag on the floor."

Droplet Precautions

• Used for infectious agents larger than 5 micrometers • Droplets from the mucous membranes of the nose or mouth • Droplets from coughing, sneezing, or talking • Droplet contracted within 3 feet or less Ex: TUBERCULOSIS , German Measles (Rubella), For a respiratory MRSA infection and for when MRSA is found in tracheal secretions."

Transmission-Based Precautions

• Used for patients known or suspected to be infected with specific pathogens • There are three subgroups of transmission-based precautions. • Subgroups can be combined for diseases with multiple transmissions routes. • Subgroups are to be used in addition to standard precautions.

Standard Precautions

• Used to help prevent nosocomial infections • Used when working with all patients • Replaces the universal precautions and the blood and body precautions • Applies to blood, all body fluids, secretions, excretions (except sweat) • To be used even if blood is not visible • Also applies to nonintact skin and mucous membranes • Designed to reduce the risk of transmission of microorganisms

Thrombolytic drugs affecting hemostasis coagulation modifier drugs

■ Alteplase ■ Anistreplase ■ Reteplase ■ Streptokinase ■Urokinase

Asthma- therapeutic interventions

■ Dilate airways 1. Bronchodilators * Give before steroids to open airways 2. Corticosteroid

Reteplase

■ Drug class: Thrombolytic ■ Peak time: 5-10 min ■ Duration: unknown ■ Half-life: 13-16 min Drugs affecting hemostasis

indicative of malnutrition

■ Loss of subcutaneous fat (triceps, chest), muscle wasting (quadriceps, deltoids), ankle edema, sacral edema, ascites. These abnormal findings are subjectively assessed as mild, moderate, or severe.

Osteoarthritis -therapeutic intervention

■ Treat pain 1. Administer analgesia * Aspirin * Monitor for GI bleed * Acetaminophen * Max 4grams/day * Topical analgesia applied to painful joint * COX-2 inhibitors (NSAIDS) * Celecoxib (Celebrex) * Corticosteroids- injected

Willebrand's disease,

■ it increases the levels of clotting factor VIII. ■ use desmopressin coagulation modifier drugs

Carbohydrates

■ table simple sugar ■comple sugar: fiber, starch grains, ■added sugar: such as sweetened ready-to-eat cereals, muffins, and pancakes

Urinary Tract Infection -therapeutic interventions

■* Treat infection 1. Antibiotics * TMP-SMZ (Bactrim) * Obtain cultures prior to giving antibiotics * Give more focused antibiotic after cultures come back ■* Treat pain and discomfort 1. Pyridium - urinary analgesic * Turns urine dark yellow/orange * Treat bladder spasticity 1. Belladona

congestice heart failure - therapeutic interventions

■* Treatment goals: 1. Decrease afterload 2. Increase cardiac output 3. Increase renal blood flow 4. Decrease edema 5. Prevent progression of failure ■* Common medications prescribed 1. ACE inhibitors (medications ending in 'pril') 2. Angiotensin Receptor blockers endin 'sartan') 3. Beta blockers (medications ending in 'olol') 4. Decrease work load of the heart 5. Diuretics (Loop, Thiazide, Potassium-sparing) * Decrease volume overload 6. Inotopics (Digoxin) * Increase contractility of the heart * Increase cardiac output 7. Sympathomimetic (Dopamine, Dobutamine) * Increase contractility of heart 8. Vasodilators * Open up vessels and decrease symptoms 9. Anti-dysrhythmic

Antibiotic-Resistant Pathogens

■Acinetobacter baumannii Multidrug-resistance (MRAB) ■Clostridium difficile Clindamycin resistant; fluoroquinolone-resistant Cipro (ciprofloxacin) and Levaquin (levofloxacin) ■Escherichia coli 80% of the bacteria are resistant to one or more drugs ■Enterococcus Vancomycin-resistant (VRE) ■Mycobacterium tuberculosis Multiple-drug resistant (MDR-TB) ■Salmonella Resistant to nine different antibiotics ■Staphylococcus aureus Methicillin resistant (MRSA), linezolid resistant, vancomycin resistant (VRSA); CA-MRSA (community-acquired MRSA) ■Streptococcus pyogenes (group A strep) Macrolide resistant ■Streptococcus pneumoniae Penicillin resistant

polar amino acids

■Asparagine ■Glutamine ■Serine ■Threonine ■Tyrosine

nonpolar amino acids

■Aspartate ■Cysteine ■Phenylalanine ■Glycine ■Isoleucine ■Leucine ■Methionine ■Proline ■Valine ■Tryptophan

Macronutrient

■Carbohydrate 1. Functions: Energy, body temperature 2. Food source: Simple: sugars, fruits, nuts Complex: grains, potatoes milk, starch, fiber . ■Fiber ■Linoleic acid ■Alpha-linolenic acid ■Protein 1. Functions: Tissue growth, tissue repair 2. Food source: .Meat, fish, eggs, milk, poultry, beans, peas, nuts ■Water 1. Function- Carries nutrients, regulates body processes, lubricates joints 2. Food source- "Liquids, most fruits and vegetables.

Nursing Diagnosis: manifested by

■DX- list signs and symptoms ■OUTCOME- consider individualizing plan of care ■INTERVENTION- individualized intervention ■EVALUATION- did the symptoms resolve Individualizing Care Plans When a patient care plan is developed, the following considerations are needed to individualize the plan to meet each patient's needs: • Age • Gender • Level of education • Developmental level • General health status (current and before illness) • Disabilities (physical or mental) • Strength • Support systems • Cultural background • Spiritual background • Emotional status

Clopidogrel (Plavix) MOA

■Drug Class -Antiplatelet ■Peak time - one hr ■Half/life - 8 hr Drugs affecting Hemostasis -Blocks the adenosine diphosphate (ADP) receptor on the platelet membrane, preventing the platelet from "connecting" with other platelets. -The reduction in aggregation, or making the platelet "slippery" is nearly the same effect that occurs with ASA, only through a different mechanism.

heparin

■Drug Class -anticoagulant ■Peak time 2-4 hr (Subcuta) 5-10 min (intramus ■ Duration 8-12 hr 2-6 hr ■Half/life 1-2 hr Drugs affecting Hemostasis

Dalteparin

■Drug class -anticoagulant ■Peak time 3-5 hrs ■Duration -12 hrs ■Half/life - 3.5 hr Drugs affecting Hemostasis

SOY PRODUCTS that are protein-sources are

■Edamame: parboiled fresh soybeans, usually in the pod, sold refrigerated or frozen ■Meat analogs: imitation burgers, hot dogs, bacon, chicken fingers, etc., made from soy, not meat ■Miso: fermented soybean paste ■Soy cheese: cheese made from soy milk; can substitute for sour cream, cream cheese, or other cheese ■Soy milk: the liquid from soaked, ground, strained soybeans; available in plain and in chocolate and vanilla flavors. ■Soy nuts: whole soybeans that have been soaked in water and then baked ■Soy nut butter: crushed soy nuts blended with soy oil to resemble peanut butter Soy sprouts: sprouted soybeans ■Tofu: soybean curd ■Tempeh: caked fermented soybeans ■Textured vegetable protein (TVP): soy flour modified to resemble ground beef when rehydrated

Dietary Intake

■How many meals and snacks do you eat in a 24-hour period? ■Do you have any food allergies or intolerances, and, if so, what are they? what are the reactions? ■What types of vitamin, mineral, herbal, or other supplements do you use and why? ■What concerns do you have about what or how you eat? ■For clients who are acutely ill: How has illness affected your choice or tolerance of food? ■Who prepares the meals? ■Do you have enough food to eat? ■How much alcohol do you consume daily?

COPD Therapeutic Interventions

■Prevent infection (pneumonia/influenza) 1. Yearly immunization ■Dilate the bronchioles 1. Beta-Adrenergic Agonist 2. Anticholinergics 3. Corticosteroids 4. Fluticasone

Acute Renal Failure -therapeutic interventions

■Restore/improve blood flow to kidneys 1. Dopamine given at lower doses restores blood flow to kidneys * Higher doses are given to help with heart problems 2. Loop diuretic- Furosemide (Lasix) * Helps remove toxins * Prevents oliguric phase of renal failure ■Prevent hypertension- which can cause renal damage 1. ACE inhibitors * 'pril' medications- example, Lisinopril ■Prevent GI bleed- common complication of renal failure 1. H2-receptor antagonist * Famotidine, Ranitidine 2. Proton Pump inhibitors * Pantoprazole, Omeprazole ■Treat hyperkalaemia (because it can lead to lethal arrhythmias) 1. Calcium Chloride 2. Insulin 3. Bicarbonate 4. Glucose

Types of Isolation Precautions

■Standard Precautions ■Transmission-Based Precautions ■Airborne Precautions Droplet Precautions Contact Precautions

Atrial Fibrillation -therapeutic interventions

■Treat arrhythmia 1. Amiodarone (given as bolus first) ■ Reduce ventricular rate 1. Beta blockers * Metoprolol 2. Diltiazem 3. Cortizem ■ Prevent clot formation- decrease risk of stroke, myocardial infarction and pulmonary embolism 1. Anticoagulant * Aspirin * Heparin

Myocardial Infarction - therapeutic interventions

■Treat pain 1. Morphine ■Improve blood flow to the heart 1. Nitro-glycerine- potent coronary vasodilator ■Break up clots 1. Aspirin 2. Fibrinolytic therapy * t-PA ■ Treat dysrhythmias ■ Decrease oxygen demands on the heart 1. Beta blockers (metoprolol, atenolol) ■ Reduce the risk of future MI's occurring 1. ACE inhibitor 2. Anticoagulant - aspirin or abciximab

Pneumonia -Therapeutic Interventions

■Types of pneumonia 1. Viral vs Bacterial 2. Community acquired vs Hospital acquired ■* Prevention 1. Influenza/Pneumonia Vaccine for +60 ■* Treat infection 1. Antibiotics:Cephalosporins, Floroquinolones ■* Treat chest pain ( w/breathing dyspnea) 1. Administer appropriate analgesia ■ Improve breathing 1. Bronchodilators 2. Albuterol (beta adrenergic agonist) * Works by opening airway ■ Treat fever 1. Antipyretics/Acetominophen * adult dose- 650mg PO q6 hrs PRN

possible indications of bleeding problems related to the anticoagulation therapy.

■restless and confused. ■change in pulse rate or rhythm, blood pressure, LOC

A home care nurse is working with a client who has osteoporosis. Which guideline for home safety would be most appropriate in preventing fractures in this client? Keep most items on lower shelves and in cupboards under the counter to avoid reaching high Cover all cords with a throw rug taped to the floor Keep lights on in the stairwell, with switches accessible at the top and bottom Avoid using the shower and only use the bathtub to bathe every day

✅Keep lights on in the stairwell, with switches accessible at the top and bottom A client with osteoporosis is at risk of injury and bone fractures if he or she falls. When working with a client who has osteoporosis and who lives at home, the nurse should teach about the safest methods of home maintenance to avoid hazards and falls that could result in a broken bone. This involves keeping items within easy reach (not too low or too high), avoiding extension cords, using a hand bar in the shower for stability, and keeping lights on in the stairwell with switches easily accessible. Keep most items on lower shelves and in cupboards under the counter to avoid reaching high Lower shelves is appropriate for placement of items, but not too low in cupboards. Items should be within easy reach. Cover all cords with a throw rug taped to the floor Throw rugs increase the chance of tripping and falling, and covering cords is a fire risk. Avoid using the shower and only use the bathtub to bathe every day Showering with a nonslip mat and a shower seat is a safe way to get clean.

A client with magnesium depletion has laboratory testing which showed a calcium level of 7.9 mg/dL. Which nursing interventions are most appropriate in this situation? Select all that apply. Monitor intake and output Perform laboratory testing for vitamin K levels Increase intake of dairy products and leafy green vegetables Encourage bed rest and frequent naps Include vitamin D supplements with calcium

✅Monitor intake and output Monitoring overall intake and output is going to help the nurse address the client's calcium issue, as calcium can be affected by diarrhea, some diuretics, and intake of calcium rich and vitamin D rich foods ✅Increase intake of dairy products and leafy green vegetables Dietary intake of calcium-rich foods will help increase a client's calcium level. ✅Include vitamin D supplements with calcium This client has a low calcium level, which is called hypocalcemia. It can be caused by a number of factors, including dehydration, thyroid problems, or deficiency in vitamin D. Nursing interventions include increasing calcium intake and vitamin D supplements and monitoring overall intake and output. Perform laboratory testing for vitamin K levels Vitamin K is not related to magnesium and calcium. Encourage bed rest and frequent naps Rest is not a solution to low calcium levels. Adding calcium to the body is what will bring the level back up.

A 70-year-old client has been diagnosed with osteoporosis after suffering an arm fracture when bumping into a counter. The client's husband asks the nurse why his wife's arm was broken so easily. Which of the following statements are correct about osteoporosis? Select all that apply. The effects of osteoporosis cause the bone structure to weaken Osteoporosis causes near-constant pain for the client Bone reaches maximum growth in a person's 20's, but stops earlier with osteoporosis Osteoporosis does not stop the healing process of a broken bone The femur is usually the most common bone fractured with osteoporosis Nursing Care Plan for Osteoporosis

✅Osteoporosis does not stop the healing process of a broken bone Despite the increase of fractures with this condition, osteoporosis does not prevent healing when a fracture occurs, although research has shown that a delay in the healing process can occur. ✅The effects of osteoporosis cause the bone structure to weaken Osteoporosis occurs when bones are demineralized from calcium and phosphorus loss. Bones are weakened as the tissue breaks down, which increases the risk of fractures. A client with osteoporosis typically does not have pain unless they have suffered a fracture. The femur is usually the most common bone fractured with osteoporosis The bones most affected by osteoporosis are the wrist, hips and vertebral column. Bone reaches maximum growth in a person's 20's, but stops earlier with osteoporosis Bone growth reaches a peak in a person's 20's. After this, bone loss begins slowly, and reaches the fastest point in women during the first few years after menopause. Bone growth does not stop earlier in osteoporosis. Osteoporosis causes near-constant pain for the client The client does not have pain from osteoporosis unless injury occurs.

A 55-year-old woman has been prescribed ibandronate sodium (Boniva) for the prevention of osteoporosis. The nurse is giving the client her prescription and should include which of the following information about taking this drug? The client must not have suffered a fracture in the past The client should not have a vitamin D deficiency The client must have had osteoporosis for at least five years prior The client cannot have a history of cancer

✅The client should not have a vitamin D deficiency Ibandronate is a type of medication known as a bisphosphonate, which works for the treatment of osteoporosis. Ibandronate works by changing how bone is formed and broken down in the body, slowing the progression of osteoporosis. The client who takes this drug must be able to sit up for at least 60 minutes after administration. It is also not intended for those who have vitamin D deficiency because this affects calcium levels in the body. Additionally, the client with kidney disease should not take bisphosphonates because of the risk of renal toxicity. The client cannot have a history of cancer Bisphosphonates are approved for clients with a history of cancer. However, clients currently being treated for cancer AND undergoing a dental procedure have a risk of osteonecrosis of the jaw, and should not take bisphosphonates. The client must have had osteoporosis for at least five years prior This drug is to treat any woman with postmenopausal osteoporosis. The client does not need to wait five years to begin taking ibandronate. The client must not have suffered a fracture in the past Often a diagnosis of osteoporosis is made because a client suffered a fracture. Being fracture free is not a prerequisite of taking ibandronate. ⏺⏺Nursing Interventions and Rationales Assess and manage pain Patients often complain of back pain as the vertebra collapse or pain from a fracture. Pain control is essential for the patient to participate in rehab. Initiate fall precautions to prevent injury Falls and injuries are more difficult to heal with osteoporosis as the bone takes longer to regenerate. Provide assistance with ambulation, remove rugs or fall hazards and maintain a clear and well-lit path. Support fracture stabilization If patient is in cast or splint, make sure the device fits properly and assess for skin integrity and circulation. Administer medications appropriately Bisphosphonates (alendronate, ibandronate

The student nurse understands that when the ventricles are depolarizing, which action is taking place? Select all that apply. A. The atria are resting and receiving blood from the ventricles B. The ventricles are resting C. The atria are contracting and pumping blood to the ventricles D. The ventricles are pumping blood to the lungs E. The ventricles are pumping blood to the body

✅The ventricles are pumping blood to the body During ventricular depolarization, the ventricles are sending blood to the body. ✅The ventricles are pumping blood to the lungs During ventricular depolarization, the ventricles are contracting and sending the blood to the lungs. The ventricles are resting The ventricles are depolarizing (contracting) not repolarizing (resting). The atria are resting and receiving blood from the ventricles Atria do not receive blood from the ventricles. The atria are contracting and pumping blood to the ventricles The atria are already in a depolarized state during ventricular depolarization.

A client with osteoporosis asks the nurse why it is important to take vitamin D. Which response by the nurse is correct? Vitamin D helps prevent constipation from increased calcium intake Vitamin D minimizes the risk of kidney stones Vitamin D reduces excretion of calcium in the kidneys Vitamin D improves the absorption of calcium

✅Vitamin D improves the absorption of calcium Taken with calcium, vitamin D aids with calcium absorption which is essential for bone building and slowing the progression of osteoporosis. Vitamin D reduces excretion of calcium in the kidneys It is taken to enhance calcium absorption, and does not have an affect on how much is excreted by the kidneys. Vitamin D helps prevent constipation from increased calcium intake Increased calcium intake can lead to constipation, but vitamin D will not improve this condition. Vitamin D minimizes the risk of kidney stones Renal calculi are common in clients with hypercalcemia, and hypercalcemia is common in clients with osteoporosis. However, vitamin D does not have an effect on the occurrence of renal calculi Nursing Care Plan for Osteoporosis

Education regarding over-the-counter drug when taking Coumadin

"Aspirin products may result in increased bleeding." patient who will be taking warfarin sodium (Coumadin) at home, coagulation modifier drugs

Split S2

(inspiratory = normal, expiratory = abnormal) Causes A. Inspiratory: Intrathoracic pressure changes, increased venous return B.Expiratory: RBBB, Left ventricular ectopy, VSD, Pulmonary stenosis, Pulmonary hypertension Location -Pulmonic

Calcium reabsorption inhibitors

(keeps Ca IN the bones) Phosphorus Calcitonin Bisphosphonates NSAIDs

Use of Heat

*Local Effects Increased skin temperature Vasodilatation, which increases oxygen and nutrients to area Increased muscle relaxation Decreased stiffness and spasm Increased peristalsis *Indications Stiffness Arthritis Pain *Contraindications Trauma because of increased bleeding Edema because of increased fluid retention Malignant tumors because of increased cell growth Burns because of increased cell damage Open wounds because of increased bleeding Acute areas such as appendix because of possible rupture Testes because of destruction of sperm Sensory-impaired patients because of increased chance of burns Confused patients because of increased chance of injury

Use of Cold†

*Local Effects Vasoconstriction, which decreases oxygen to area Decreased metabolism and thus decreased oxygen needs Decreased fluid in area and thus decreased swelling Decreased pain through numbness Impaired circulation and increased cell death caused by lack of oxygen *Indications Sprains Fractures Swelling Bleeding *Contraindications Open wounds because of decreased chance of healing Impaired circulation because of increased chance of injury Sensory-impaired patients because of increased chance of injury Confused patients because of increased chance of injury

Sepsis -therapeutic interventions

*Treat rapid drop in blood pressure 1. V fluid bolus (volume replacement) 2. 2L of NS * Treat infection 1. After obtaining blood cultures 2. Start antimicrobials 3. Vancomycin, Clindamycin * Improve perfusion 1. Vasopressors 2. Levophed, Phenylephrine, Epinephrine, Vasopressin

Structures of the Brain

*cerebral cortex *hypothalamus *pituitary gland *pons *reticular formation *medulla *spinal cord *mid Brain *cerebellum *arbor vitae *pineal gland *skull *corpus callosum (of cerebrum)

body alignment/posture

-Brings body parts into position that promotes optimal balance and body function -Person maintains balance as long as line of gravity passes through center of gravity and base of support -reduce risk of injury to allow mobility without strain, decrease muscle energy, homeostasis Feet parallel, 8 in apart, Slightly bend arms, Shoulder down Weight bearing Full/independent -pt perform task safety Partial - pt requires stand by help None/dependent

Low fat diet

-reduces calories from fat and minimizes cholesterol intake Description Little butter, cream, whole milk, or eggs Patient Complaints:Gallbladder, liver, or heart disease

Name the three types of lipoproteins that are always found in the blood. Nutrition and Metabolism

.Very-low-density lipoproteins (VLDL), low-density lipoproteins (LDL), and high-density lipoproteins (HDL)

Strength of pulse scale

0: Absent, not palpable 1+: Pulse diminished, barely palpable 2+: Expected/normal 3+: Full pulse, increased 4+: Bounding pulse

Advantages of breastfeeding

1) Passive transfer of T-cell immunity: Decreased risk of allergies and gastro intestinal and respiratory infections 2) Psychological/emotional: Maternal-infant bonding, reduced likelihood of overfeeding, less cost, 3 Cow's milk may cause GI/Kidney issues is a poor source of iron/Vitamin C/E

mechanical soft diet

1. Description -Regular diet but chopped or ground 2. Patient Complaints -Difficulty chewing

bland diet

1.Description -No spicy food 2.Patient Complaints-Ulcers or colitis

low residue diet

1.Description No bulky food, apples, or nuts 2Patient complaints-Rectal disease

Typhoid fever

7-10 day incubation General symptoms: *fever *abdominal/muscle pain *GI problems *loss of appetite *Diarrhea <->constipation Transmission through contaminated water, urine, or feces Considerations Good hygiene, sanitary water, proper sewage care, and vaccinations

Group the 31 pairs of spinal nerves into five subdivisions. The Nervous System

8 cervical nerve pairs, 12 thoracic nerve pairs, 5 lumbar nerve pairs, 5 sacral nerve pairs, and 1 coccygeal pair of spinal nerves PERIPHERAL NERVOUS SYSTEM

A focused database is used: A) to monitor progress of short-term or chronic health concerns. B) for a limited or short-term problem usually consisting of one problem, one cue complex, or one body system. C) to evaluate the etiology (cause) of disease. D) to perform a thorough or comprehensive health history and physical examination.

A focused database is conducted for a limited or short-term problem, not for a patient with several chronic problems. In a focused or problem-centered database, the nurse collects a "mini" database, which is smaller in scope than the completed database. This mini database primarily concerns one problem, one cue complex, or one body system.

Nosocomial infection

A hospital-acquired infection (not present or incubating on admission)

activated partial thromboplastin time (aPTT)

A laboratory test used to measure the effectiveness of heparin therapy coagulation modifier drugs The aPTT is a measure of the intrinsic system. Hence, factors XII, XI, IX, and VIII are tested, in addition to factors common to both systems. A surface activator (e.g. kaolin), phospholipid, and calcium ions are added to plasma, which should produce a clot within 30-40 seconds. Both PT and aPTT are normally correctable by adding normal plasma to the plasma being tested. Failure of this maneuver to correct PT or aPTT suggests that an inhibitor of coagulation is present (e.g. FDPs, heparinoids).

Status epilepticus (SE)

A state of continuous or frequently reoccurring seizures lasting 30 minutes or more"

Why is a state of negative nitrogen balance also considered a state of "tissue wasting?" Nutrition and Metabolism

A state of negative nitrogen balance is often referred to as a state of "tissue wasting" because more of the tissue proteins are being catabolized than are being replaced by protein synthesis.

antiplatelet drug

A substance that prevents platelet plugs from forming hypersensitivity-bronchospasm ■ Aspirin ■ Dipyridamole ■ Ibuprofen ■ Ketorolac ■ Pentoxifylline ■ Plavix ■ Sulfinpyrazon ■ Ticlopidine coagulation modifier drugs

Protamine sulfate MOA

A substance that reverses the effect of heparin Positively charged molecule that binds negatively charged heparin. coagulation modifier drugs

vitamin K

A substance that reverses the effect of warfarin sodium needed for proper function of clotting factors II, VII, IX, and X and proteins C and S (K is for Koagulation) After the use of vitamin K for warfarin toxicity, warfarin resistance will occur for up to 7 days; thus the patient cannot be anticoagulated by warfarin during this period. In such cases, eitherheparin or an LMWH may need to be added to provide adequate anticoagulation if necessary. coagulation modifier drugs

metabolic syndrome

A syndrome marked by the presence of usually three or more of a group of factors (as high blood pressure, abdominal obesity, high triglyceride levels, low HDL levels, and high fasting levels of blood sugar) that are linked to increased risk of cardiovascular disease and Type 2 diabetes.

Protein

A three dimensional polymer made of monomers of amino acids. The primary structure is the sequence of amino acids. Secondary structure incorporates more information into the protein by organizing some sections along the chain into regular conformations,The side chains project outward into the surrounding solution or environment. The tertiary structures of thousands of proteins are known at the atomic level, and have been described by several methods, including X-ray and electron diffraction of protein crystals (Fig. 1.3b), and mass spectrometry and nuclear magnetic reso- nance of small proteins in solution

A client with type 1 diabetes has an above-the-knee amputation because of severe lower extremity arterial disease. What is the nurse's primary responsibility two days after surgery when preparing the client to eat dinner? Checking the client's serum glucose level Assisting the client out of bed into a chair Placing the client in the high-Fowler position Ensuring the client's residual limb is elevated

A. Checking the client's serum glucose level

A nurse is formulating a teaching plan for a client recently diagnosed with type 2 diabetes. What interventions should the nurse include to decrease the risk of complications? . Select all that apply. Examine the feet daily Wear well-fitting shoes Perform regular exercise Powder the feet after showering Visit the primary healthcare provider weekly Test bathwater with the toes before bathing

A. Examine the feet daily B. Wear well-fitting shoes C. Perform regular exercise

The nurse develops a teaching plan for a client with diabetes who has been diagnosed with lower extremity arterial disease (LEAD). What measures should the nurse include to increase arterial blood flow to the extremities? Exercises that promote muscular activity Meticulous care of minor skin breakdown Elevation of the legs above the level of the heart Soaking the feet in hot water each day

A. Exercises that promote muscular activity

Which long-acting insulin mimics natural, basal insulin with no peak action and a duration of 24 hours? A. Insulin glargine (Lantus) B. Insulin glulisine (Apidra) C. Regular insulin (Humulin R) D. NPH insulin

A. Insulin glargine (Lantus)

The nurse is explaining insulin needs to a client with gestational diabetes who is in her second trimester of pregnancy. Which information should the nurse give to this client? Insulin needs will increase during the second trimester. Insulin needs will decrease during the second trimester. Insulin needs will not change during the second trimester. Insulin will be switched to an oral antidiabetic medication during the second trimester.

A. Insulin needs will increase during the second trimester.

A nurse is caring for an older client who had non-insulin dependent diabetes for 15 years that progressed to insulin-dependent diabetes 2 years ago. What common complications of diabetes should the nurse assess for when examining this client? Select all that apply. Leg ulcers Loss of visual acuity Increased creatinine clearance Prolonged capillary refill in the toes Decreased sensation in the lower extremities

A. Leg ulcers B. Loss of visual acuity D. Prolonged capillary refill in the toes E. Decreased sensation in the lower extremities

Pramlintide (Symlin) is prescribed as supplemental drug therapy to the treatment plan for a patient with type 1 diabetes mellitus. What information should the nurse include when teaching the patient about the action of this medication? Pramlintide slows gastric emptying. Pramlintide increases glucagon excretion. Pramlintide stimulates glucose production. Pramlintide corrects insulin receptor sensitivity.

A. Pramlintide slows gastric emptying

Describe how the antidiuretic hormone (ADH) prevents excessive loss of water. The Urinary System

ADH targets cells of the distal and collecting tubules and causes them to become more permeable to water. When this happens, water is permitted to flow osmotically out of the tubule and into the interstitial fluid toward equilibrium. As water is reabsorbed under the influence of ADH, the total volume of urine is reduced by the amount of water removed from the tubules.

An adult client presents to the emergency department with a chief complaint of heartburn. What is the nurse's immediate priority? A.Start an IV B.Inspect the client's mouth C. Find out the last time the client had oral intake D. Call for an EKG

ANS :D ✅Call for an EKG When a client has complaints of discomfort or pain in the chest area, including heartburn, the client will need an EKG. This client will be simultaneously having vital signs taken and the EKG will be ready shortly after, but calling for the EKG can occur as soon as the healthcare team learns of the chief complaint. Find out the last time the client had oral intake While this question will be asked if it is determined that the client will need a surgical procedure, it is not a priority when the client presents to the ED. Start an IV The client will need an IV line if one was not inserted in the field, but an EKG to rule out MI must be called for immediately after learning of the chest pain complaint. Inspect the client's mouth This will be a necessary assessment if the client will be undergoing general anesthesia, but is not a priority at this time.

The nurse is caring for a client who has been diagnosed with nephrotic syndrome. Which of the following assessment findings would be inconsistent with this diagnosis? A. Hyperalbuminemia B.Proteinurea C. Edema D. Hypoalbuminemia

ANS A * ✅ Hyperalbuminemia This question is asking for which assessment finding is NOT expected in the client with nephrotic syndrome. The client with nephrotic syndrome will have a LACK of serum protein. This is due to excessive loss of protein through the urine due to damaged kidneys. Hyperalbuminemia is an excess of protein in the blood, which is inconsistent with nephrotic syndrome. * Edema Edema is an expected finding with nephrotic syndrome. * Hypoalbuminemia Hypoalbuminemia is an expected finding with nephrotic syndrome due to excessive loss of protein through the urine because of damaged glomerulus. * Proteinurea In nephrotic syndrome excessive amounts of protein are excreted in the urine, causing proteinurea.

The nurse is assessing a client with multiple sclerosis and notes the client is experiencing tremors and muscle weakness. The client is scheduled for several tests that require transport off the floor, as well as a physical therapy session during the shift. Which of the following nursing actions would be most helpful for this client? * Contact physical therapy to delay the session until the client is finished with the tests * Coordinate with various disciplines to perform tests back-to-back * Administer analgesics as ordered to provide comfort for the day's activities * Encourage activity independence for ADLs prior to going for tests

ANS A * ✅Coordinate with various disciplines to perform tests back-to-back A client with multiple sclerosis experiences fatigue and weakness due to sensorimotor spasticity. As the disease progresses, this worsens, so the nurse must help manage the client's care to avoid exhausting the client. Clustering care is important for the fatigued client, which will involve coordinating with various disciplines to efficiently move the client from one test to another in a row, rather than taking multiple trips off the floor. * Contact physical therapy to delay the session until the client is finished with the tests Rather than delaying the physical therapy session, the nurse should inform physical therapy about what the client's schedule will be for the day so that the physical therapist can incorporate a session while getting the client ready for transport to tests. * Encourage activity independence for ADLs prior to going for tests Since the client will be overexerted with activities for the day, it is not a time for activity independence. * Administer analgesics as ordered to provide comfort for the day's activities While analgesics will be helpful for this client to be comfortable, but this does not address the client's fatigue and weakness.

The nurse is working a shift in the hospital and is assigned to care for a client with advanced multiple sclerosis. Which of the following would most contribute to the client's overall level of functioning? * Encourage the client's activity independence * Educate on fall safety measures for the home * Teach the client to lower the water heater temperature at home * Maximize fluid intake to prevent constipation

ANS A * ✅Encourage the client's activity independence A client with multiple sclerosis will lose their independence over time, due to the degenerative nature of the disease. While all of these interventions are helpful for the client, encouraging the client to have activity independence will MOST contribute to the client's overall level of functioning. * Educate on fall safety measures for the home While this is an important concept to teach the client, it does not help with overall functioning for the client in the hospital. * Maximize fluid intake to prevent constipation Bowel and bladder training is an important part of nursing care for the client with multiple sclerosis, but is just one element of many parts to the management of multiple sclerosis. The most effective contribution to the client's overall level of functioning is to encourage activity independence. * Teach the client to lower the water heater temperature at home While this is an important concept to teach the client, it does not help with overall functioning for the client in the hospital.

The nurse is assessing a client who presents to the emergency department with complaints of abdominal pain. Which sign or symptom would indicate to the nurse that the client is having an emergency? * Sudden relief of the pain * Dark tarry stools * Bright red blood in stools * Pain upon palpation

ANS A * ✅Sudden relief of the pain Appendicitis is painful until the appendix ruptures, at which point the pain will suddenly stop. It is an emergency when the client has abdominal pain and then has a sudden relief of pain that is unrelated to receiving pain medication. If the appendix ruptures, the client will get peritonitis and possibly sepsis if not emergently treated. * Dark tarry stools Dark tarry stools indicate an upper GI bleed, which is unrelated to appendicitis. This has the potential to be an emergency, but only if other signs and symptoms are also present, such as a low hemoglobin/hematocrit, thready pulse, dizziness, and/or additional signs of shock. * Bright red blood in stools Bright red stools indicate a lower GI bleed, which is unrelated to appendicitis. This could considered an emergency if there were other signs and symptoms present, but since the question does not list additional signs and symptoms, this is not the correct option. * Pain upon palpation This is an expected finding with appendicitis. It does not indicate that the appendix has burst.

A nurse is assisting a bedridden client with moving up in bed and repositioning. Which of the following interventions would most likely prevent friction against this client's skin while transferring? Select all that apply. A. Apply a barrier dressing over high-risk areas B. Use a slide sheet while moving the client C. Avoid dragging a client across the bed when moving D. Raise the head of the client's bed before repositioning E. Ensure the client receives adequate hydration and nutrition

ANS A ,B, C * ✅Use a slide sheet while moving the client A slide sheet protects a client from the shearing force of the client's body weight moving across the bed. ✅Avoid dragging a client across the bed when moving Dragging a client across the bed creates friction and shear which can damage a client's skin. Ensure the client receives adequate hydration and nutrition This is an important intervention to lessen the client's risk of skin breakdown intrinsically, but does not prevent friction. ✅Apply a barrier dressing over high-risk areas Friction may cause skin trauma as the skin rubs against another surface and can eventually lead to skin breakdown. The nurse can best avoid traumatizing the skin through friction forces by using a slide sheet or transfer board to move the client, avoiding dragging the client across a bed or chair, and evaluating high-risk areas and applying a barrier to protect these sites. Raise the head of the client's bed before repositioning The head of the bed should be lowered for repositioning to spread out the client's weight over their whole body surface, and make sliding easier.

A nurse is teaching a client with bursitis about how to walk with a cane. The nurse knows that the client understands how to use the cane properly when the client demonstrates: A . Advancing the cane at the same time as the affected leg B. Gripping the cane with the fingertips C. Leaning forward and supporting weight against the cane D. Holding the cane on the affected side

ANS A ✅Advancing the cane at the same time as the affected leg A client who experiences pain with walking may benefit from the use of a cane. The nurse may need to provide education about how to use some assistive devices, including the proper use of a cane. The client should hold the cane on the unaffected side and keep it close to the body to avoid leaning too heavily on it. When taking a step, the client should advance the cane forward at the same time as the affected leg. Holding the cane on the affected side The client needs to hold the cane on the unaffected side of the body, but move the cane along with the affected leg. Leaning forward and supporting weight against the cane The client needs to stand up straight and support their weight on their strong extremity as much as possible to minimize the risk of a fall. Gripping the cane with the fingertips The cane should be held so that the palm of the hand is touching it to maximize grip and support.

A 67-year-old client tells the nurse that he used to run marathons, but no longer has the stamina to run as far. Which of the following changes in exercise tolerance are associated with the aging process? Select all that apply. A.Decreased lean muscle mass B.Decreased testosterone secretion C.Increased total body water D.Increased bone loss E.Increased oxidative capacity

ANS A, B, D Increased total body water As a person ages, water decreases intracellularly, and therefore total body water decreases. ✅Decreased lean muscle mass As a person ages, stamina and ability to exercise is decreased. The normal aging process results in decreased muscle mass, decreased hormone secretion, and increased bone loss that can negatively impact a person's ability to exercise. Increased oxidative capacity A person's oxidative capacity is the ability of the muscles to use oxygen. This decreases with age. ✅Decreased testosterone secretion This contributes to decreased ability to exercise with age. ✅Increased bone loss This contributes to decreased ability to exercise with age.

A nurse is changing the colostomy bag for a client with a new colostomy. Which elements of skin care should the nurse provide during this process? Select all that apply. * * Avoiding ripping the pouch off of the client's skin * * Drying the skin thoroughly before reapplying the pouch * * Avoiding adhesive remover or water when removing the pouch from the skin * Using sterile gauze to wipe the stoma and absorb exudate * * Cleaning around the stoma with warm water and a washcloth

ANS A, B, E * ✅Avoiding ripping the pouch off of the client's skin A client who uses a colostomy bag is at risk of skin breakdown around the ostomy site. The pouch should be removed by pushing gently down on the skin with one hand, while pulling up on the pouch with the other hand. * * * ✅Cleaning around the stoma with warm water and a washcloth Skin care includes inspecting the skin and stoma site carefully, washing the skin with soap and water. Ensuring the skin is dry before applying a new pouch. * ✅Drying the skin thoroughly before reapplying the pouch Skin care also includes ensuring the skin is dry before applying a new pouch. If the skin is irritated, it is possible that the pouch opening is too large, excessively exposing the surrounding skin to feces which leads to skin irritation and breakdown. Using sterile gauze to wipe the stoma and absorb exudate Sterile supplies are not necessary with this area, as it is routinely exposed to feces. Avoiding adhesive remover or water when removing the pouch from the skin These items are okay to use for pouch removal.

The client has a sudden onset of decreased urine output, proteinuria, increased serum BUN, and increased creatinine. What are some potential causes? Select all that apply. A. BPH B.Nephrotoxic drugs C. Hypertension D. Acute glomerulonephritis E. Ureteral stone

ANS A,B, E * ✅BPH This symptom is related to the urinary tract and kidney dysfunction. Causes include stones, acute glomerulonephritis, BPH, and nephrotoxic drugs, but other causes are cancer of the prostate or kidneys. ✅ Nephrotoxic drugs This symptom is related to the urinary tract and kidney dysfunction. Causes include stones, acute glomerulonephritis, BPH and nephrotoxic drugs, but other causes are cancer of the prostate or kidneys. * ✅Ureteral stone This symptom is related to the urinary tract and kidney dysfunction. Causes include stones, acute glomerulonephritis, BPH, and nephrotoxic drugs, but other causes are cancer of the prostate or kidneys. * Acute glomerulonephritis This symptom is related to the urinary tract and kidney dysfunction. Causes include stones, acute glomerulonephritis, BPH and nephrotoxic drugs, but other causes are cancer of the prostate or kidneys. * Hypertension Hypertension does not cause sudden-onset renal issues.

A nurse is helping a client who will be going home with an ostomy after surgery. Which of the following demonstrates how the nurse would prepare the client for ostomy care at home? Select all that apply. Educating the client about signs and symptoms of stoma dermatitis Talking with the client about alterations in body image * Overseeing appropriate measurement of the stoma barrier * Instructing the client to change the pouch each day * Teaching the client to scrub the stoma with soap and water

ANS A,B,C * ✅Talking with the client about alterations in body image Home education about ostomy care is important to prepare the client to manage health conditions at home. When caring for an ostomy, the nurse should be sure the client knows that having an ostomy may impact body image. The client should also understand the practical aspects of caring for an ostomy, such as by checking for irritation and understanding how to appropriately fit the barrier and pouch. * . ✅Educating the client about signs and symptoms of stoma dermatitis Home education about ostomy care is important to prepare the client to manage health conditions at home. When caring for an ostomy, the nurse should be sure the client knows that having an ostomy may impact body image. The client should also understand the practical aspects of caring for an ostomy, such as by checking for irritation and understanding how to appropriately fit the barrier and pouch. ✅Overseeing appropriate measurement of the stoma barrier Home education about ostomy care is important to prepare the client to manage health conditions at home. When caring for an ostomy, the nurse should be sure the client knows that having an ostomy may impact body image. The client should also understand the practical aspects of caring for an ostomy, such as by checking for irritation and understanding how to appropriately fit the barrier and pouch. Teaching the client to scrub the stoma with soap and water The pouch should only be changed every two to three days, and the client should not scrub the stoma, but gently wash it with warm soapy water. Instructing the client to change the pouch each day The pouch should only be changed every two to three days, and the client should not scrub the stoma, but gently wash it with warm soapy water.

The nurse is caring for a client with an ileostomy. During assessment of the client, the nurse notes that the pouch opening is 1/2 inch larger than the stoma site. Which of the following poses a risk for this client? * Leakage and odor from the site * Peristomal skin breakdown * Stoma pain and burning * Pouch system detachment

ANS B * ✅ Peristomal skin breakdown The opening of the skin barrier and pouch should be just 1/8 inch larger than the stoma. If the opening is any larger than this, the fecal matter will irritate the surrounding skin and can cause skin breakdown. When applying the skin barrier, the nurse should ensure that the skin around the stoma is clean and dry. The stoma is measured, and an opening 1/8 inch larger is applied to the skin using a skin barrier paste. The bag is then attached to the barrier. * Pouch system detachment The pouch system is generally reliable and would not detach due to an incorrect opening size. The bag should be emptied when it is 1/3 to 1/2 full, and if this is managed appropriately and the pouch is correctly attached, it should stay in place. * Leakage and odor from the site An incorrect opening size would not result in leakage and odor. Rather, leakage and odor occurs if the bag is not properly pressed into the flange, or the bag is not sealed appropriately. * Stoma pain and burning The stoma itself does not contain pain receptors. The skin around the stoma can become painful if the opening is too large, but not the stoma itself.

A client with trigeminal neuralgia complains to the nurse that she feels extreme pain in her face any time she touches it. Which of the following medications would most likely be ordered for pain control for this condition? * Morphine * Carbamazepine * Ketorolac * Meperidine

ANS B * ✅Carbamazepine The pain caused by trigeminal neuralgia (TN) is pain originating in the nerves; therefore anticonvulsants are an effective treatment. Trigeminal neuralgia is a painful condition that affects the trigeminal nerve (the 5th cranial nerve) in the face. The condition causes severe pain with even a slight touch or by performing routine activities such as combing the hair or brushing the teeth. * Morphine It is an opioid, and opioids are typically ineffective in providing relief for nerve pain. * Meperidine It is an opioid, which is a class of drugs that has been shown ineffective for nerve pain management. Additionally, meperidine is highly addictive and has many dangerous side effects. It would not be prescribed for long term management of pain due to its many precautions. * Ketorolac It is an NSAID, which has been shown to be ineffective in the treatment of nerve pain.

Your patient had an ileostomy placed 2 weeks ago. Which bowel pattern would be expected for this patient? * Output every 2-3 hours * Continuous output * Continent, controlled output * Output once per day

ANS B * ✅Continuous output An ileostomy is placed in the ileum of the small intestine. Therefore output will be liquid or semi-soft and will be continous. As the peristalsis occurs in the small intestine, stool output will be pushed out - therefore clients should expect continuous output throughout the day and night. * Output every 2-3 hours Because output from the ileostomy moves with peristalsis, it will be continuous, not intermittent. * Continent, controlled output Clients with an ileostomy do not have control over their output like they might with a colostomy or other types of bowel ostomies. It is a continuous output. * Output once per day Output from an ileostomy is liquid or semi-soft and flows continuously with peristalsis, NOT once daily.

The nurse is assessing a client after having a colostomy placed the day before. On assessment, which of the following findings would be the MOST concerning? * Red stoma * Hypoactive bowel sounds * Mucous in stool * Blood in stool

ANS B * ✅Hypoactive bowel sounds Hypoactive bowel sounds post colostomy are concerning for intestinal obstruction. The colostomy was done the day prior so bowel sounds should be returned to normal post surgery by this time. This needs to be reported to the healthcare provider as soon as possible. * Blood in stool Some blood in the stool should be expected in the first couple of days after surgery. * Mucous in stool Mucus will be expected for a couple of days as the client begins to increase their diet. * Red stoma A red stoma is a desired finding. This means the stoma is getting plenty of blood flow. A concerning finding would be a bluish or dusky stoma, which would indicate perfusion problems.

A client is dying and has become unconscious but is still breathing. While standing next to the client's bed, his sister says, "I always wanted to tell him that I loved him, but it was so hard for me. Now it is too late." Which response of the nurse is most appropriate? A. He probably already knows B. You can still tell him. Hearing is the last sense to go so he may hear you C. Next time don't wait. Tell the person when you have the chance D. We should talk about this somewhere else and not at the bedside

ANS B * ✅You can still tell him. Hearing is the last sense to go so he may hear you A client who is dying and has become unconscious may still be able to hear what is going on. Studies have shown that hearing is the last sense to go when a person dies. The nurse should not make the client's family member feel guilty about her feelings, but should instead encourage her to communicate. * He probably already knows By stating this, the nurse is making an assumption that may not be true. * We should talk about this somewhere else and not at the bedside The nurse should encourage family members to be at ease and discuss their feelings if they would like to do this. * Next time don't wait. Tell the person when you have the chance By saying this, the nurse is sending the message that it is too late, when it may not be too late for the family member to talk to the dying person.

A client is being cared for after a traumatic brain injury. During an initial assessment, the nurse performs the Glasgow Coma Scale and gives the client a score of 8. Which of the following responses from the nurse is appropriate to manage the client's respiratory rate? A. Remove oxygen and assess the client's pulse oximetry B. Prepare for intubation C. Administer oxygen via non-rebreather mask D. Administer oxygen via nasal cannula

ANS B ✅Prepare for intubation The Glasgow Coma Scale is a method of determining the neurological state of a client who has suffered an injury that could impact brain function. The lower the level determined with the GCS, the more likely the client has a significant brain injury and is most likely unconscious and not responding. A GCS score less than 9 indicates that the client is impaired enough that he may not be able to breathe on his own without assisted ventilation. When the score is less than 9, the nurse should prepare for intubation (which will be at the discretion of the provider). * Administer oxygen via non-rebreather mask The assumption of a GCS less than 9 is that the client is unable to protect their own airway, as most of these clients do not have a gag reflex. A non-rebreather mask would not be enough to sustain ventilation in this client. The client may or may not need oxygen based on their vital signs, but could rapidly become airway compromised. * Administer oxygen via nasal cannula Oxygen via nasal cannula is not enough to sustain ventilation in a client with a GCS score <9. * Remove oxygen and assess the client's pulse oximetry A room air trial is inappropriate for a client with a GCS score <9. This client needs breathing support.

Incorrect * Question 7 of 10 The student nurse understands that when the heart ventricles are depolarizing, the atria are doing which of the following? A. Filling the ventricles with blood B. Repolarizing C. Depolarizing D. Contracting

ANS B ✅Repolarizing During ventricular depolarization, the atria are repolarizing (resting) Depolarizing The ventricles are depolarizing rather than the atria. The atria are repolarizing during this time. Contracting The ventricles are contracting. The atria and ventricles do not contract at the same time. Filling the ventricles with blood The ventricles are contracting not filling with blood.

A nurse is caring for a client who had knee surgery and is morbidly obese. The client has difficulties with mobility and needs two-person assistance when getting out of bed. Which of the following best describes appropriately education about preventing skin breakdown? A. Teaching the client that because of his immobility, skin breakdown cannot be prevented but can be treated B. Teaching the client that increased pressure on certain body areas can cause poor circulation and skin breakdown C. Telling the client to closely monitor the area behind the knee for skin breakdown after knee surgery D. Educating the client that increased adipose tissue results in an increased risk for skin breakdown at bony prominence sites

ANS B * ✅Teaching the client that increased pressure on certain body areas can cause poor circulation and skin breakdown An immobile client is at higher risk of skin breakdown in abnormal areas for multiple reasons. In this situation, the client's obesity can contribute to increased pressure in certain areas that can reduce circulation to the tissues and can cause skin wounds. The nurse should remind the client to reposition himself at least every 2 hours. The nurse should also provide skincare and help with movement and repositioning as needed. Educating the client that increased adipose tissue results in an increased risk for skin breakdown at bony prominence sites The more adipose tissue covering a bony prominence, the less likely the client will have skin breakdown in that area. Telling the client to closely monitor the area behind the knee for skin breakdown after knee surgery The anatomy of a leg and the direction a knee bends, the area behind the knee is not a high risk area for skin breakdown. Teaching the client that because of his immobility, skin breakdown cannot be prevented but can be treated With good nursing management of the immobile and obese client, skin breakdown can absolutely be prevented.

A client who had an indwelling catheter has developed a urinary tract infection that has spread to the kidneys. The nurse should include which of the following information as part of her teaching? A. The client should restrict fluids until the infection has cleared B. The client will need to confirm the diagnosis with urine testing, kidney x-ray, and a voiding cystourethrogram C. The client will need surgery for removal of infected tissue D. The client can continue to recover with antibiotics at home

ANS B * ✅The client can continue to recover with antibiotics at home An indwelling catheter places a client at a higher risk of a urinary tract infection (UTI). A kidney infection can develop if the UTI is left untreated, when the bacteria ascend from the lower urinary tract. While a kidney infection, or pyelonephritis, can be very serious and may require hospitalization if complications develop, most clients can recover at home on oral medication after a loading dose of IV antibiotics. * The client will need surgery for removal of infected tissue Pyelonephritis could also involve an abscess, which if severe enough, could warrant surgery. However, this does not occur with the majority of pyelonephritis cases. * The client should restrict fluids until the infection has cleared The client with a kidney infection should be encouraged to take in fluids to help flush the infection from the kidneys. * The client will need to confirm the diagnosis with urine testing, kidney x-ray, and a voiding cystourethrogram A kidney infection is usually diagnosed with a urine culture and possible blood cultures.

The nurse is caring for a client who has been diagnosed with a urinary tract infection. Which of the following isolation precautions should be implemented? A. Droplet B. Standard C. Airborne D. Contact

ANS B Standard ✅A client with a UTI should be placed on standard precautions, since the question does not state the type of bacteria that is present. If the bacteria was a resistant drug organism such as methilicin-resistant staph aureus (MRSA), the client would be placed on contact precautions. Since the question does not specify the pathogen, contact precautions is incorrect. * Contact The client simply needs to be on standard precautions. * Airborne The client simply needs to be on standard precautions. * Droplet The client simply needs to be on standard precautions.

The nurse is caring for a client who has trigeminal neuralgia. Which of the following are appropriate nursing interventions? Select all that apply. * * Give narcotics for pain management * Avoid triggers for the attacks * Assist in glycerol injection of the nerve * Administer carbamazepine for symptom relief * Have the client chew on the side of the pain

ANS B,C,D * ✅Avoid triggers for the attacks The client with trigeminal neuralgia should avoid triggers for the attacks, including extreme temperatures and light stimulation of the area * * ✅Assist in glycerol injection of the nerve The nurse can assist in glycerol injection of the nerve, which may help block pain signals. * ✅Administer carbamazepine for symptom relief An anticonvulsant such as carbamazepine (Tegretol) can be given to help relieve pain. Give narcotics for pain management The first line drugs for nerve pain are anticonvulsants. Narcotics are not very effective for neuralgias. Have the client chew on the side of the pain The client should chew on the opposite side of the pain.

A client who has sustained a head injury is being evaluated in the emergency room. The nurse performs a neurological assessment and notes that the client is somnolent. Which of the following describes an somnolent client response? Select all that apply. A. Alert and following commands B. Falls asleep between stimulation C. Difficult to arouse D. Needs stimulation to follow commands E. Does not respond at all

ANS B,C,D ✅Difficult to arouse A client who is somnolent is difficult to arouse and has trouble staying awake and following commands. This type of response may be seen in some types of head injuries. ✅Needs stimulation to follow commands A client who is somnolent is difficult to arouse and has trouble staying awake and following commands. This type of response may be seen in some types of head injuries. ✅Falls asleep between stimulation A client who is somnolent is difficult to arouse and has trouble staying awake and following commands. This type of response may be seen in some types of head injuries. Alert and following commands This describes a normal level of consciousness. Does not respond at all This describes a comatose state.

The nurse is caring for a client in end stage renal disease. The client has an internal arteriovenous fistula for dialysis. Which of the following interventions are appropriate in caring for the client with an AV fistula? Select all that apply. A.Monitor client for neurological changes B.Palpate the fistula to ensure patency C. Avoid checking blood pressure on the extremity with the fistula D. Assess for prolonged Q-T intervals on the ECG E.Monitor for signs of blood clotting

ANS B,C,E * ✅Palpate the fistula to ensure patency An internal arteriovenous (AV) fistula is created by surgically joining an artery and a vein in the arm. This is permanent access for the client with chronic kidney disease. Care for the client with an AV fistula includes preventing and monitoring for complications. "Feel the thrill and hear the bruit" is important to assess for fistula patency. " ✅Avoid checking blood pressure on the extremity with the fistula Teach the client to notify all health care personnel of the presence of a fistula, because the affected extremity should not be used for blood pressure checks, IV lines, blood draws, or injections. ✅Monitor for signs of blood clotting The client should be monitored for signs of clotting, infection, heart failure and arterial steal syndrome. Heart failure can occur if too much arterial blood is shunted into the venous system. Arterial steal syndrome can occur if not enough arterial blood reaches the hand due to shunting into the venous system, compromising circulation to the hand. Assess for prolonged Q-T intervals on the ECG An AV fistula does not affect the Q-T interval. However, the client is at risk for heart failure if too much blood enters the venous system and overloads the heart. Monitor client for neurological changes An AV fistula does not affect the client's neurological status. However, if a client with kidney disease begins to demonstrate neurological changes, the nurse should monitor laboratory values and notify the provider.

The nurse is admitting a client with a history of chronic kidney disease. Which of the following lab values would you NOT expect to see? * Creatinine 2.1 mg/dL * BUN 42 mg/dL * BUN 7 mg/dL * K 6.1 mEq/L

ANS C * ✅BUN 7 mg/dL A client with kidney problems will have an increased BUN rather than a low-to-normal BUN. * BUN 42 mg/dL This is a high blood urea nitrogen level, which is expected in the client with chronic kidney failure. * Creatinine 2.1 mg/dL This is a high creatinine level, which is expected in the client with chronic kidney failure. * K 6.1 mEq/L This electrolyte would build up in the blood with chronic filtering problems, and since normal potassium is 3.5-5 mEq/L, this is an expected lab value.

The nurse is caring for a client with glomerulonephritis. Which of the following orders would the nurse question for this client? A.Plasmapheresis B.Protein restriction C. Fluid resuscitation of 30 ml/kg of 0.9% normal saline D. Dialysis

ANS C * ✅Fluid resuscitation of 30 ml/kg of 0.9% normal saline A client with glomerulonephritis will retain fluid, so IV fluids would not be indicated. * Dialysis Dialysis is an appropriate order for the client with glomerulonephritis, because dialysis will filter the blood and remove excess fluid. * Plasmapheresis A client with excess fluid will benefit from plasmapheresis. This order is appropriate for the client. * Protein restriction This is appropriate for the client with glomerulonephritis, because when protein is broken down, it produces the waste product urea. Urea cannot be filtered appropriately during glomerulonephritis, so the client must restrict protein.

A 56-year-old client is suffering from interstitial nephritis and is seen in the hospital for care. The client's creatinine levels are elevated and the client has poor skin turgor and dry mucous membranes upon exam. The nurse ensures that the client does not receive any nephrotoxic medications that would worsen the condition. Which medication should be avoided? A. Amantadine B. Abilify C. Gentamicin D. Combivir

ANS C * ✅Gentamicin Nephrotoxic medications are those that can cause damage to the kidneys. When a client is at risk, nephrotoxic drugs could cause such damage that a client goes into a state of acute renal failure. Categories of nephrotoxic medications include aminoglycoside antibiotics (gentamicin is in this category), antineoplastics, and nonsteroidal antiinflammatory drugs. * Abilify This drug is an atypical antipsychotic drug, used to manage symptoms of certain mood disorders. It has multiple side effects, but does not cause nephrotoxicity. * Combivir This drug is a nucleoside reverse transcriptase inhibitor, which is used to prevent HIV from multiplying in the body. It can cause mitochondrial toxicity, but is not known for being nephrotoxic. * Amantadine This is an antiviral drug that is used to treat influenza. Amantadine overdose can cause toxicity to the cardiac, respiratory, and central nervous system, but is not nephrotoxic.

A client who has been incontinent of stool needs skin cream to prevent skin breakdown. How does application of skin barrier prevent wound development? * It fights off fungi that may have accumulated on the skin * It prevents skin maceration from exposure to stool * It prevents breakdown from ammonia properties found in stool * It promotes beneficial bacteria on the skin surface to fight off infection

ANS C * ✅It prevents skin maceration from exposure to stool A client who is incontinent of stool may be more likely to develop skin breakdown when the skin is exposed to stool for long periods. The skin may become macerated from the moisture and acid in the stool, which causes it to become soggy and more prone to breakdown. Skin emollient provides a barrier against moisture that leads to maceration. * It promotes beneficial bacteria on the skin surface to fight off infection Skin cream does not promote any type of bacterial growth. It is used as a barrier. * It prevents breakdown from ammonia properties found in stool Stool and urine combine to create ammonia. If this comes into contact with the skin, it will cause irritation. Skin cream acts as a barrier to prevent irritants from contacting the skin. * It fights off fungi that may have accumulated on the skin Skin cream is a barrier protection. It does not have an anti-fungal effect

The nurse is caring for a client with a BUN of 28. The nurse recognizes this as a sign of dysfunction of which of the following? A. Spleen B. Liver C. Kidneys D. Heart

ANS C * ✅Kidneys Kidneys excrete blood urea nitrogen (BUN) and if they are in acute or chronic failure, they will not eliminate BUN and it will remain high in the blood. Normal BUN range is 7-20 mg/dL. * Heart The organs responsible for filtering blood urea nitrogen are the kidneys. * Liver The organs responsible for filtering blood urea nitrogen are the kidneys. * Spleen The organs responsible for filtering blood urea nitrogen are the kidneys.

A 61-year-old client recently had surgery where the surgeon placed an ostomy appliance. Which of the following teaching principles would the nurse include when instructing this client about ostomy care? * Empty the bag every 24 hours to remove all elimination from bowel movements * When applying a new bag, cut an opening ½ inch smaller than the size of the stoma before putting it on * Remove the bag and wash the stoma site with soap and water at least once every three to seven days * Never put any kind of lotion or skin barrier near the ostomy site

ANS C * ✅Remove the bag and wash the stoma site with soap and water at least once every three to seven days A client who has a new ostomy appliance requires extensive teaching on the principles of its care as well as lifestyle changes that may be necessary. The nurse should teach the client to empty the bag when it becomes half full and to place barrier ointment on the skin surrounding the site in order to adhere the pouch to the skin. The nurse should also teach the client to remove the bag at least once every three to seven days and to wash the stoma site with soap and water and let fully dry before attaching a new pouch. * Empty the bag every 24 hours to remove all elimination from bowel movements The bag should be emptied when it is half full, not on a time schedule. * Never put any kind of lotion or skin barrier near the ostomy site The skin will need a barrier applied to protect it from breakdown and to encourage adherence of the pouch. * When applying a new bag, cut an opening ½ inch smaller than the size of the stoma before putting it on If the opening is too small, it will not work appropriately and will cut off circulation to the stoma. The opening should be cut to fit closely to the size of the stoma. A measuring tool may be used.

A client receiving medication as an anti-infective starts showing signs of nephrotoxicity. The nurse knows that which of the following are signs of nephrotoxicity? A.. Increased urine output B.. Low blood pressure C. Fluid retention D. Decreased urine output E. Hearing loss

ANS C,D * Increased urine output Damaged renal cells cause a decrease in urine production, not an increase. * * ✅ Decreased urine output Damaged renal cells cause a decrease in urine production. * * ✅ Fluid retention Damaged renal cells cause fluid retention. Hearing loss This symptom is related to ototoxicity. Low blood pressure Nephrotoxicity can cause an irregular heartbeat and might cause high blood pressure to occur because of the increase in fluid retention.

The nurse is assessing a client in the urgent care clinic who is complaining of burning with urination. The urinalysis shows that the client has a urinary tract infection (UTI). Which of the following medications is an appropriate medication to treat this client's UTI? * A.Cimetidine B. Clopidogrel C.* Captopril D. Cephalexin

ANS D * ✅Cephalexin This is an anti-infective medication used for skin infections, pneumonia, UTIs, and otitis media. * Captopril This is an ACE inhibitor that treats hypertension and CHF. * Cimetidine This medication is an anti-ulcer H2 antagonist that treats GERD ulcers, and for the prevention of GI bleeding. * Clopidogrel Clopidogrel is an antiplatelet agent used to manage cerebrovascular accident, myocardial infarction, and peripheral vascular disease.

A nurse is educating a client with a traumatic brain injury (TBI) about drug and alcohol use. During the teaching, the nurse is providing information about how alcohol affects the brain in a client with a TBI. Which information is correct? * It takes much more alcohol to achieve the same effects in a person who has had a brain injury * Most people with a brain injury overdose the first time they use drugs or alcohol after the injury * Drinking alcohol after a brain injury can cause severe illness that requires hospitalization * After a head injury, a person who drinks alcohol is at higher risk of seizures and other complications

ANS D * ✅After a head injury, a person who drinks alcohol is at higher risk of seizures and other complications Alcohol and drug use can be dangerous for anyone, but a client with a head injury should be counseled to avoid these substances for safety purposes. A person who has experienced a brain injury is more likely to suffer from seizures and other negative complications with even mild alcohol use. Alcohol and drugs often have much more powerful effects on a brain-injured person and the person is at greater risk of injury or having another head injury when using these substances * It takes much more alcohol to achieve the same effects in a person who has had a brain injury A client's tolerance of alcohol does not increase following a TBI. Rather, their brain usually becomes MORE sensitive to alcohol * Drinking alcohol after a brain injury can cause severe illness that requires hospitalization The client is at risk for seizures and accidents while drinking following a TBI, but not a severe illness. * Most people with a brain injury overdose the first time they use drugs or alcohol after the injury While overdosing on alcohol IS a risk for a client with a TBI, the statement is incorrect because the majority of clients do not overdose the first time they use drugs or alcohol after their injury.

A nurse is caring for a client post-op colon resection and notes a decreased blood pressure, rigid abdomen and the client reports severe pain. Based on these findings and knowing the possible complications, which of the following lab values would be the MOST appropriate to evaluate at this time? * Potassium * Lactic acid * Neutrophils * Hemoglobin

ANS D * ✅Hemoglobin One of the most common complications after bowel surgery is internal bleeding. Signs of internal bleeding would include hypotension, tachycardia, severe abdominal pain, and a rigid abdomen. Since these are the signs seen in this client, checking a hemoglobin level would be the most appropriate to compare to the pre-op levels. * Potassium Potassium levels be significantly affected by severe diarrhea or potassium loss in stool. However, this client is showing signs of internal bleeding. Therefore checking a hemoglobin level is the most appropriate. * Lactic acid Lactic acid might increase if the client was experiencing necrotic bowel or sepsis. Signs of this would include hypoactive or absent bowel sounds, fever, and tachycardia. These are not the signs this client is presenting with at this time. Therefore a hemoglobin is the most appropriate. * Neutrophils Neutrophils (white blood cells) would increase if the client was experiencing a post-op infection. Signs of this would include a fever, pain, and redness or drainage at the surgical site. Instead, the client is exhibiting signs of hemorrhage. Therefore a hemoglobin would be the most appropriate.

Your patient had an ileostomy placed 2 weeks ago. Which bowel pattern would be expected for this patient? * Output once per day * Output every 2-3 hours * Continent, controlled output * Continuous output

ANS D * ✅Continuous output An ileostomy is placed in the ileum of the small intestine. Therefore output will be liquid or semi-soft and will be continous. As the peristalsis occurs in the small intestine, stool output will be pushed out - therefore clients should expect continuous output throughout the day and night. * Output every 2-3 hours Because output from the ileostomy moves with peristalsis, it will be continuous, not intermittent. * Continent, controlled output Clients with an ileostomy do not have control over their output like they might with a colostomy or other types of bowel ostomies. It is a continuous output. * Output once per day Output from an ileostomy is liquid or semi-soft and flows continuously with peristalsis, NOT once daily.

A nurse is ambulating a client who is using a cane. Which is the proper side for the cane? A.The affected side only if a nurse is ambulating with them B. Affected side C. The unaffected side only if the nurse is ambulating with them D. Unaffected side

ANS D ✅Unaffected side The cane should be held on the unaffected side of the client. Affected side A nurse should stand on the affected side while the cane is held on the unaffected side of the client. The affected side only if a nurse is ambulating with them The cane should not be held on the affected side. The unaffected side only if the nurse is ambulating with them The cane should be held on the unaffected side whenever ambulating.

A nurse is performing a focused cardiac assessment on a client with heart disease. The nurse notes that the client's heart is beating in a regular rhythm and the cardiac conduction system appears to be working correctly upon auscultation. The heart's ability to keep a pace using its own electrical activity is known as which of the following? A. Automaticity B. Contractility C. Sensitivity D. Irritability

ANS- A ✅Automaticity Cardiac automaticity is the ability of the heart to control its own rate and pace. The heart's electrical conduction system sends messages to various points across the heart that stimulate it to contract and propel blood forward into circulation. The heart has its own system that is not consciously controlled by the client. Contractility Contractility refers to the force at which the heart muscle is able to contract. Irritability Irritability refers to how regularly the electrical nodes in the heart are conducting electrical impulses. Sensitivity This refers to the response of the cardiac system to cardioactive drugs.

A client with a gastric ileus is on bed rest with a nasogastric tube in place for nutrition. Which actions should the nurse perform that would best prevent aspiration of stomach contents in this client? Select all that apply. Assess placement of the feeding tube prior to each feeding * . Provide sedatives to keep the client calm * . Avoid giving bolus feedings * . Get a swallowing assessment before introducing oral feedings * Maintain the head of the bed at a 90-degree elevation

ANS. A,C,D * Maintain the head of the bed at a 90-degree elevation The head of the bed should be between 30-45 degrees to prevent aspiration. * * ✅Assess placement of the feeding tube prior to each feeding A nasogastric tube increases the risk of aspiration in the affected client. There is risk for food and fluids administered through the tube to be regurgitated, causing the client to potentially swallow or cough and aspirate some secretions into the lungs. The nurse can minimize this risk by elevating the head of the bed between 30-45 degrees, avoiding giving too much solution through the enteral feeding at once, and monitoring the placement of the tube. Once the client's status is appropriate to discontinue tube feedings and the provider has ordered to begin oral feedings, the client must pass a swallow assessment before beginning oral intake. * * * ✅Avoid giving bolus feedings A nasogastric tube increases the risk of aspiration in the affected client. There is risk for food and fluids administered through the tube to be regurgitated, causing the client to potentially swallow or cough and aspirate some secretions into the lungs. The nurse can minimize this risk by elevating the head of the bed between 30-45 degrees, avoiding giving too much solution through the enteral feeding at once, and monitoring the placement of the tube. Once the client's status is appropriate to discontinue tube feedings and the provider has ordered to begin oral feedings, the client must pass a swallow assessment before beginning oral intake. * ✅Get a swallowing assessment before introducing oral feedings A nasogastric tube increases the risk of aspiration in the affected client. There is risk for food and fluids administered through the tube to be regurgitated, causing the client to potentially swallow or cough and aspirate some secretions into the lungs. The nurse can minimize this risk by elevating the head of the bed between 30-45 degrees, avoiding giving too much solution through the enteral feeding at once, and monitoring the placement of the tube. Once the client's status is appropriate to discontinue tube feedings and the provider has ordered to begin oral feedings, the client must pass a swallow assessment before beginning oral intake. * .f10d46a1-5df6-40bf-8547-f79466edf340{fill:#231f20} Provide sedatives to keep the client calm Once a nasogastric tube is in place, it does not usually produce anxiety, so sedatives would not be appropriate in this situation.

A patient has been unable to eat solid food for 2 weeks and is in the clinic today complaining of weakness, tiredness, and hair loss. The patient states that her usual weight is 175 pounds, but today she weighs 161 pounds. What is her recent weight change percentage? _______

ANS: 8% To calculate recent weight change percentage, use this formula: Usual weight - Current weight/Usual weight × 100 175 - 161 = 14 pounds 14 ÷ 175 = 0.080.08 × 100 = 8%

TRUE or FALSE Determining what the patient normally eats can help diagnose the role of intake in the nutritional problem as primary, secondary, or insignificant.

ANS: True

TRUE or FALSE People who take five or more prescription drugs, over-the-counter drugs, or dietary supplements are at increased risk for developing drug-induced nutrient deficiencies.

ANS: True

TRUE or FALSE Nutritional screening uses a small number of factors to identify patients or clients with malnutrition or at risk of malnutrition.

ANS: True

* Question 10 of 10 A nurse is taking a client's blood pressure and measures 128/82 mmHg. What is the client's pulse pressure? A. 46 mm Hg B. 82 mm Hg C. 32 mm Hg D. 100 mm Hg

ANS: A

A client who sustained a fractured arm requires range-of-motion exercises to promote circulation. The nurse educates the client about performing active range of motion. Which of the following best describes how this motion is performed? A. The client performs the exercises without assistance B. The client wears an electronic device that performs range of motion C. The nurse and the client perform the exercises together D. The nurse performs the exercises for the client

ANS: A ✅The client performs the exercises without assistance Active range of motion (ROM) is a type of exercise in which the client performs the motions without assistance. The nurse may provide stand-by assistance and may teach the client about proper body mechanics, but the client does the work on their own. The nurse performs the exercises for the client Active ROM means that the client is doing the exercises, while passive ROM means the nurse performs the movement for the client. The nurse and the client perform the exercises together Active ROM does not require the nurse to help move the client's body. The client wears an electronic device that performs range of motion This does not describe active ROM.

The nurse is admitting a client who states, "When I was pregnant I would get heartburn so bad, I was eating antacids like crazy. I now feel the same way, except I am not pregnant. The antacids aren't helping me and now I have this pain between my shoulder blades." Which part of this statement is most concerning and must be investigated further? A."I have this pain between my shoulder blades" B."The antacids aren't helping me" C."When I was pregnant I would get heartburn so bad" D."I was eating antacids like crazy"

ANS: A "✅I have this pain between my shoulder blades" This female client is showing signs of a myocardial infarction. Women often present with atypical symptoms and will often describe a feeling of acid reflux that is actually a heart attack. This client needs an EKG (ECG) immediately to rule out a STEMI. "When I was pregnant I would get heartburn so bad" Heartburn is common during pregnancy. "I was eating antacids like crazy" This is a safe substance for pregnant women to consume, so this statement is not concerning. "The antacids aren't helping me" This is concerning, but the most concerning statement is the client describing pain between the shoulder blades.

A nurse is caring for a client who has experienced a head injury. The nurse is performing a focused neurological exam and documents that the client's mental status is obtunded. Which best describes a client's affect as obtunded? A.The client has very slow responses and decrease interest in the environment B.The client only arouses to painful stimuli C. The client is easily awakened with voice from normal sleep D. The client does not respond to any stimulation

ANS: A * ✅The client has very slow responses and decrease interest in the environment Following a head injury, a nurse may perform a neurological exam that assesses the client's mental status. Several terms are used to describe the client's level of consciousness. When a nurse describes a client's mental status as obtunded, it means that the client is difficult to arouse, has very slow responses, and excessive sleepiness. The client is easily awakened with voice from normal sleep This describes an alert level of consciousness. The client only arouses to painful stimuli This describes a stuporous level of consciousness. The client does not respond to any stimulation This describes a comatose level of consciousness

The nurse is conducting a class on alcohol and the effects of alcohol on the body. What information about the number of standard drinks (each containing 14 grams of alcohol) associated with a 32% increase in breast cancer should the nurse include in the class? a. ≥ 2.1/day b. ≥ 4.2/day c. ≥ 8/week d. ≥ 15/week

ANS: A Drinking ≥30 grams/day of alcohol (2.1 standard drinks) increases the rate of breast cancer by 32% compared to those with no alcohol intake. The mechanism in causing breast cancer is likely an increase in estrogen steroids, increasing the risk for hormone-sensitive tumors. Drinking ≥ 15 drinks per week for men and ≥ 8 drinks per week for women increases the risks for chronic diseases such as hypertension; heart disease; stroke; breast, oral, esophageal, liver, and colorectal cancers; and mental illness. The number of drinks associated with a 32% risk for developing breast cancer is ≥ 2.1 drinks/day. DIF: Cognitive Level: Remembering (Knowledge) MSC: Client Needs: Physiological Integrity: Reduction of Risk Potential

When considering a nutritional assessment, what should the nurse be aware is/are one of the most common anthropometric measurements? a. Height and weight b. Leg circumference c. Chest and waist measurements d. Skinfold thickness of the biceps

ANS: A The most commonly used anthropometric measures are height, weight, body mass index, waist-to-hip ratio, and arm span or total arm length. DIF: Cognitive Level: Remembering (Knowledge) MSC: Client Needs: Health Promotion and Maintenance

The student nurse understands an abnormality in which electrolyte will cause an abnormal T wave? A. Potassium B.Chloride C. Sodium D. Calcium

ANS: A ✅Potassium Abnormal potassium will cause an abnormal T wave. Sodium Sodium will not cause an abnormal T wave. Calcium Calcium will not cause an abnormal T wave. Chloride Chloride does not cause an abnormal T wave

Which of these techniques uses the sense of touch to assess texture, temperature, moisture, and swelling when the nurse is assessing a patient? a. Palpation b. Inspection c. Percussion d. Auscultation

ANS: A Palpation uses the sense of touch to assess the patient for the factors in the question (texture, temperature, moisture, and swelling). Inspection involves vision; percussion assesses through the use of palpable vibrations and audible sounds; and auscultation uses the sense of hearing. DIF: Cognitive Level: Remembering (Knowledge) MSC: Client Needs: Health Promotion and Maintenance

What do the patient's record, laboratory studies, objective data, and subjective data combine to form? a. Database b. Admitting data c. Financial statement d. Discharge summary

ANS: A Together with the patient's record and laboratory studies, the objective and subjective data form the database. The other items are not part of the patient's record, laboratory studies, or data.

Which of the following recommendations would be most effective for someone wanting to eat more fiber? a. Eat legumes more often. b. Eat raw vegetables in place of cooked vegetables. c. Use potatoes in place of white rice. d. Eat fruit for dessert in place of ice cream.

ANS: A Whole grains, bran cereals, legumes, and unpeeled fruits and vegetables are the best sources of fiber.

The nurse recognizes that the client is suffering from acute delirium. What is the first action the nurse should take? A.Contact the provider because this is considered a medical emergency B.Increase client checks to every two hours until it resolves C. Document the findings at the end of the shift D. Recognize that this represents an exacerbation of the client's underlying dementia

ANS: A ✅Contact the provider because this is considered a medical emergency Delirium is considered to be a medical emergency because it is a sign that there is an acute illness or disease process occurring that has lead to the condition. Older clients diagnosed with delirium are at an increased risk for mortality during the 12 months following hospital admission, and delirium is an independent risk factor of increased mortality. The provider should be notified as soon as possible. * Increase client checks to every two hours until it resolves Simply observing the client is not appropriate, because the client needs intervention and treatment to treat the underlying physiological process that is causing delirium. * Document the findings at the end of the shift The nurse must notify the provider so the client can be assessed medically and treated appropriately. * Recognize that this represents an exacerbation of the client's underlying dementia A client can develop delirium whether or not they have underlying dementia.

After measuring the PR interval, the student nurse notices the PR interval is 0.18 seconds. The student nurse would need to take which of the following actions? A. Continue to monitor the client B. Call the provider C. Obtain a 12 lead electrocardiogram D. Check the client's pulse

ANS: A ✅Continue to monitor the client 0.18 seconds is a normal PR interval. Call the provider 0.18 seconds is a normal PR interval so the client should be monitored and there is no need to do an electrocardiogram or check the pulse. Obtain a 12 lead electrocardiogram 0.18 seconds is a normal PR interval so the client should be monitored and there is no need to do an electrocardiogram or check the pulse. Check the client's pulse 0.18 seconds is a normal PR interval so the client should be monitored and there is no need to do an electrocardiogram or check the pulse.

The student nurse is reviewing the electrical activity of the heart. The student nurse is aware that the electrical impulse begins with which of the following? SA Node Bundle of His Purkinje Fibers AV Node

ANS: A ✅SA Node This is where the electrical conduction begins. AV Node The AV node receives the signal from the SA node where electrical conduction begins. Bundle of His The bundle of his receives the signal from the AV node. Purkinje Fibers The Purkinje fibers are the last to receive the electrical stimulus, which begins at the SA node.

The telemetry nurse is caring for a client who complains of chest pain. The ECG shows ST depression Which orders does the nurse anticipate? Select all that apply. A.Nitroglycerin SL B. BMP C. 12 lead EKG} 2 liters O2 by NC D. Cardiac enzymes

ANS: A, C, D, E ✅ Cardiac enzymes A series of cardiac enzymes, known as troponins, will be ordered for this client. ✅12 lead EKG . A 12-lead EKG can be anticipated to confirm. ✅Nitroglycerin SL Nitroglycerin dilates the coronary vessels, and would be given to relieve chest pain. ✅2 liters O2 by NC Oxygen is given to a client with chest pain and ST segment depression to reduce oxygen demands.

* Question 6 of 10 A nurse is caring for a 30-year-old client who states, "I am overwhelmed by stress in my life and I don't know how to change." Which suggestions could the nurse make to this client that would help with stress management? Select all that apply. A. The client should look for ways to reduce life stressors B. The client should seek medication therapy for the stress C. The client should seek cognitive-behavioral therapy D. The client should determine the source of the stress E. The client should prioritize stress management

ANS: A, D, E

A client with irritable bowel syndrome has asked the nurse if there is anything that can be done to decrease the frequency of diarrhea. Which of the following advice is appropriate for a client to develop regular bowel functioning? Select all that apply. * . Decrease activity levels and increase rest * Increase intake of fiber * . Drink 8 to 10 glasses of liquids per day * Eat the largest meal of the day in the evening * Utilize narcotics to reduce pain

ANS: A,B * ✅Increase intake of fiber The nurse who works with a client who has irritable bowel syndrome may counsel the client to increase dietary fiber, drink plenty of fluids, limit greasy and fatty foods, and increase activity levels in order to best regulate bowel function. If dietary and lifestyle changes do not assist the client in managing IBS, medications may be used. * ✅Drink 8 to 10 glasses of liquids per day Chronic diarrhea can lead to dehydration and electrolyte imbalances for the affected client. The client should stay hydrated to avoid further irritation of the bowels. * * Utilize narcotics to reduce pain The client with irritable bowel syndrome should be instructed to avoid narcotics. * * Eat the largest meal of the day in the evening A steady diet of small meals seems to work better for clients with IBS than large meals. Decrease activity levels and increase rest Having a regular exercise regimen can help clients manage symptoms of irritable bowel syndrome

The nurse is caring for a client who states, "I don't have a family history of hypertension so I don't have to worry about that." Which of the following risk factors for hypertension should the nurse teach the client? Select all that apply. A. Stress B. Low fiber diet C. Physical inactivity D. Age E. High sodium diet

ANS: A,C,D, E A: stress, causative factor of primary HTN C: Physical inactivity, sedentary lifestyle causative factor of primary HTN D: AGE, certain ethnicities have a higher incidence of HTN (African Americans). The older a perso is, the higher the likelihood E:High sodium diet, too much sodium causative factor of primary HTN

A nurse is completing a neurological assessment in a client with a back injury. Which of the following steps would be part of a test of motor function in this client? Select all that apply. A. Determine if the client opens his or her eyes spontaneously B. Have the client flex and extend the feet C. Assess whether the client knows the date and time D. Ask the client to squeeze the nurse's fingers E. Tell the client to push his or her arms away from the nurse

ANS: A,c, D * ✅Ask the client to squeeze the nurse's fingers The client's test of motor function during the exam assesses abilities to perform certain actions and indicates a specific level of brain function. Assessing the client's ability to squeeze the nurse's fingers is a common method for determining motor function. ✅Have the client flex and extend the feet Flexing and extending the feet is a test of motor function. ✅Tell the client to push his or her arms away from the nurse Asking the client to push his or her hands against resistance is a test of motor function. Assess whether the client knows the date and time This is part of a neurological assessment instead of motor function assessment. A client who knows the date and time has some amount of orientation. Determine if the client opens his or her eyes spontaneously This is part of a neurological assessment. Spontaneous eye opening indicates brainstem activity.

The client has been diagnosed with Guillain-Barre syndrome. Which of the following are appropriate nursing interventions? Select all that apply. Assisting with plasmapheresis * Encouraging ambulation * Giving steroids in the acute phase * Assisting in range of motion exercises * Place the client in the Trendelenburg position in bed

ANS: A,c,d * ✅Assisting in range of motion exercises Bedrest should be encouraged with the head of the bed elevated. The nurse should assist in range of motion exercises and should observe for respiratory failure. * * ✅ Giving steroids in the acute phase In Guillain-Barre syndrome, the client should have steroids or plasmapheresis in the acute phase. * * ✅Assisting with plasmapheresis Plasmapheresis is an appropriate treatment in the acute phase of Guillain-Barre syndrome. * * Encouraging ambulation Bedrest, not ambulation, should be encouraged for the client with Guillain-Barre syndrome. Place the client in the Trendelenburg position in bed The optimal position for a client with Guillain-Barre syndrome is with the head of the bed elevated.

The nurse is caring for a client with hypertension and is administering an inch of nitropaste. Which of the following is an important action for the nurse to take? A. Wear a mask while administering the nitropaste B. Wear gloves while administering the nitropaste C. Use cloth tape to place nitropaste on the chest D. Wash hands before and after touching the paste

ANS: B MUST WEAR GLOVES-This medication is DESIGNED to absorb though the skin and WILL absorb through yours. Apply one inch dosing paper, cover with plastic wrap and secure with tape. is important secondary.

The nurse is caring for a client with an acute kidney injury. Which of the following dietary recommendations is inappropriate? A. Increase fresh vegetables and fruit B. Increase foods high in potassium C. Limit intake of whole grains D. Limit processed or canned foods

ANS: B * ✅Increase foods high in potassium This is an inappropriate dietary recommendation for a client with acute kidney injury, because the kidneys are not able to excrete potassium as well as normal. If the client increases these foods, he or she could get hyperkalemia. * Increase fresh vegetables and fruit This is an appropriate dietary recommendation for a client with an acute kidney injury. * Limit intake of whole grains This is an appropriate recommendation for the client with a kidney injury, because whole grains contain phosphorus, which should be excreted by the kidneys. Because the kidneys are injured, this electrolyte will build up in the body, causing an imbalance. * Limit processed or canned foods This is an appropriate recommendation, because these foods have a high sodium content. The injured kidneys will have difficulty excreting sodium.

A nurse finished getting report on four clients and now is deciding who is the priority. The client with which of the following should be seen first? . A.Failure to thrive and in need of assistance ordering dinner B.STEMI with heparin infusing and requested to go to the bathroom C. A right below knee amputation and a history of diabetes D.Pneumonia with IV antibiotics infusing and the pump is currently alarming

ANS: B * ✅STEMI with heparin infusing and requested to go to the bathroom Clients receiving heparin are at an increased risk for bleeding, therefore a fall can be extremely detrimental. This client needs to be seen first to assess their gait, heparin drip, IV patency, and level of assistance to the bathroom. * Failure to thrive and in need of assistance ordering dinner This client can be assisted by a nurse assistant. * A right below knee amputation and a history of diabetes BKA diabetic can perhaps have eyes laid on them but are not in need of anything at this time. It is more of a priority to see the client that had a STEMI and has heparin infusion. * Pneumonia with IV antibiotics infusing and the pump is currently alarming This client can wait. It is more of a priority to see the client that had a STEMI and has a heparin infusion.

The nurse is working in the intensive care unit and is taking report on a client who is septic in the emergency department. The ED nurse reports the client is immobile with a stage II pressure ulcer on the coccyx. Which is the priority question for the nurse to ask for further clarification? A. Is the skin around the pressure ulcer blanchable? B. When was the last time he was turned? C. What is the measurement of the pressure ulcer? D. Is he incontinent?

ANS: B * ✅When was the last time he was turned? This client is at risk for skin breakdown. A client who is immobile must be kept on a two hour turning schedule, especially if a pressure ulcer is present. Is he incontinent? Incontinence is an element of the nursing assessment, and while the nurse can ask prior to assessing the client, it is not a priority question that is dependent on the client's previous nurse. Is the skin around the pressure ulcer blanchable? The nurse will be able to assess this once the client is in the unit. It is not a priority question. What is the measurement of the pressure ulcer? The wound will be measured by the nurse once the client is on the unit.

A 19-year-old college student is seen in the university health clinic for symptoms of dehydration and weight loss. The client is a member of the school's track team and spends several hours a day training. Which information should the nurse give to this client to prevent dehydration associated with excessive exercise? Establish a baseline body weight by weighing every week to determine fluid needs Drink fluids starting several hours before exercise begins Remember that fluid replacement after exercise cannot fully replace that which was lost with activity Avoid electrolyte replacement beverages when exercising and drink only water

ANS: B ✅Drink fluids starting several hours before exercise begins A person who exercises a great amount, such as with a student athlete, may be at risk of dehydration and electrolyte imbalance from water loss through sweat and respiration. The nurse should counsel the client to drink electrolyte replacement beverages or water and to drink fluids throughout the entire period before, during, and after exercising. Establish a baseline body weight by weighing every week to determine fluid needs This is an extremely complicated regimen. Simply drinking water starting earlier and continuing to drink after exercise is over is a better piece of advice from the nurse. Avoid electrolyte replacement beverages when exercising and drink only water Electrolyte replacement is extremely important, as the client is losing electrolytes through sweat and replacing them will help prevent an electrolyte imbalance in the body. Remember that fluid replacement after exercise cannot fully replace that which was lost with activity While this is true, it is not a positive change that the client can implement to prevent dehydration.

The nurse is reviewing concepts related to one's heritage and beliefs. Which concept refers to belief in a divine or superhuman power(s) to be obeyed and worshipped as the creator(s) and ruler(s) of the universe? a. Culture b. Religion c. Ethnicity d. Spirituality

ANS: B A belief in a divine or superhuman power(s) to be obeyed and worshipped as the creator(s) and ruler(s) of the universe refers to religion. Religion is an organized system of beliefs concerning the cause, nature, and purpose of the universe, especially belief in or the worship of God or gods. Culture is a complex phenomenon that includes attitude, beliefs, self-definitions, norms, roles, and values learned from birth through the processes of language acquisition and socialization. It does not refer to a belief in a divine or superhuman power. Ethnicity pertains to a social group within the social system that claims to possess variable traits, such as a common geographic origin, religion, race, and others. Spirituality is a broad term focused on a connection with something bigger than oneself and a belief in transcendence. DIF: Cognitive Level: Remembering (Knowledge) MSC: Client Needs: Psychosocial Integrity

When reviewing the demographics of ethnic groups in the United States, the nurse recalls that which is the largest and fastest growing population? a. Asian b. Hispanic c. American Indian d. African American/black

ANS: B Hispanics are the largest and fastest growing population in the United States, followed by African Americans/blacks, Asians, two or more races, American Indians and Alaska natives, and other groups. DIF: Cognitive Level: Remembering (Knowledge) MSC: Client Needs: General Chapter 2 - Cultural Assessment

During a session on substance abuse, the nurse is reviewing statistics with the class. For people aged 12 years and older, which illicit substance was most commonly used? a. Heroin b. Marijuana c. Crack cocaine d. Hallucinogens

ANS: B In people age 12 years and older who reported using during the past month, marijuana (hashish) was the most commonly used illicit drug reported. DIF: Cognitive Level: Remembering (Knowledge) MSC: Client Needs: Psychosocial Integrity

During a seminar on cultural aspects of nursing, the nurse recognizes that the definition stating "the specific and distinct knowledge, beliefs, customs, and skills acquired by members of a society" reflects which term? a. Norms b. Culture c. Ethnicity d. Assimilation

ANS: B The culture that develops in any given society is unique, encompassing all of the knowledge, beliefs, customs, and skills acquired by members of the society. The other terms do not fit the given definition. Norms refers to the typical or usual. Ethnicity refers to a social group that may possess shared traits, such as common geographic origin, migratory status, religion, language, values, traditions, or symbols and food preferences. Assimilation refers to taking on the characteristics of the dominant culture. DIF: Cognitive Level: Remembering (Knowledge) MSC: Client Needs: Psychosocial Integrity

When performing a physical assessment, what technique should the nurse always perform first? a. Palpation b. Inspection c. Percussion d. Auscultation

ANS: B The skills requisite for the physical examination are inspection, palpation, percussion, and auscultation. The skills are performed one at a time and in this order (with the exception of the abdominal assessment, during which auscultation takes place before palpation and percussion). The assessment of each body system always begins with inspection. A focused inspection takes time and yields a surprising amount of information. DIF: Cognitive Level: Remembering (Knowledge) MSC: Client Needs: Health Promotion and Maintenance

The nurse is preparing to examine a 4-year-old child. Which action by the nurse is appropriate for this age group? a. Explain the procedures in detail to alleviate the child's anxiety. b. Give the child feedback and reassurance during the examination. c. Do not ask the child to remove his or her clothes because children at this age are usually very private. d. Perform an examination

ANS: B With preschool children, short, simple explanations should be used. Children at this age are usually willing to undress. An examination of the head should be performed last. During the examination, needed feedback and reassurance should be given to the preschooler. This is a preschool-aged child so the nurse should not explain procedures in detail as that will likely make the child anxious. Children at this age are usually willing to undress and should do so as needed for a thorough examination. An examination of the head should be performed last, not first. During the examination of a preschool-aged child, needed feedback and reassurance should be given to the child and short, simple explanations should be used.

The student nurse is assessing the waveforms on an electrocardiogram (EKG) and suddenly notices a flat line. Which action should the student nurse take first? A.Obtain a 12 lead EKG B.Check the client's pulse C.Call the provider D:Call a code and begin compressions

ANS: B ✅Check the client's pulse In case the lead fell off or false artifact, always assess the client first. Call a code and begin compressions The client should be assessed first in case the leads fell off. Call the provider The client should be assessed first in case the leads fell off. Obtain a 12 lead EKG The client should be assessed first in case the leads fell off.

The student nurse is observing the waveforms on an electrocardiogram (EKG) and notices an abnormal QRS complex on the EKG. The student nurse understands this is a problem with which of the following? A. Atrial depolarization B.Ventricular depolarization C.Ventricular repolarization D.Atrial repolarization

ANS: B ✅Ventricular depolarization Ventricular depolarization produces the QRS complex. Atrial depolarization P wave is produced from atrial depolarization. Atrial repolarization The P wave is produced from atrial depolarization, atrial repolarization is not seen on an EKG. Ventricular repolarization Ventricular repolarization produces the T wave.

The nurse is caring for a client whose EKG (ECG) results have just been obtained. The nurse recognizes which of the following rhythms as an emergency? A. Atrial fibrillation B. Ventricular fibrillation C. Sinus tachycardia D. Sinus bradycardia

ANS: B ✅Ventricular fibrillation A client with ventricular fibrillation is a medical emergency. This client has ZERO cardiac output, because the heart is unable to pump blood. The nurse should stay with the client, call for help, delegate someone to get a crash cart, call the healthcare provider, and prepare for a potential code blue. Atrial fibrillation This rhythm may or may not be emergent, but is less life-threatening than ventricular fibrillation. The atria are unable to pump blood very effectively, resulting in decreased cardiac output. The client may or may not be symptomatic. Sinus tachycardia Sinus tachycardia is a heart rate above 100 bpm. The client may be stable or unstable with symptoms such as palpitations and lightheadedness. This is not a medical emergency, but should be managed by treating the underlying cause (fever, anxiety), teaching the client relaxation techniques, or administering beta blockers. Sinus bradycardia Sinus bradycardia is a heart rate below 60 bpm. This client may be asymptomatic, or may have symptoms such as lightheadedness or vertigo, depending on the cause. This rhythm is much less concerning than ventricular fibrillation.

The nurse is assessing a client who will have an atrial pacemaker inserted due to a non-functioning SA node. Prior to the procedure, the nurse expects which of the following pulse rates upon assessment? A. 40-60 beats per minute B. 60-100 beats per minute C. 120-150 beats per minute D. 20-40 beats per minute

ANS: B ✅40-60 beats per minute If the SA node is non-functioning, the nurse would expect the AV node to take over as the pacemaker of the heart. The pulse rate falls between 40-60 beats per minute when the AV node is pacing. 60-100 beats per minute 60-100 beats per minute is the rate at which the SA node paces the heart. If the SA node is not working, the AV node would take over at a rate of 40-60 beats per minute. 20-40 beats per minute This is the rate seen when the right and left bundle branches take over as pacemaker. If the SA node is not working, the next to take control of the heart rate would be the AV node, not the bundle branches. 120-150 beats per minute A client with atrial fibrillation (a-fib) would see a heart rate in this range. This does not occur due to a non-functioning SA node, but rather heart defects, coronary artery disease, stimulants, and other comorbidities.

A student nurse understands that when the heart atria are depolarizing, which action is taking place? A. The ventricles are resting B. The ventricles are filling with blood C. The atria re-filling with blood D. The atria are resting

ANS: B ✅The ventricles are filling with blood During atrial depolarization, the atria are contracting and filling the ventricles with blood. The ventricles are resting The ventricles are repolarizing. The atria are resting That is atrial repolarization. The atria re-filling with blood The ventricles are filling with blood during atrial depolarization.

The student nurse is precepting on the telemetry floor. The preceptor asks about the pathway of electrical conduction in the heart. Which of the following is a correct response by the student? A. AV node- SA node - Right and left Bundle Branches - Bundle of His - Purkinje fibers B. SA node - AV node - Bundle of His - Right and Left Bundle Branches - Purkinje fibers C. Bundle of His - AV node - SA node - Purkinje fibers - Right and Left Bundle Branches D. AV node - Purkinje fibers - SA node - Bundle of His - Right and Left Bundle Branches

ANS: B ✅SA node - AV node - Bundle of His - Right and Left Bundle Branches - Purkinje fibers This is the correct path of electrical conduction in the heart. AV node - Purkinje fibers - SA node - Bundle of His - Right and Left Bundle Branches This is incorrect. The correct path of electrical conduction is SA node - AV node - Bundle of His - Right and Left Bundle Branches - Purkinje fibers. AV node- SA node - Right and left Bundle Branches - Bundle of His - Purkinje fibers This is incorrect. The correct path of electrical conduction is SA node - AV node - Bundle of His - Right and Left Bundle Branches - Purkinje fibers. Bundle of His - AV node - SA node - Purkinje fibers - Right and Left Bundle Branches This is incorrect. The correct path of electrical conduction is SA node - AV node - Bundle of His - Right and Left Bundle Branches - Purkinje fibers.

A client is preparing to undergo a craniotomy for treatment of a brain bleed. The nurse performs the pre-operative assessment and includes the Glasgow Coma Scale (GCS). The client is intubated, sedated, and has an endotracheal tube in place for airway management. Which intervention should the nurse utilize to perform the pre-operative assessment? A. Assess the client's eye opening but do not assess verbal response B. Do not perform the GCS or any neuro function test, as they will not be accurate C. Perform the GCS as ordered and document the results D. Perform as much of the neuro exam as possible but avoid the GCS, since the client is intubated

ANS: C Perform as much of the neuro exam as possible but avoid the GCS, since the client is intubated The GCS can still be performed, with the modifications listed. * ✅Perform the GCS as ordered and document the results The GCS is used to assess neurological function and is most commonly used in situations where a client has suffered a head injury. The GCS assesses certain functions including motor response, eye opening, and verbal response. The nurse must indicate that the client is intubated and sedated on the GCS form, because this will affect the score. If the client is intubated, the nurse writes the letter "T" or "ETT" on the GCS form, which shows that the client might be able to verbalize if not intubated. If the client was sedated during the exam, the nurse writes the letter "S", which indicates that sedation may have decreased the GCS score. Assess the client's eye opening but do not assess verbal response The GCS can still be performed, with the modifications listed. Do not perform the GCS or any neuro function test, as they will not be accurate The GCS can still be performed, with the modifications listed.

When administering subcutaneous heparin, the nurse will remember to perform which action? a. Use the same sites for injection to reduce trauma. b. Use a 1-inch needle for subcutaneous injections. c. Inject the medication without aspirating for blood return. d. Massage the site after the injection to increase absorption.

ANS: C The anticoagulant effects of heparin can be reversed with protamine sulfate. In general, 1 mg of protamine sulfate can reverse the effects of 100 units of heparin.

The nurse is assessing a 30-year-old unemployed immigrant from an underdeveloped country who has been in the United States for 1 month. Which of these problems r/t his nutritional status might the nurse expect to find? a. Obesity b. Hypotension c. Osteomalacia d. Coronary artery disease

ANS: C General undernutrition- 70% to 80% of ideal weight suggests moderate malnutrition, hypertension, diarrhea, lactose intolerance, osteomalacia (soft bones), scurvy, and dental caries are among the more common nutrition-related problems of new immigrants from developing countries. Obesity, hypotension, and coronary artery disease are not nutritionally related problems commonly found in those newly immigrated to the United States from developing countries. Instead, general undernutrition, hypertension, diarrhea, lactose intolerance, osteomalacia, scurvy, and dental caries are among the more common nutrition-related problems of new immigrants from developing countries. DIF: Cognitive Level: Remembering (Knowledge) MSC: Client Needs: Health Promotion and Maintenance

A 2-year-old child has been brought to the clinic for a well-child checkup. What is the best way for the nurse to begin the assessment? a. Ask the parent to place the child on the examining table. b. Have the parent remove all of the child's clothing before the examination. c. Allow the child to keep a security object such as a toy or blanket during the examination. d. Initially focus the interactions on the child, essentially ignoring the parent until the child's trust has been obtained.

ANS: C The best place to examine the toddler is on the parent's lap. Toddlers understand symbols; therefore, a security object is helpful. Initially, the focus is more on the parent, which allows the child to adjust gradually and to become familiar with you. A 2-year-old child does not like to take off his or her clothes. Therefore, ask the parent to undress one body part at a time.

The nurse recognizes which of these people is at greatest risk for undernutrition? a. 30-year-old man b. 50-year-old woman c. 5-month-old infant d. 20-year-old college student

ANS: C Vulnerable groups for undernutrition are infants, children, pregnant women, recent immigrants, people with low incomes, hospitalized people, and aging adults. DIF: Cognitive Level: Remembering (Knowledge) MSC: Client Needs: Health Promotion and Maintenance

The nurse is taking a family history. Which specific disease or problem should be included in the assessment? a. Emphysema b. Head trauma c. Mental illness d. Fractured bones

ANS: C When reviewing the family history, the nurse should ask specifically about any history of heart disease, high blood pressure, stroke, diabetes, obesity, blood disorders, breast and ovarian cancers, colon cancer, sickle cell anemia, arthritis, allergies, alcohol or drug addiction, mental illness, suicide, seizure disorder, kidney disease, and tuberculosis (TB). Although a family history of emphysema and fractured bones may be good to know, they are not one of the specific diseases that should be specifically asked. A family history of head trauma is not a familial or hereditary characteristic and is not necessary to include in the assessment. DIF: Cognitive Level: Remembering (Knowledge) MSC: Client Needs: Health Promotion and Maintenance

A client is brought to the trauma bay after a motor vehicle collision with rollover. The client is unconscious but breathing. On the neurological assessment, the client does not respond to his/her name. The nurse performs a sternal rub and the client's eyes open, arms flex in no particular direction, and the client mumbles incoherently. The nurse anticipates the next action to be which of the following? A. Move on to your secondary survey B.Administer normal saline through 2 large bore IV's C. Prepare for rapid sequence intubation D.Facilitate family presence

ANS: C Facilitate family presence While that may be done for this client, this point in the initial survey is too early for this step. This client's diminished GCS is the priority to address next. * ✅Prepare for rapid sequence intubation The client is exhibiting a diminished Glasgow coma score (Eye-opening to painful stimulation = 2, Withdrawal to painful stimulation = 4, incomprehensible sounds = 2, Total score = 8) A GCS of 8 is usually an indicator of an acute decrease in level of consciousness and intubation is recommended to protect the client's airway. GCS 8=intubate. * Administer normal saline through 2 large bore IV's While this may be an action to perform during the initial survey, the nurse is assessing neurological status and fluids would not be a priority intervention for such. This client's diminished GCS is the priority to address next. * Move on to your secondary survey The findings on your initial survey require intervention before moving on. This client's diminished GCS is the priority to address next.

The nurse is assessing a patient who is obese for signs of metabolic syndrome. This condition is diagnosed when three or more certain risk factors are present. Which of these assessment findings are risk factors for metabolic syndrome? (Select all that apply.) a. Triglyceride level of 120 mg/dL b. Blood pressure reading of 110/80 mm Hg c. Blood pressure reading of 140/90 mm Hg d. Fasting plasma glucose level less than 100 mg/dL e. Fasting plasma glucose level greater than or equal to 110

ANS: C, E Metabolic syndrome is diagnosed when three or more of the following risk factors are present: (1) fasting plasma glucose level greater than or equal to 100 mg/dL; (2) blood pressure greater than or equal to 130/85 mm Hg; (3) waist circumference greater than or equal to 40 inches for men and 35 inches for women; (4) high-density lipoprotein cholesterol less than 40 in men and less than 50 in women; and (5) triglyceride levels greater than or equal to 150 mg/dL. DIF: Cognitive Level: Applying (Application)

A client presents to the emergency department with suspected pacemaker malfunction. The nurse obtains vital signs and gets an EKG. The vital signs are as follows: Blood pressure 78/45, pulse 32 beats per minute, oxygen 75% on room air, respiratory rate 32. The EKG does not show any pacer spikes. Which of the following situations is most likely occurring? A. The pacemaker has malfunctioned and the AV node has taken over as the pacemaker of the heart B. The pacemaker is functioning but the battery has run out of power C. The pacemaker is functioning but set to a rate that is too low D. The pacemaker has malfunctioned and the bundle branches have taken over as the pacemaker of the heart

ANS: D D ✅The pacemaker has malfunctioned and the bundle branches have taken over as the pacemaker of the heart This client is dependent on their pacemaker. The pacemaker has malfunctioned because the EKG does not show any pacemaker spikes, and the pulse rate is extremely low. This is an emergency. The bundle branches have taken over as the pacemaker, because they fire at a rate of 20-40 beats per minute. The pacemaker has malfunctioned and the AV node has taken over as the pacemaker of the heart The pacemaker in this scenario has malfunctioned, because the pulse rate is too low and the EKG shows no pacer spikes. However, if the AV node took over, the heart rate would be between 40-60 beats per minute. A pulse rate of 32 beats per minute indicates that the bundle branches have takes over as the heart's pacemaker. The pacemaker is functioning but the battery has run out of power The pacemaker is not functioning in this scenario. The pacemaker is functioning but set to a rate that is too low The pacemaker is not functioning at all, because the EKG does not show any pacer spikes.

The nurse is assigned to an older adult client in the emergency department whose family members report she is "acting bizarre". The nurse notes a decreasing level of consciousness and confusion. The nurse obtains vital signs which are within normal limits. What is the nurse's next priority? * Obtain a urine sample * Obtain an EKG * Ask the client if she knows where she is * Obtain a POC glucose

ANS: D * ✅Obtain a POC glucose Any client with altered mental status should be checked for hypoglycemia, because hypoglycemia can quickly deteriorate into an emergent situation as the glucose level drops and the client moves toward unresponsiveness. * Obtain a urine sample A urine sample is necessary if an older adult becomes confused because this can indicate a urinary tract infection. For this client, the decreasing level of consciousness is more consistent with hypoglycemia. * Ask the client if she knows where she is With the decreasing level of consciousness, the client may have trouble answering questions. The nurse should do a quick glucose test first. * Obtain an EKG There may be an order for an EKG for this client, but the symptoms described do not relate to a cardiac issue, so this is not the priority intervention.

* Question 2 of 10 A client is undergoing cardiac stress testing at a rehabilitation center. During the test, the nurse notes that the client becomes short of breath with a blood pressure drop from 130/80 mmHg to 98/58 mmHg. Which action of the nurse is most appropriate? A. Continue to monitor the client's blood pressure and continue the stress testing B Administer a cardiac glycoside medication such as Digoxin C. Have the client take a break for five minutes before resuming the test D. Stop the stress test immediately

ANS: D Stop the stress test immediately

A 66-year-old client has been diagnosed with heart disease and the provider orders an ECG. The nurse is preparing the client for the test by explaining what will happen. Which of the following statements by the nurse is correct? A. The ECG can be painful but I will give you some medication to help with the pain B. You will lie on a table while a technician uses an ultrasound to look at your heart C. The test should take about an hour, but you can sleep through it D. You will have leads connected to your chest that record your heart's activity

ANS: D ✅You will have leads connected to your chest that record your heart's activity An electrocardiogram (ECG) measures the electrical conduction of the heart. It involves placing leads on the client's chest that measure the heart's activity. An ECG usually only takes a few minutes and it is not painful for the client, but the client should be instructed to lie still during the procedure. The test should take about an hour, but you can sleep through it An ECG is relatively quick, lasting just a few minutes. This is useful when caring for a client with a suspected cardiac status change, as the ECG gives a good picture of the electrical activity of the heart. The ECG can be painful but I will give you some medication to help with the pain An ECG is not painful. You will lie on a table while a technician uses an ultrasound to look at your heart This is referring to an echocardiogram, which gives a three-dimensional view of the heart instead of recording the electrical activity of the heart.

The telemetry nurse observes an abnormally long PR interval. The nurse recognizes there is a delay at which of the following components of the cardiac conduction system? A.Purkinje fibers B.Bundle of His C.SA node D.,AV node

ANS: D ✅AV node The AV node is the area where a slight pause occurs before the electrical impulse continues from the atria to the ventricles. If the PR interval is longer than normal, this means there is a delay at the level of the atrioventricular node SA node The SA node is located in the atria, and is responsible for the initiation of an impulse that will then travel down and throughout the heart to create the contraction of the heart muscle. A lengthened PR interval means there is a delay in the AV node, not the SA node. Bundle of His The PR interval represents the time for the electrical current to reach the ventricles at the level of the AV node, and the impulse then travels to the Bundle of His. Purkinje fibers The Purkinje fibers are the last portion of the ventricles to receive the electrical impulse. The PR interval does not represent this component of the cardiac conduction system.

What is a focus of the health professional in the health promotion model? a. Changing people's perceptions of disease b. Identifying biomedical model interventions c. Identifying negative health acts of people d. Teaching and helping people choose a healthier lifestyle

ANS: D In the health promotion model, the focus of the health professional is on teaching and helping the consumer choose a healthier lifestyle. Changing people's perception of disease, identifying biomedical model interventions, and identifying negative health acts of people are not the focus of the health promotion model. In the health promotion model, the focus of the health professional is on teaching and helping the consumer choose a healthier lifestyle. DIF: Cognitive Level: Remembering (Knowledge) MSC: Client Needs: Health Promotion and Maintenance

When assessing a patient's nutritional status, what does the nurse need to recognize that sufficient nutrients need to do for optimal nutrition? a. Provide for the minimum body needs. b. Provide an excess of daily body requirements. c. Provide for daily body requirements but do not support increased metabolic demands. d. Provide for daily body requirements and support increased metabolic demands.

ANS: D Optimal nutritional status is achieved when sufficient nutrients are consumed to support day-to-day body needs and any increased metabolic demands resulting from growth, pregnancy, or illness. DIF: Cognitive Level: Remembering (Knowledge) MSC: Client Needs: Health Promotion and Maintenance

The nurse is preparing to do a functional assessment. Which statement best describes the purpose of a functional assessment? a. The functional assessment assesses how the individual is coping with life at home. b. It determines how children are meeting developmental milestones. c. The functional assessment can identify any problems with memory the individual may be experiencing. d. It helps determine how a person is managing day-to-day activities.

ANS: D The functional assessment measures how a person manages day-to-day activities. The other answers do not reflect the purpose of a functional assessment. The functional assessment measures how a person manages day-to-day activities. Options A, B, and C do not reflect the purpose of a functional assessment. DIF: Cognitive Level: Remembering (Knowledge) MSC: Client Needs: Health Promotion and Maintenance

What data should the nurse collect during the interview portion of a health assessment. a. Physical b. Historical c. Objective d. Subjective

ANS: D The interview is the first, and really the most important, part of data collection. During the interview, the nurse collects subjective data; that is, what the person says about him or herself. Physical data is gathered in the physical examination portion of a health assessment. Historical data is gathered from the medical records. Objective data is what the nurse observes, this typically occurs during the physical examination, not the interview. During the interview, the nurse collects subjective data; that is, what the person says about him or herself. DIF: Cognitive Level: Remembering (Knowledge) MSC: Client Needs: Psychosocial Integrity

Which term refers to a wound produced by the tearing or splitting of body tissue, usually from blunt impact over a bony surface? a. Hematoma b. Abrasion c. Contusion d. Laceration

ANS: D The term laceration refers to a wound produced by the tearing or splitting of body tissue. An abrasion is caused by the rubbing of the skin or mucous membrane. A contusion is injury to tissues without breakage of skin, and a hematoma is a localized collection of extravasated blood. The description in the question describes a laceration. DIF: Cognitive Level: Remembering (Knowledge) MSC: Client Needs: Safe and Effective Care Environment: Safety and Infection Control

A client has a family history of myocardial infarction and has been experiencing multiple episodes of chest pain. The client is prescribed a medication to prevent heart attacks. Which of the following medications is appropriate for this condition? A.Loperamide B.Indomethacin C.Ibuprofen D.Propranolol

ANS: D ✅Propranolol This is a beta blocker that is used to treat hypertension, angina, arrhythmias, and heart attacks. Indomethacin This is an antirheumatic medication used most often for clients with rheumatoid arthritis. Loperamide This medication is an antidiarrheal medication. Ibuprofen This is a non-steroidal anti-inflammatory agent that treats mild to moderate pain and inflammation. ✅Propranolol This is a beta blocker that is used to treat hypertension, angina, arrhythmias, and heart attacks.✅Propranolol This is a beta blocker that is used to treat hypertension, angina, arrhythmias, and heart attacks.

Question 8 of 10 The provider orders an antihypertensive medication for a client whose blood pressure is 205/110 mm Hg. The nurse reviews the medication administration record and sees an order for which of the following blood pressure medications? A. Neomycin B. Nalbuphine C. Nystatin D. Nitroprusside

ANS: D Generic Name nitroprusside Trade Name Nitropress Indication hypertensive crisis, cardiogenic shock Action peripheral vasodilation of arteries and veins decreasing preload and afterload Therapeutic Class antihypertensive Pharmacologic Class vasodilator Nursing Considerations • monitor HR, BP, and EKG continuously during therapy • may cause cyanide toxicity • sympathomimetics may decrease effectiveness • PAOP monitoring may help with MI

A nurse in the emergency room has clients that are all complaining of pain. The nurse knows that the client having pain in which of the following areas is the priority? Lower back Bilateral flank Lower abdomen Jaw and neck

ANS: D ✅Jaw and neck These symptoms can occur in a client having a myocardial infarction, especially in a female client. This client should be seen first to rule out an MI. Lower back This client is not a priority over symptoms of an MI, which are radiating jaw and neck pain. Bilateral flank This is a sign for kidney infection, which needs assessed and treated, but the client with a suspected MI needs to be seen first. Lower abdomen This is a sign for kidney infection, which needs assessed and treated, but the client with a suspected MI needs to be seen first.

A nurse is helping a client to develop a sleep hygiene program to improve sleep habits. Which of the following would the nurse suggest for this client as part of sleep hygiene? Select all that apply. A. The client should try to wake up at an earlier time than usual B. The client may sleep better if another person is in the room C. The client should go to bed later than usual D. The client should develop a familiar nighttime routine E..The client should try to find a comfortable position for sleeping

ANS: D,E * ✅The client should try to find a comfortable position for sleeping Sleep hygiene is the process of developing practices that will best promote sleep for the client. The nurse can counsel the client about better sleep hygiene by suggesting that they keep the same schedule for sleeping and waking up, avoiding caffeine and alcohol near bedtime, and finding a comfortable position for sleep. * ✅The client should develop a familiar nighttime routine The nurse can counsel the client about better sleep hygiene by suggesting that they keep a familiar evening routine. * The client should go to bed later than usual The best practice for sleep and wake time is to keep it the same as much as possible. The client may sleep better if another person is in the room In general, a person sleeps better when alone. The client should try to wake up at an earlier time than usual If a person must deviate from their routine, it is preferable to wake up at the same time every day, even if the person went to bed late.

A nurse is helping an immobile client by performing passive range of motion exercises. Which of the following principles should the nurse keep in mind when performing these exercises? A. If the client complains of pain, discontinue the exercise on that joint B.Position the client prone with the head to one side to begin the exercises C. Begin with the minimum range of motion the joint can tolerate and increase the amount each week D. Start with the lower extremities and move toward the head E.Increased oxidative capacity

ANS: a ✅If the client complains of pain, discontinue the exercise on that joint There may be times when a nurse must perform passive range of motion exercises for a client. This involves the nurse doing the work of the range of motion for the client without any help from the client. The nurse should remember that although passive range of motion moves the client's extremity, the nurse should stop if the client complains of pain. If the client cannot help with the exercise, he or she should not endure pain as the nurse forces a joint through range of motion. Start with the lower extremities and move toward the head A general rule is to start with the upper extremities and move down the body. Begin with the minimum range of motion the joint can tolerate and increase the amount each week The nurse should take each joint through the full range of motion each time. Position the client prone with the head to one side to begin the exercises The client should be positioned supine with arms at their sides for these exercises.

A nurse is caring for a client who had foot surgery and who has returned to the clinic for a follow up appointment. The nurse is instructing this client about performing range of motion exercises in his toes to promote circulation. Which of the following range of motion exercises can be performed using the toes? Select all that apply. Abduction Flexion Supination Pronation Rotation

ANS: a,b ✅Abduction Abduction is possible to perform with the toes. Abduction involves spreading the toes while feet are flat on the floor, while the toes are flat and not flexed. ✅Flexion Range-of-motion exercises may be indicated as part of muscle strengthening. Depending on the affected area, only certain ROM exercises can be implemented because of the structure of muscles and joints. To perform ROM in the toes, the client can utilize abduction, adduction, flexion, and extension. Flexion involves curling the toes under. Pronation This motion is not possible with the structure of the toes. Rotation This motion is not possible with the structure of the toes. Supination This motion is not possible with the structure of the toes.

A 90-year-old client uses a 2 point gait cane while walking around the long-term care unit where she lives. Which of the following activities indicates that this client is using her cane correctly? Select all that apply. Moving the cane at the same time as the affected leg Holding the cane at the side of the body Holding the cane out far in front of the body Holding the cane on the same side as the stronger leg Leaning onto the cane

ANS: a,b, d ✅Holding the cane on the same side as the stronger leg The cane should be held on the strong side of the body. ✅Moving the cane at the same time as the affected leg The client moves the strong leg forward, then moves the cane along with the weak leg forward at the same time. ✅Holding the cane at the side of the body A cane is a common device used to assist with ambulation, but is often used incorrectly. The client should not lean onto the cane or hold it far out from the body. It is simply to be used as support for the client to walk. Holding the cane out far in front of the body This is not proper a way to use a cane because it will not be a supportive device if held far from the body. Leaning onto the cane This is not the proper way to use a cane. It is dangerous and could result in a fall.

The student nurse is observing the waveforms on an electrocardiogram (EKG) and notices an abnormal P wave on the EKG. The student nurse knows that this is a problem with which of the following? A.Atrial repolarization B.Atrial depolarization C.Ventricular repolarization D. Ventricular depolarization

ANS: b ✅Atrial depolarization The P wave is produced from atrial depolarization. Atrial repolarization The P wave is produced from atrial depolarization, atrial repolarization is not seen on an EKG. Ventricular depolarization Ventricular depolarization produces the QRS complex. Ventricular repolarization Ventricular repolarization produces the T wave.

* Question 5 of 10 A client with hypertension asks the nurse about what non-prescription medications to take for seasonal allergies. Which response from the nurse is correct? A. It is safe for you to take an allergy medicine with your condition B. You will need to use herbal remedies for allergies, since you cannot take any over-the-counter medications with your blood pressure C. You should only take prescription allergy medication and never over-the-counter drugs for allergies D. You should discuss the allergy medication with your provider before buying it

ANS: c

correct * Question 6 of The nurse caring for a client with a poorly functioning SA node knows that which of the following will take over in the conduction pathway to maintain adequate heart rate and perfusion? * A. Bundle of His B. Purkinje Fibers C. AV node D.?Right and Left Bundle Branches

ANS: c ✅AV node It initiates the intrinsic rate of 40-60 bpm if the SA node is unable to initiate an electrical impulse Purkinje Fibers The Purkinje fibers do not initiate an intrinsic rate which is done by the AV node. Right and Left Bundle Branches The Right and Left Bundle Branches initiate an intrinsic rate if the AV node is unable to first. Bundle of His The Bundle of His does not initiate an intrinsic rate as the AV node does.

A patient is at risk for a stroke. Which drug is recommended to prevent platelet aggregation for stroke prevention by the American Stroke Society? a. aspirin (cyclo-oxygenase inhibitor) b. warfarin sodium (Coumadin) c. heparin (anticoagulant) d. alteplase (Activase)

ANS:A ■ Drug class- Antiplatelet ■ Peak time 15 min- 2 hr ■ Duration 4-6 ■ Half/life 15-30 min Non-opioid non-steroidal anti-inflammatory antipyretic Blood thinner Aspirin = binds to Cox 1/Cox 2 (stops platelet aggregation, gi upset, tinnitus, HA, sweating) coagulation modifier drugs

The nurse is helping to manage a client who has suffered an acute MI. Which of the following laboratory findings does the nurse expect to find following this event? Select all that apply. Elevated CKMB Negative CK isoforms Negative substance P Elevated Myoglobin Negative troponins

ANS:A ✅Elevated CKMB Findings in an acute MI an elevated CKMB band level, elevated troponins, elevated myoglobin, and elevated CK Isoforms. Negative troponins After an MI, a client will have positive troponins. Negative substance P Substance P is related to pain, but is not indicative of a myocardial infarction. Elevated Myoglobin Findings in an acute MI an elevated CKMB band level, elevated troponins, elevated myoglobin, and elevated CK Isoforms. Negative CK isoforms A client who experienced an MI will have an elevation in CK isoforms.

The nurse is caring for a client who has just been diagnosed with a urinary tract infection. The provider places orders. Which order needs to be carried out first? A.Administer ciprofloxacin B. Blood cultures C.Administer ceftriaxone D. Vital signs

ANS:B * ✅Blood cultures It is vital to obtain blood cultures before starting any antibiotics. * Administer ceftriaxone Ceftriaxone is an antibiotic which should not be started before blood cultures are drawn. * Administer ciprofloxacin Ciprofloxacin is an antibiotic. Blood cultures must be drawn before an antibiotic is given to the client. * Vital signs Getting vitals can happen in any order, since the client is not in distress. This is not the priority.

A nurse is preparing to administer pain medication to a client who is experiencing pain in his chest from myocardial ischemia. Which factor would the nurse most likely consider when giving pain medication to this client? A.The client is at risk of aspirating the medication B.The client will need to sleep after taking this medication C.The client should receive oxygen when the pain occurs D.The client should learn to notify the nurse if the pain medicine is working

ANS:C ✅The client should receive oxygen when the pain occurs Management for angina pectoris due to myocardial ischemia includes a pain assessment, pain medications, administering oxygen, assessing vital signs, and monitoring cardiac status. Morphine and nitroglycerin are common pain medications given for angina. Angina can occur when the heart is not getting enough oxygen, so the nurse recognizes that the client should receive oxygen to correct this. The client should learn to notify the nurse if the pain medicine is working When pain medicine is given, the nurse will re-assess the client's pain level to ensure the intervention worked. Rather, the client is taught to notify the nurse if the pain medicine is NOT working. The client will need to sleep after taking this medication Sleep is not a requirement of receiving pain medication. The client is at risk of aspirating the medication Pain medication does not carry a risk of aspiration. The client may have a risk of aspiration, but this would be caused by intrinsic client factors, not the medication itself.

A unconscious client is brought into the trauma bay after being "found down" in the community. There is no reportable history and no witnesses to any events leading up to the client being found. All except which of the following diagnostic tests would be appropriate for this client while in the trauma bay? A.Finger stick for blood sugar B. CT Scan of the brain C. MRI of the brain D.ETOH level

ANS:C * ✅MRI of the brain This is the incorrect diagnostic tool for this client making it the correct answer. This is not a diagnostic test usually performed while in the trauma bay. While is may be diagnostically significant, the MRI is a much longer test than the CT scan and is usually done after other, faster diagnostic exams have proved inconclusive. A CT scan will give quick results that are needed for prompt treatment of this client. CT Scan of the brain A CT scan would be done because with the clients altered level of consciousness, a CT scan of the brain would help to diagnose a neurological condition. * Finger stick for blood sugar A blood sugar assessment would be done because the decreased level of consciousness could be related to a hypoglycemic emergency and would need immediate intervention. * ETOH level An alcohol level is appropriate because the decreased level of consciousness could be related to an elevated ETOH level and would require some intervention depending on the client's GCS. *

A client who has suffered a head injury is brought in to the emergency department. The nurse assesses the client's level of consciousness by first checking for alertness and orientation. What is the most appropriate question for the nurse to ask in order to assess if the client is alert and oriented? A.Did anyone come in with you? B. Do you know where you are right now? C. What month is it? D. Are you having any pain?

ANS:C * ✅What month is it? To assess a client's level of consciousness, the nurse should ask the client their name, the time or date, and where they are. The nurse should avoid yes or no questions and instead focus on those that require a short answer. This question may be frustrating, but it is important to evaluate the type of question being asked (using questions that require more than yes or no). * Do you know where you are right now? This is a yes or no question * Did anyone come in with you? This is a yes or no question * Are you having any pain? This is a yes or no question

A 56-year-old client is being seen at the primary care clinic for follow-up after having chest pain. The client tells the nurse, "I think I had a heart attack yesterday but it could have just been chest pain. It still hurts some, though." The provider orders a troponin level and an ECG. Which response from the nurse is most appropriate? A.If you had a heart attack, the ECG will be back to normal now. We just need this as a baseline B.If you had come in when you were having chest pains, we would have more accurate outcomes. These results may be skewed now C.The ECG level may show damage to your heart, but the troponin level will no longer be accurate D.Both the ECG and troponin levels may be abnormal, even if the heart attack was yesterday

ANS:D ✅Both the ECG and troponin levels may be abnormal, even if the heart attack was yesterday A client may experience a heart attack but may not necessarily know if it was an actual myocardial infarction or simply angina. The outcome may not be obvious right away, but laboratory levels can be drawn that will demonstrate changes in cardiac enzymes showing if the client actually suffered damage to the heart muscle. The troponin levels may remain abnormal for up to ten days after a heart attack. Additionally, an ECG may also demonstrate changes as part of the fully evolved post-myocardial stage, which occurs within hours to days after a heart attack. If you had a heart attack, the ECG will be back to normal now. We just need this as a baseline The ECG would show changes if the client experienced a heart attack. If you had come in when you were having chest pains, we would have more accurate outcomes. These results may be skewed now The results of these two tests will reveal whether or not the client had an MI. The ECG level may show damage to your heart, but the troponin level will no longer be accurate The troponin level will remain abnormal for up to ten days following a cardiac event.

Seizure

Abnormal electrical activity in the brain

S3 Ventricular gallop (kentucky)

Abnormal he's sound Cause- Rapid ventricular filling into a dilated ventricle Location- Mitral if LV Tricuspid if RV

Split S1

Abnormal heart sound Cause-RBBB, Left ventricular pacemaker, Left ventricular ectopy Location -Tricuspid

Sterile

Absence of all microorganisms

absorbent - Antidiarrheal Medications

Absorbs gas

At-risk drinking

According to the Centers for Disease Control and Prevention, having eight or more drinks a week or four or more drinks per occasion is considered at-risk drinking for women. For women, having seven or more drinks a week or three or more drinks per occasion is considered at-risk drinking, according to the National Institute on Alcohol Abuse and Alcoholism.

What are the functions of the pancreas The Digestive System

Acinar units secrete digestive enzymes, beta cells secrete insulin, and alpha cells secrete glucagon

Types of Immunity

Active Passive

Diazepam (Valium)

Adult: (1)seizure activity: 5-10 mg IV q 10-15 min PRN (5 mg over 5 min) to max dose of 30 mg, (2) sedation for cardioversion: 5-15 mg IV 5-10 min prior to cardioversion. rectal 0.5 mg/kg via two in rectal catheter and flush with 2-3 ml air after administration Pediatric dose : (1)seizure activity: 0.2-0.3 mg/kg/dose IV q 15-30 min PRN (no faster than 3 mg over 5 min) to max dose of 10 mg(2) sedation for cardioversion: 5-15 mg IV 5-10 min prior to cardioversion. (3)rectal 0.5 mg/kg via two in rectal catheter and flush with 2-3 ml air after administration Uses: Hemorrhagic stroke (2-10 mg IV) , 1. Calms patient 2. Reduces vertigo, nausea and vomiting 3. Helps patient rest and relax during attack Mechanism of Action: Benzodiazepines—long-acting A. Alter movement of Na, K, Ca & Mg ions B. Stabilized & cell membranes & make cells less responsive C. Decrease excitability & responsiveness of brain neurons Overview 1. Anti-anxiety meds (anxiolytics) depress CNS which increases GABA, therefore producing a relaxation effect. 1. Most common = Benzodiazepines Nursing Points General 1. Benzodiazepines 1. Reduce anxiety 2. Anticonvulsant 3. Sedative effect Nursing Considerations 1. General interventions 1. Caution: 1. Elderly 2. Avoid ETOH - enhances sedative effect 3. Glaucoma 4. Increases fall risk 2. Safety first - sedative effects 3. Drug-drug Interactions 1. Notify MD before starting OTC meds 4. Do NOT abruptly stop 1. Taper down over weeks 2. Watch for signs of withdrawal and toxicity 3. Withdrawal 1. Irritability 2. Sleep disturbances 3. Tremors, usually hand 4. Nausea, vomiting 5. Cramping 6. Restlessness 7. Seizures Toxicity 1. Toxicity 1. Antidote: Flumazenil (Romazicon) IV 2. Symptoms: 1. Confusion and ↓ LOC 2. Impaired balance and motor function 3. CNS depression 3. Possible paradoxical (opposite) reaction: 1. Hallucinations 2. Aggression 4. Can progress to coma, death

Fentanyl (Sublimaze)

Adult: 1 mcg/kg max dose of 150 mcg, slow IV push. IM or nasally. Pediatric: 1 mcg/kg max dose of 150 mcg, slow IV push. IM or nasally. *class*: Opioid Analgesic *Indication* supplement to general anesthesia, continuous IV infusion for purpose of analgesia *Action*: binds to opiate receptors in CNS altering perception of pain, producing CNS depression *Nursing Considerations*: - use caution with increased ICP, head trauma, adrenal insufficiency - avoid use with MAOIs - may cause apnea, laryngospasm, decreased respirations, bradycardia, hypotension - do not consume grapefruit while taking this medication - monitor hemodynamics during administration - assess patient pain scale frequently

Thiamine (Vitamin B1)

Adult: 100 slow IV or IM Pediatric dose; 10-25 mg slow IV or IM

Dextrose

Adult: 12.5- 25 gram slow IV; may be repeated as necessary Pediatric dose: 0.5-1 gram/kg/dose slow IV ; may be repeated as necessary

Lorazepam (Ativan)

Adult: 2-4 mg slow IV at 2 mg/mi or IM; may repeat in 15-20 min. max dose of 8 mg Pediatric dose; 0.05-0.2 mg/kg slow IV, IO slowly over 2 mins. or IM; may repeat in 15-20 min. max dose of 0.2 mg/kg *side IV: Dilute, IM: do not dilute

Lidocaine

Adult: 20 mg for IO administration Pediatric: 0.5 mg/kg to max dose of 20 mg for IO administraion Local anesthetic, medium duration amide prototype: highly selective use−dependent Group IB antiarrhythmic; used for nerve block and post MI ischemic ventricular arrhythmias. Tox: CNS excitation

Vasopressin (ADH)

Adult: 40 units (as a single dose only) IV or IO, to replace the first or second dose of epinephrine in cardiac arrest Pediatric: Not used

Ipratropium Bromide

Adult: 500 mcg NEB q 6-8 hr, may mix neb solution with Albuterol if used within one hr. Pediatric: 250 mcg NEB q 6- 8 hr, may mix neb solution with Albuterol if used within one hr.

Nitro Paste

Adult: CHF/PE, HTN apply a inch, cover with plastic wrap and secure with tape Pediatric: Not recommended

Albterol

Adult: administer 2.5 mg (dilute 0.5 ml of 0.5% solution of inhalation with with 2.5 ml normal saline) in nebulizer over 10-15 min Pediatric: administer 1.25 mg dose (diluted in 3 ml of 0.9% NS) may repeat every 20 min max 3 times

Oxygen

Adult: cardiac arrest and Carbon Monoxide poisoning: 100% , Hypoxemia: 10-15 L/min, via non-rebreather, COPD: 0-2 L/min, via nasal cannula or 28-35 % venturi mask. Be prepared to provide ventilatory support if higher concentrations of oxygen needed Pediatric: cardiac arrest and Carbon Monoxide poisoning: 100% , Hypoxemia: 10-15 L/min, via non-rebreather, COPD: 0-2 L/min, via nasal cannula or 28-35 % venturi mask. Be prepared to provide ventilatory support if higher concentrations of oxygen needed

Nitroglcerin

Adult: tablets 0.3 -0.4 mg SL, may repeat in 3-5 min to max dose of 3 doses, spray 0.4 mg SL, 1-3 sprays, IV infusion: 5 mcg/min increased by 5-10 mcg/min every 5 mins until desired effect Pediatric: not recommenced

Tuberculosis

Agent • Mycobacterium tuberculosis (Note that some strains are becoming resistant to antibiotics.) • Bovine TB (Mycobacterium bovis), which is transmitted through cattle and unpasteurized milk Reservoir • Primarily humans • Diseased cattle • Badgers • Other small mammals Mode Of Transmission • Spread by respiratory droplets • Direct invasion through mucous membranes Incubation • 4 to 12 weeks • Subsequent risk of pulmonary infection is greatest within the first year • Injections may persist for a lifetime Prevention • Education regarding the mode of transmission and early diagnosis • Monitoring of groups at risk (people who are HIV positive, recent immigrants, homeless people, people residing in crowded substandard housing) • Report new cases for public health follow-up • Implement standard precautions and transmission-based airborne precautions immediately with any suspected cases (see p. 65) • Eliminate tuberculosis among dairy cattle • Pasteurize milk" .

A nurse in a long-term care facility has many clients who have difficulties with urinary incontinence and immobility. Which methods could the nurse implement that would most likely reduce moisture content on the skin of these clients? Select all that apply. A. Manage urinary and stool incontinence B. Avoid allowing small amounts of moisture to remain on the skin C. Utilize skin barriers and absorbent materials D. Implement a bladder training program E. Decrease fluid intake to reduce voiding

Answe A,B C, D * ✅Manage urinary and stool incontinence The contents of bowel and urine are very caustic to the skin, and when the nurse is managing incontinent clients, keeping the skin clean and dry is going to be effective to prevent skin breakdown. ✅Implement a bladder training program Bladder training can assist some clients with continence. ✅Utilize skin barriers and absorbent materials Skin barriers and creams, as well as absorbent materials will assist in preventing skin breakdown between cleanings. Decrease fluid intake to reduce voiding Decreased fluid intake leads to dehydration which leads to many problems. This is not an appropriate approach to stop skin breakdown. ✅Avoid allowing small amounts of moisture to remain on the skin Excess skin moisture can lead to skin maceration, which is when the skin becomes soft and waterlogged. Macerated skin makes it more prone to skin breakdown. The nurse should ensure the residents are kept clean and dry, which is difficult to achieve in a long-term care facility. Implementing routine skin checks and scheduled cleanings for the incontinent clients will be helpful.

A client presents to the clinic with a history of kidney stones. The client states he is having symptoms of kidney stones again. The nurse knows that which of the following symptom is consistent with this condition? A. Colicky abdominal pain B. Bloating C. Anuria D. Pus at the urethral meatus

Answer A ✅* Colicky abdominal pain When a renal calculus travels down the ureter, the client will experience severe pain in the abdomen, groin, or on one side of the back. Other symptoms of renal calculi include discolored or bloody urine, fever, chills, nausea, and vomiting. * Bloating These are not signs of a renal calculus. This is not a symptom of renal calculus which would include abdominal pain, discolored or bloody urine, fever, chills, nausea, and vomiting. * Pus at the urethral meatus These are not signs of a renal calculus. This is not a symptom of renal calculus which would include abdominal pain, discolored or bloody urine, fever, chills, nausea, and vomiting. * Anuria These are not signs of a renal calculus. This is not a symptom of renal calculus which would include abdominal pain, discolored or bloody urine, fever, chills, nausea, and vomiting

Despite trying multiple interventions to manage a client's behavior, the nurse has decided to use restraints through an order from the provider. In which situation would the client most likely need mitt restraints? * The client is trying to scratch himself * The client needs to stay seated in a chair * The client keeps trying to climb out of bed * The client tried to punch his roommate

Answer A * ✅The client is trying to scratch himself * The client tried to punch his roommate * The client keeps trying to climb out of bed * The client needs to stay seated in a chair

A 76-year-old client is being seen for a follow-up appointment after starting a prescription for Parkinson's disease. The nurse asks the client if she experiences symptoms that indicate changes in blood pressure. Which symptom would the nurse expect to assess if this client is experiencing this side effect? A. Dizziness and fainting B. Epistaxis C. Anxiety and agitation D. Cardiac arrhythmias

Answer A * ✅Dizziness and fainting A client with Parkinson's disease is at increased risk of hypotension because the disease affects the body's ability to maintain normal blood pressure with position changes. Some of the drugs used to manage Parkinson's may also lead to hypotension. The client should be assessed for signs that can indicate a drop in blood pressure, such as dizziness, fainting, lightheadedness, blurred vision, and cool, clammy skin. * Cardiac arrhythmias This is not a symptom of hypertension. * Anxiety and agitation This is not a symptom of hypertension. * Epistaxis This is not a symptom of hypertension.

The nurse is assigned to a client with an acute kidney injury. The nurse understands that hemodialysis will become necessary when which of the following signs or symptoms are demonstrated? * Fluid overload with K 6.4 * Blood pH 7.5 * BUN 18 mg/dL * Malignant hypertension

Answer A * ✅Fluid overload with K 6.4 This answer indicates multiple symptoms of kidney failure, which means the client has complications. This, or severe fluid overload with refractory hypertension, severe metabolic acidosis, or BUN >70 mg/dL indicates the need for hemodialysis. * Malignant hypertension This must be accompanied by another related sign or symptom to truly indicate a need for hemodialysis. * BUN 18 mg/dL This is a normal blood urea nitrogen level, so it does not indicate a need for hemodialysis. * Blood pH 7.5 This pH is slightly alkaline. The client would have to show severe metabolic acidosis in order to require hemodialysis.

A nurse is caring for a client who has an order for a CT scan with contrast. Which of the following medication orders would require further clarification from the provider? * Glucophage * Regular insulin * Furosemide * Cholecalciferol

Answer A * ✅Glucophage When given near IV contrast administration, Glucophage (metformin), can significantly increase the risk for contrast-induced nephropathy and may need to be held and/or additional pre-procedure medications may need to be given. This would require the nurse to call the provider to clarify and get an order to hold the medication. * Cholecalciferol Cholecalciferol is Vitamin D and has no impact on, nor is it impacted by a CT scan with contrast. This order doesn't need to be clarified. * Regular insulin Insulin administration is not impacted by a CT scan. The only thing the nurse would need to consider is timing of meals - ensuring that the client can eat before going to scan if they need to receive their insulin. * Furosemide Clients at risk for kidney issues may receive a fluid bolus and furosemide (Lasix) after the scan to help clear their kidneys, this would not require further clarification.

A nurse is caring for a client who is experiencing trouble with coordination and balance, spastic muscles, and numbness and tingling in the extremities. This client is most likely suffering from which of the following? A. Multiple sclerosis B. Myasthenia gravis C.Fibromyalgia D. Guillain Barre syndrome

Answer A * ✅Multiple sclerosis Multiple sclerosis is caused by a deterioration of the myelin sheath, and is characterized by fatigue, tremors, weakness, bowel and bladder dysfunction, and muscle spasticity. Myasthenia gravis This is characterized by double vision, ptosis, and weakness and fatigue. Fibromyalgia This is characterized by wide spread muscle pain and weakness. Guillain Barre syndrome This is characterized by weakness, breathing problems, and numbness and tingling.

While preparing to give a bed bath to a client with Guillain-Barre syndrome, the nurse knows that this client will likely have which of the following deficits? * Symmetrical weakness * Sensitivity to touch * Weakness to one side of the body * Complete paralysis

Answer A * ✅Symmetrical weakness Guillain-Barre syndrome (GBS) is an autoimmune disorder that attacks the peripheral nervous system, causing progressive symmetrical weakness until almost complete paralysis. Weakness to one side of the body While weakness to one side of the body often remains after a stroke, it is not characteristic of Guillain-Barre syndrome. * * Complete paralysis While Guillain-Barre syndrome does not result in total paralysis, it can lead to near total paralysis. * Sensitivity to touch GBS does not cause a sensitivity to touch.

A client with Parkinson's disease has been started on carbidopa-levodopa for the management of tremors and slow movement. The nurse instructs the client's family about serious side effects of this drug, which include which of the following? A.Tardive dyskinesia B. Ventricular tachycardia C. Catatonia D. Orthostatic hypotension

Answer A * ✅Tardive dyskinesia Carbidopa is a medication prescribed for the treatment of symptoms associated with Parkinson's disease, including tremor and slow movements. Carbidopa works by preventing the breakdown of the neurotransmitter dopamine, which reduces some of the negative effects of Parkinson's disease. Unfortunately, tardive dyskinesia is a side effect that can occur with chronic use of medications such as carbidopa and levodopa. TD causes involuntary movements of the face, mouth, tongue, neck, arms, and legs. * Ventricular tachycardia This is not a side effect associated with carbidopa-levodopa use. * Orthostatic hypotension This is not a side effect associated with carbidopa-levodopa use. * Catatonia This is not a side effect associated with carbidopa-levodopa use.

A 58-year-old client with multiple sclerosis is suffering from bowel incontinence after having a stroke. Which intervention would most likely help the client to manage this condition? A. Teach the client about what foods to eat and how much fluid to drink to help with stool consistency B. Explain that the client needs to increase fiber intake and provide a list of foods to eat C. Tell the client to write down how many bowel movements occur each day D. Discuss with the client why the client is struggling with bowel incontinence

Answer A * ✅Teach the client about what foods to eat and how much fluid to drink to help with stool consistency New onset fecal incontinence after a stroke can be frustrating for a client. The nurse can help the client cope with this situation by developing a bowel program to improve bowel control. The goals of bowel incontinence include an optimal stool consistency, reduced bowel motility, and minimizing how much stool is in the rectum through regular bowel movements. The nurse should teach the client to have a regular intake of fiber, drink plenty of fluids, and avoid food and drink that cause diarrhea or urgency. * Discuss with the client why the client is struggling with bowel incontinence The reason is already known, the client had a stroke. * Explain that the client needs to increase fiber intake and provide a list of foods to eat An appropriate amount of fiber is important, but if the client has too much fiber it will lead to too much bulk in the stool, and can potentially increase incontinence problems. Fiber intake, along with other aspects of a bowel program are necessary to help the client with fecal incontinence. * Tell the client to write down how many bowel movements occur each day It is appropriate for the client to keep track of bowel movements to get an idea of management strategies, but the dietary training component of a bowel program is more important.

A client is brought to the trauma bay after sustaining multiple gunshot wounds to the left flank and abdomen. Assessment reveals gross hematuria and an expanding palpable mass to the left flank. Vital Signs are the following: HR 145 BP 78/42 RR 36 O2 sat 96% The nurse anticipate which of the following diagnosis? A.Major kidney injury B. Minor kidney injury C. Grade III Splenic Laceration D. Grade III Liver Laceration

Answer A ✅Major kidney injury The client is showing signs of shock along with gross hematuria, and a palpable expanding mass over the flank are all signs of a severe kidney injury. * Grade III Liver Laceration Liver lacerations would rarely cause hematuria and the palpable mass over the flank is not indicative of a liver injury. * Grade III Splenic Laceration Splenic lacerations would rarely cause hematuria and the palpable mass over the flank is not indicative of a splenic injury. * Minor kidney injury While the identification of a kidney injury is correct, the presence of hematuria, an expanding mass over the flank, and signs of shock indicate a more severe injury.

The nurse is receiving report on an elderly client who has developed a pressure ulcer from long periods of immobility. The outgoing nurse states the wound is a stage I pressure ulcer. Which of the following findings would the nurse expect to see with a stage I pressure ulcer? A. Intact reddened skin, non-blanchable erythema B. Shallow open ulcer with red wound bed C. Full thickness skin loss with exposed tendon or bone D. Full thickness skin loss extending into subcutaneous tissue

Answer A * ✅Intact reddened skin, non-blanchable erythema A stage I pressure ulcer is the beginning stage in which the skin is intact, but the area is reddened and does not blanch when pressed. The client may complain of pain in the area. The client with a stage I ulcer must be positioned off the pressure area, and frequently repositioned to prevent pressure ulcers in other areas. The nurse will perform frequent skin assessments and keep the client's skin clean and dry. A transparent dressing or hydrocolloid dressing may be applied to facilitate healing. Shallow open ulcer with red wound bed This is a stage II pressure ulcer. Full thickness skin loss extending into subcutaneous tissue This is a stage III pressure ulcer. Full thickness skin loss with exposed tendon or bone This is a stage IV pressure ulcer.

A client delivered her third baby via cesarean section 1 day ago. The client needed an indwelling urinary catheter placed because she had an epidural for the procedure. Which nursing intervention would best prevent a catheter-associated urinary tract infection (CAUTI) in this client? A.Keep the catheter in place only for the least amount of time necessary B.Open the drainage system only to empty the catheter bag C.Apply antibiotic ointment to the catheter insertion site once per shift D.Keep the drainage bag at the level of the client's hip

Answer A * ✅Keep the catheter in place only for the least amount of time necessary A catheter-associated urinary tract infection (CAUTI) occurs when an infection develops in an indwelling catheter and is a source of significant expense and disability as a type of hospital-acquired infection. The nurse can take steps to reduce the risk of a CAUTI by using aseptic technique with the catheter, keeping the drainage bag closed and below the level of the client's bladder, and only using the catheter for the least amount of time necessary. * Open the drainage system only to empty the catheter bag The outflow drainage port should be used to empty the drainage bag, but the drainage system itself should not be opened because pathogens can be introduced and cause infection. * Apply antibiotic ointment to the catheter insertion site once per shift Catheter care should be performed once per shift, which includes cleaning the tube and perineal area with chlorhexidine or warm water and soap, depending on facility policy. Antibiotic ointment is not used. * Keep the drainage bag at the level of the client's hip The drainage bag should be kept below the level of the bladder at all times. Since the hip could be above the bladder depending on the position of the client, this is not a correct landmark at which to place the bag.

A client with hypertriglyceridemia has been ordered to undergo plasmapheresis. Which of the following information is true regarding this procedure? Select all that apply. A. The process is also called therapeutic plasma exchange (TPE) B. Plasmapheresis may be used to remove toxins and clotting factors from circulation C. Plasmapheresis removes plasma from the cellular component of blood D. A centrifuge is used for plasmapheresis E. Plasmapheresis is used for autoimmune diseases to remove certain antibodies

Answer A, B,C ,D ✅Plasmapheresis may be used to remove toxins and clotting factors from circulation This is true of plasmapheresis. ✅Plasmapheresis removes plasma from the cellular component of blood Plasmapheresis describes a process that involves separating plasma from blood cells. The process is somewhat similar to hemodialysis and may be used to remove such elements as toxins, clotting factors, or triglycerides from a patient's circulation. It is also known as therapeutic plasma exchange (TPE). Test taking tip: SATA questions on the NCLEX may have one, some, or all options correct. ✅A centrifuge is used for plasmapheresis This is true of plasmapheresis. * ✅The process is also called therapeutic plasma exchange (TPE) This is true of plasmapheresis. Plasmapheresis is used for autoimmune diseases to remove certain antibodies This is true of plasmapheresis.

A 78-year-old client is recovering from coronary artery bypass graft (CABG) surgery. Because of the client's health status and level of immobility, he is at risk of skin breakdown. Which of the following factors is considered an intrinsic factor in the development of post-operative pressure ulcers? Select all that apply. A. The client's weight B. Difficulties with getting out of bed to the chair C. The client's continence status D. The temperature of the client's room E. Excessive sweating contributing to moist skin

Answer A, C, E * ✅The client's continence status Intrinsic factors come from within the client rather than an outside source. Examples of intrinsic factors include advanced age, mobility status, and the client's weight. A client who is immobile following major cardiac surgery is at increased risk of skin breakdown if he is unable to keep his body clean or if skin breaks down from lack of blood flow. ✅Excessive sweating contributing to moist skin A tendency to sweat excessively is intrinsic, and moist skin increases the risk of pressure ulcers. ✅Difficulties with getting out of bed to the chair A client who is immobile following major cardiac surgery is at increased risk of skin breakdown for many reasons. When the client has difficulties with mobility, this is an intrinsic risk factor for the development of pressure ulcers. ✅The client's weight Weight contributes to difficulty with repositioning and mobility, and is an intrinsic risk factor for skin breakdown, because its source is the client rather than the environment. The temperature of the client's room This is extrinsic to the client, because it originates in the environment rather than the client.

The nurse is managing a client's care who has kidney disease. Which of the following interventions are appropriate because of this condition? Select all that apply. A. Take daily weights at the same time each day B. Monitor urine output C. Ensure the client is following a high protein diet D. Assess apical pulse each shift E. Watch for changes in mental status

Answer A,B E ✅Monitor urine output Urine output decreases as the kidneys become less effective at filtering the blood. Urine output is a sign of how well the kidneys are functioning. ✅Take daily weights at the same time each day The client with kidney disease is unable to get rid of fluid at the same rate as a person with properly functioning kidneys. This client will need daily weights to monitor whether they are retaining fluid. * ✅Watch for changes in mental status If a client with kidney disease has a sudden change in mentation, it may be a sign of cerebral edema or stroke which are increased risks in kidney disease. Ensure the client is following a high protein diet A diet appropriate for kidney disease includes low protein, low sodium, and low potassium. Excessive *or even normal* amounts of these lead to an increased concentration in the blood from the kidney's lack of filtration. Assess apical pulse each shift The pulse rate should be assessed routinely regardless of whether a client has kidney disease or not.

A nurse is caring for a client who is being discharged home after knee surgery. The client will need to have a cane to ambulate at home, as well as wound care assistance. Which referrals will the client need in order to get set up at home? Select all that apply. * Home care nurse Physical therapy Infusion therapy Hospice nurse Respiratory therapy

Answer A,B ✅Home care nurse Referrals are resources outside of the hospital to help the client successfully transition to home. This client needs assistance with wound care by a wound care nurse. Respiratory therapy The scenario does not indicate a need for respiratory assistance. ✅Physical therapy This client needs a physical therapist to help strengthen the client once home. Infusion therapy The scenario does not indicate a need for infusions. Hospice nurse The scenario does not indicate a need for hospice

A nurse is providing discharge teaching to a client who has had a colostomy surgically placed during the hospital stay. What information would the nurse most likely include about the stoma for this clent? Select all that apply. The stoma is typically round or oval in shape * * The stoma is painful when touched * The stoma may be slightly swollen just after surgery * The stoma is usually red in color * The stoma is usually flat or inverted

Answer A,B,D * ✅The stoma is painful when touched A stoma has no nerves for feeling, but the client may need analgesics for surgical pain. ✅The stoma is typically round or oval in shape A client with a new colostomy may be surprised at the appearance of the stoma and requires teaching about what to expect as well as how the stoma may change. The stoma site may be slightly swollen just after surgery, but this should resolve with time. The client should understand what a normal stoma looks like in order to detect potential complications if there is a change in appearance. A red or pink stoma indicates high vascularization, which is normal. If the stoma becomes pale pink, the client may have low hemoglobin and hematocrit levels. * . ✅The stoma is usually red in color The client should understand what a normal stoma looks like in order to detect potential complications if there is a change in appearance. A red or pink stoma indicates high vascularization, which is normal. If the stoma becomes pale pink, the client may have low hemoglobin and hematocrit levels. The stoma may be slightly swollen just after surgery A client with a new colostomy may be surprised at the appearance of the stoma and requires teaching about what to expect as well as how the stoma may change. The stoma site may be slightly swollen just after surgery, but this should resolve with time. The stoma is usually flat or inverted The stoma will protrude out slightly from the skin.

A nurse is using the Banner Mobility Assessment Tool (BMAT) to determine a client's level of mobility while in the long-term care center. Which of the following is an element of assessment during this portion of the exam? Select all that apply. A. Ask the client to point and flex the foot B. Ask the client to stand at the bedside and turn in a circle C. Ask the client to stretch the leg forward to straighten the knee D.Ask the client to reach down and try to touch his toes E. Ask the client to reach across midline to shake a hand

Answer A,C, E ✅Ask the client to reach across midline to shake a hand This is an element of the BMAT. ✅Ask the client to stretch the leg forward to straighten the knee This is an element of the BMAT. ✅Ask the client to point and flex the foot The BMAT is an assessment tool the nurse may use to determine a client's level of mobility in order to be able to prevent complications. The tool consists of four assessments: Sit up in bed and shake a hand by reaching across the midline, stretch and straighten a knee then point toes, stand, and march in place then advance a step. Ask the client to stand at the bedside and turn in a circle This is not a typical movement and is not assessed on the BMAT. Ask the client to reach down and try to touch his toes This is not a part of the BMAT

A nurse is discussing options for home hemodialysis with a client who has kidney disease. Which of the following is correct regarding home hemodialysis? Select all that apply. A. Home hemodialysis may involve nocturnal sessions B. The home care nurse will be present to assist with the dialysis process * . C. Home hemodialysis promotes client independence * D. The catheter system must remain sterile in the home E. Home hemodialysis sessions are shorter than outpatient sessions

Answer A,C,D, ✅Home hemodialysis promotes client independence Home peritoneal hemodialysis is an option for some clients with renal failure. It allows the client to live a more normal life, and often the client can perform dialysis independently. ✅The catheter system must remain sterile in the home The same rules of cleanliness apply in the home that would apply in a facility. If there is a catheter that enters the body internally, it must be kept clean. In this case, sterility must be maintained to avoid an infection. * ✅Home hemodialysis may involve nocturnal sessions There are different types of peritoneal dialysis; continuous cycling, intermittent and nightly peritoneal dialysis. All involve the instillation of dialysate into the peritoneum, allowed to dwell, and drained. This process of exchange is repeated on a schedule based on which type of dialysis is being utilized. Home hemodialysis sessions are shorter than outpatient sessions Peritoneal dialysis is the method used in the home, dwell time is factored into the session time, which takes longer than a targeted hemodialysis done with a machine in a facility. The home care nurse will be present to assist with the dialysis process The client who is proficient with peritoneal dialysis may perform the process independently in the home.

The nurse is caring for a client with Parkinson's disease. During the physical assessment, the nurse notes that which of the following signs and symptoms are consistent with this disease? Select all that apply. A. Difficulty Swallowing Tardive dyskinesia B. Drooping of eyelids C. Pill-rolling tremor D. Shuffling gait

Answer A,D,E ✅Difficulty swallowing This is a common symptom in the client with Parkinson's disease. Each client should have a swallow study in order to determine whether they are at risk for aspiration with oral intake. * ✅Pill-rolling tremor This describes a tremor in which the thumb moves against the index finger in a way that resembles rolling a pill. * ✅Shuffling gait This is a common characteristic of the client with Parkinson's. Drooping of eyelids This is a characteristic of a client with myasthenia gravis, but not Parkinson's. * Tardive dyskinesia Tardive dyskinesia (TD) is a side effect of antipsychotics. With TD, the client experiences involuntary muscle movements. In akinesia, which is common with Parkinson's, the client experiences muscle rigidity. Some medications taken for Parkinson's disease may cause TD, but this is related to medication use, not Parkinson's disease.

A 10-month-old child is recovering from a cleft lip repair, and the provider has determined the child would benefit from restraints post-operatively. Which kind of restraints would the nurse most likely apply to this child? * A jacket restraint * Elbow restraints * None, unless the child demonstrated being uncooperative * A mummy restraint

Answer B * ✅Elbow restraints Elbow restraints are a type of restraint that is wrapped around a client's arm at the elbow to prevent him or her from bending the arm. In this situation, the child may try to bring his hands to his mouth and/or pick at the stitches. Elbow restraints would be applied to prevent him from bending his arms, but still allow movement of the arms at the shoulders. * A mummy restraint The situation does not warrant such a restrictive restraint. * A jacket restraint Hospital staff should never apply restraints as a response to the child being uncooperative. Elbow restraints are used in this situation if the child is picking at the stitches, sucking his or her thumb, or acting in a manner that would harm the surgical site, but not if the child is simply being uncooperative. * None, unless the child demonstrated being uncooperative Hospital staff should never apply restraints as a response to the child being uncooperative. Elbow restraints are used in this situation if the child is picking at the stitches, sucking his or her thumb, or acting in a manner that would harm the surgical site, but not if the child is simply being uncooperative.

A client with diverticulosis had a colostomy placed 5 days ago and is now being discharged. Which of the following would NOT be appropriate discharge teaching for this client? * Increase fluid intake * Increase fiber intake to 25-30 g/day * Masticate fully before swallowing * Let any sealers dry before applying a new appliance

Answer B * ✅Increase fiber intake to 25-30 g/day Fiber intake should be introduced slowly after a colostomy, and only after the first 2 months. Therefore, increasing the fiber intake to 25-30 g/day right away would be inappropriate. * Masticate fully before swallowing Chewing food fully before swallowing can help to take some of the burden off of the intestines for the physical digestion process, especially for hard-to-digest foods. This is appropriate. * Increase fluid intake Clients with colostomies should always make sure they have adequate fluid intake to prevent blockage of the stoma and encourage GI motility. This is an appropriate intervention to teach. * Let any sealers dry before applying a new appliance This is appropriate. If sealing paste is used and the appliance is applied too soon while it is still wet, there is a chance it will not function properly.

A nurse is assessing a client with an indwelling urinary catheter for signs of a catheter-associated urinary tract infection (CAUTI). Which assessment finding would best help to identify a CAUTI? A.Temperature of 99.0 F B. Suprapubic pressure and pain C. Excess urine output D. Frequent diarrhea

Answer B * ✅Suprapubic pressure and pain A catheter-associated urinary tract infection (CAUTI) is a potential complication of indwelling catheter use that refers to a bacterial infection that would not have occurred if the catheter were not present. The nurse can assess for signs or symptoms of a CAUTI by checking the catheter site and assessing urine output. The client may complain of suprapubic tenderness or pressure, the urine may be cloudy or bloody with a strong odor, and the client may have a fever over 100.0 F. * Temperature of 99.0 F A temperature of 100.0 F can indicate infection, not 99.0 F. * Frequent diarrhea A urinary tract infection does not affect the GI system. * Excess urine output Urine may be cloudy and/or have a strong odor, but excess urine output is not a sign of a CAUTI.

A nurse is teaching a community class about the effects of traumatic brain injury (TBI). During the class a student says, "I thought that a concussion is not the same as a traumatic brain injury." Which response from the nurse is most accurate? * A concussion is the most serious form of TBI and can cause lifelong cognitive changes * A concussion is one type of TBI that may be serious * A concussion is considered a TBI when it occurs in a child * A concussion and a TBI are not the same, but they may cause similar outcomes

Answer B * ✅A concussion is one type of TBI that may be serious A concussion is a form of traumatic brain injury in which a client suffers closed trauma to the head. The concussion may be mild or it can be very serious and cause long-term disability. The person who experiences a concussion should be educated about the symptoms that can indicate problems, including worsening headaches, repeated vomiting, seizures, ringing in the ears, and changes in the sense of smell or taste. * A concussion and a TBI are not the same, but they may cause similar outcomes A concussion is a type of traumatic brain injury. * A concussion is the most serious form of TBI and can cause lifelong cognitive changes A concussion can be mild or severe, but there are other types of traumatic head injuries that are serious as well, including fractures, lacerations, contusions, and hematomas. * A concussion is considered a TBI when it occurs in a child A concussion is always considered a traumatic brain injury.

A nurse is working with a woman who cares for her mother in her home. The mother has Parkinson's disease and has difficulty getting around. What information would most likely help this client to keep her mother safe in their home? A. Have the client wear socks or slippers when going into the bathroom B. Arrange furniture so that there is plenty of space between items to move around C. Do not allow the client to go into the kitchen when food is being prepared D. Replace wood or vinyl floors with carpet

Answer B * ✅Arrange furniture so that there is plenty of space between items to move around A person with Parkinson's disease is at risk of being injured because of changes in mobility and subsequent risk for falls. The nurse should counsel this family to leave enough space between furniture for the client to get through without falling and to decrease clutter to avoid tripping hazards * Replace wood or vinyl floors with carpet Carpet is associated with an increased fall risk, so hard surface flooring is best. * Do not allow the client to go into the kitchen when food is being prepared The client should feel free to move about the house, but if food is being prepared, care should be taken to avoid kitchen accidents that may harm the client, such as from hot surfaces and sharp utensils. * Have the client wear socks or slippers when going into the bathroom People are often in a hurry to get to the bathroom, so it can be a situation where there is an increased risk of falls. Socks and slippers can further increase the risk of slipping and tripping unless they are made to be non-slip.

A client has presented to the emergency department full of energy and unable to sit still. The nurse notes dilated pupils. Vital signs are normal except for tachycardia is present. These assessment findings are consistent with abuse of which substance? A. Alcohol B. Cocaine C. Heroin D.Methamphetamine

Answer B * ✅Cocaine With cocaine abuse, the client will generally appear as overly energetic, with an increased heart rate and dilated pupils. * Heroin With heroin abuse, the client will generally appear to be sleepy with very small pupils. * Methamphetamine Methamphetamine "meth" usage will also have dilated pupils but the client becomes overheated easily and will sweat. They also present with a dry mouth, which causes teeth to rot. Meth users also tend to pick at their skin, and appear malnourished due to appetite suppression from the drug. * Alcohol Clients who are addicted to alcohol will typically be slurring their speech, and generally smell of alcohol, with bruising, and may present with injuries sustained while intoxicated.

The nurse notes that a client in the emergency department waiting room is nodding off. The nurse takes their vital signs which are within normal limits. The nurse notes bilateral pinpoint pupils. Which substance does the nurse suspect is being abused by this client? A. Cocaine B. Heroin C. Methamphetamine D. Alcohol

Answer B * ✅Heroin A person using heroin will generally appear to be sleepy with very small pupils. * Cocaine A person using cocaine will generally appear as overly energetic, with an increased heart rate and dilated pupils. * Methamphetamine Methamphetamine, or "meth" usage will present with dilated pupils, poor oral hygiene, and having a malnourished appearance. * Alcohol An alcoholic may be nodding off, but will not have pinpoint pupils. They may slur their speech.

A client has acute renal failure. Which of the following is the most serious complication of acute renal failure? A. Hypernatremia B. Hyperkalemia C.Hypokalemia D. Hyponatremia

Answer B * ✅Hyperkalemia Acute renal failure causes an increase in certain serum electrolytes, including potassium and sodium. The most serious complication of acute renal failure is hyperkalemia, because this affects the client's heart rhythm and is potentially fatal. * Hyponatremia Hyponatremia may occur during certain stages of kidney failure, but in ARF it is more likely for the client to present with hypernatremia than hyponatremia. * Hypernatremia While hypernatremia is a symptom of ARF, it is not as dangerous for a client as hyperkalemia. * Hypokalemia Decreased serum potassium is not associated with acute renal failure.

An 80-year-old client is confused because he has developed a urinary tract infection. The client cowers and screams when the nurse enters the room. Which action of the nurse best demonstrates that she is attempting to minimize the client's fear in this situation? A.Help the client to verbalize that he is not afraid of the nurse * B. Remain calm and continue to try to reorient the client C. Explain to the client about his condition and provide information in printed form D. Tell the client that everything is okay and he will be just fine

Answer B * ✅Remain calm and continue to try to reorient the client A client who is confused may be afraid of once-familiar people or the healthcare providers caring for him. In this case, the nurse probably cannot reach the client by giving him information to read that will help him with his diagnosis. Instead, the nurse should talk to the client calmly and try to reorient them if possible. * Tell the client that everything is okay and he will be just fine When a client behaves in this manner, they are not likely to believe the nurse who tells them that everything will be just fine. The best approach is to speak directly to the client in a normal voice to reorient the client and remain calm. * Help the client to verbalize that he is not afraid of the nurse The client IS afraid of the nurse, so it would not be helpful to tell the client to verbalize something that they do not feel. * Explain to the client about his condition and provide information in printed form This client is cowering and screaming, so the nurse knows that it would not be effective to provide printed information.

A client is admitted to the unit with glomerulonephritis. Which of the following answers is a common cause of this condition? A. Bacteremia B. Systemic lupus erythematosus C. Acute cystitis D. Staphylococcus infection

Answer B * ✅Systemic lupus erythematosus Glomerulonephritis is a term used to describe an inflammatory injury to the glomerulus. This is most commonly caused by an immunological reaction, such as from systemic lupus erythematosus or scleroderma. B-hemolytic streptococcal invasion of the skin or pharynx and history of pharyngitis or tonsillitis can also predispose a person to develop glomerulonephritis. Loss of kidney function develops as a result of this condition. * Staphylococcus infection Glomerulonephritis is a term used to describe an inflammatory injury to the glomerulus. A staphylococcus infection does not predispose a person to develop glomerulonephritis. * Bacteremia Glomerulonephritis is a term used to describe an inflammatory injury to the glomerulus. Bacteremia is a blood infection that does not predispose a person to develop glomerulonephritis. * Acute cystitis Glomerulonephritis is a term used to describe an inflammatory injury to the glomerulus. Acute cystitis is an inflammation of the bladder that does not predispose a person to develop glomerulonephritis.

The provider has ordered a nephrostomy tube for a client in the nurse's care. How should the nurse explain the procedure for nephrostomy tube placement to the client? * The tube is placed through the ureter into the renal pelvis * The tube is placed directly into the kidney to drain urine * The tube is placed into the renal cortex to facilitate excretion * The tube is placed in the ureter above the site of the blockage

Answer B * ✅The tube is placed directly into the kidney to drain urine In nephrostomy tube placement, the tube is placed directly into the kidney in order to drain urine. Any handling of the nephrostomy tube must include strict asepsis by the nurse, since the tube is placed directly into an internal organ. The provider may write an order to irrigate the tube. Irrigation should only be done IF there is an order, using strict aseptic technique, with a maximum of 5 mL sterile normal saline per flush. * The tube is placed through the ureter into the renal pelvis The tube enters through the skin in the client's back. * The tube is placed into the renal cortex to facilitate excretion The nephrostomy tube terminates in the renal pelvis, or collecting system. * The tube is placed in the ureter above the site of the blockage The tube is not placed in the ureter, but enters directly through the kidney.

A client with Parkinson's disease has tremors in the hands that make eating difficult. Which of the following adaptive devices could the nurse employ that would help this client eat? A. A bendable straw B. Weighted utensils C. A flattened steak knife D. A divided plate

Answer B * ✅Weighted utensils A client with a condition that causes tremor, such as Parkinson's disease, may have a difficult time eating. Shakiness and tremor can be significant enough that a client has trouble meeting nutrition and dietary needs. Weighted utensils can be useful for these client, because the heavier weight of the utensils helps to hold the hands steady when bringing food to the mouth. * A divided plate This devices is not helpful in lessening the tremor so that the client can eat. * A bendable straw This devices is not helpful in lessening the tremor so that the client can eat. * A flattened steak knife This devices is not helpful in lessening the tremor so that the client can eat.

A 56-year-old client with heart disease is experiencing angina that increases in intensity and lasts 30 minutes, followed by a period of decreased intensity and duration. Which of the following best describes this type of angina? Decubitus angina Unstable angina Nocturnal angina Intractable angina

Answer B Unstable angina A client who suffers from angina may have constant symptoms or may have pain that comes and goes. A person who experiences angina that increases in intensity and then decreases is having unstable angina, also known as preinfarction angina. It can occur unpredictably from exercise or emotion, and increases in severity over time. Intractable angina This is a chronic angina that can incapacitate a person and is unresponsive to nitroglycerin. Decubitus angina This type of angina occurs when a person is lying down. Nocturnal angina Nocturnal angina occurs at night.

A child was injured in an accident and has become a paraplegic. The child must use crutches on a permanent basis. Which type of crutches would most commonly be used in this situation? A. Platform crutches B. Forearm crutches C. Axillary crutches D. Leg support crutches

Answer B ✅Forearm crutches Crutches are used as methods of support for children or adults who cannot bear weight on their legs, such as after an injury. There are various types of crutches available, which are utilized depending on their need. Forearm crutches have a cuff that encircles the lower arm. These types of crutches tend to be used for long-term use, such as with braces when a child suffers paralysis. Axillary crutches These are for short-term use. Platform crutches This is a subset of a forearm crutch, and is used when a person cannot grip. Whether the child has grip strength or not is unknown from the question, so the correct answer is forearm crutch. Leg support crutches These are used for lower leg injuries rather than spinal cord injuries.

A 20-year-old client is admitted to a rehab unit following a motorcycle accident. Which would be the most appropriate method for measuring the client for crutches? Measure five finger breadths under the axilla Measure 1 1/2 inches under the axilla Measure the client with the elbows flexed at 10 degrees Measure the client with the crutches 18 inches from the side of the foot

Answer B ✅Measure 1 1/2 inches under the axilla When measuring a client for crutches the nurse should measure the crutches to be one to one and one half inches under the axilla. This allows for good control without pinching the tissues of the axilla. Measure five finger breadths under the axilla The proper way to measure for crutches is to use the axilla as the guide, and ensure that the crutch ends one to one and one half inches below the top of the axilla. Measure the client with the elbows flexed at 10 degrees The proper way to measure for crutches is to use the axilla as the guide, and ensure that the crutch ends one to one and one half inches below the top of the axilla. Measure the client with the crutches 18 inches from the side of the foot The proper way to measure for crutches is to use the axilla as the guide, and ensure that the crutch ends one to one and one half inches below the top of the axilla.

The nurse is caring for a client who is newly diagnosed with Parkinson's disease. Which of the following statements by the client's spouse indicates that more education is needed? Select all that apply. A. "I will make sure to increase his fiber and protein intake" B. "I will limit his fluid intake to 2L/day" C. "I will make sure he eats a lot of sunflower seeds" D. "I will provide only a few larger meals during the day" E. "I will follow up to make sure we have appointments scheduled with physical and occupational therapy"

Answer B,C, D ✅"I will make sure he eats a lot of sunflower seeds" Sunflower seeds are high in Vitamin B6, which blocks the effects of antiparkinson's medications. This statement requires further teaching. "I will make sure to increase his fiber and protein intake" Increasing protein and fiber intake is appropriate for the client with Parkinson's disease. * * "✅I will provide only a few larger meals during the day" Clients with Parkinson's benefit from small. frequent, nutrient-dense meals rather than fewer, larger meals. * "✅I will limit his fluid intake to 2L/day" This statement indicates a need for more teaching. The spouse should encourage fluid intake, not limit it. "I will follow up to make sure we have appointments scheduled with physical and occupational therapy" Communicating with PT/OT to ensure they've got appointments lined up is appropriate, and does not indicate the need for further teaching.

A client who suffers from multiple sclerosis (MS) struggles with spasticity of the leg muscles and requires assistance with ambulation. Which ambulatory assistive device would be most appropriate for a client with muscle spasticity? Select all that apply. A. Cane B. Front-wheeled walker C. Crutches D. Standard walker E. Rollator

Answer B,C,D * ✅Standard walker A client with spasticity (stiffness) may find it difficult to ambulate and may need help through an assistive device. The best device is one that is stable and provides good support, such as a walker, cane, or crutches, depending on the severity of the client's condition. Wheels are not recommended on the walker, because they are not as stable as a walker without wheels. * ✅Cane A client with spasticity (stiffness) may find it difficult to ambulate and may need help through an assistive device. The best device is one that is stable and provides good support, such as a walker, cane, or crutches, depending on the severity of the client's condition. Wheels are not recommended on the walker, because they are not as stable as a walker without wheels. * ✅Crutches A client with spasticity (stiffness) may find it difficult to ambulate and may need help through an assistive device. The best device is one that is stable and provides good support, such as a walker, cane, or crutches, depending on the severity of the client's condition. Wheels are not recommended on the walker, because they are not as stable as a walker without wheels. * Front-wheeled walker These devices have wheels, which are not appropriate for a client who has spasticity in the legs. The wheels may cause a fall if the client becomes unstable while ambulating. * Rollator These devices have wheels, which are not appropriate for a client who has spasticity in the legs. The wheels may cause a fall if the client becomes unstable while ambulating.

The nurse is caring for a client who has suffered an acute kidney injury. Which of the following nursing interventions are appropriate? Select all that apply. A. 1 gm NaCl tabs q6hrs B. Fluid restriction C. Monitor I&O D. Daily weights E. Head CT with contrast

Answer B,C,D ✅Monitor I&O A client with an acute kidney injury should be monitored for I&O to see how much fluid is being retained, and how much urine is being made by the kidneys. ✅Daily weights Injury to the kidneys can result in fluid retention, and tracking the client's daily weights is a way to monitor how well the kidneys are working. ✅Fluid restriction With a fluid restriction, the client will retain less fluid and fluid overload will be less of a risk. With fluid overload, the client will begin to demonstrate adventitious heart sounds and crackles in the lungs, which indicates the need for diuresis and/or dialysis. 1 gm NaCl tabs q6hrs Administering salt tablets will cause further fluid to be retained. Salt tabs are contraindicated. Head CT with contrast The contrast dye given with CT scans is contraindicated, because it is hard on the kidneys. Additionally, there is no reason for a head CT for the client in kidney failure.

The nurse is caring for a client who has been diagnosed with chronic kidney failure. Which of the following aspects of this client's history could have contributed to this? A.Generalized anxiety disorder B. Rheumatoid arthritis C.Hypertension D. Transient ischemic attack (TIA) E. Diabetes mellitus

Answer B,C,E * ✅Diabetes mellitus Diabetes can cause chronic kidney failure, because the high glucose level causes damage to the nephrons, which decreases their ability to filter the blood. ✅Hypertension Hypertension causes too much pressure on the structures of the kidneys over time, causing them to fail. * ✅Rheumatoid arthritis Autoimmune disorders can cause chronic kidney failure when the body's immune system begins to attack these organs. * .f Generalized anxiety disorder This does not cause chronic kidney failure. Transient ischemic attack (TIA) This is not a direct cause of chronic kidney failure.

A client has been diagnosed with REM sleep behavior disorder. Which of the following is associated with this type of sleep disorder? Select all that apply. A. The client usually suffers from restless leg syndrome B. The client may be aggressive and violent while asleep C. The client may be dangerous toward their bed partner D. The client often needs psychotherapy and counseling for treatment E. The client may get up out of bed and act out activities

Answer B,C,E ✅The client may get up out of bed and act out activities This is part of REM sleep behavior disorder. ✅The client may be aggressive and violent while asleep This is true of REM sleep behavior disorder. ✅The client may be dangerous toward their bed partner REM sleep behavior disorder describes a type of sleep disorder in which a client may get up out of bed while still asleep and act out dreams. The client may become aggressive or even violent during an episode. The condition is often diagnosed after a bed partner notices the behavior. The client usually suffers from restless leg syndrome This is not associated with REM sleep behavior disorder. The client often needs psychotherapy and counseling for treatment This is not associated with REM sleep behavior disorder.

The client has an acute flare up of diverticulitis. Nursing interventions to prevent complications include which of the following? Select all that apply. Give the client an enema * Give antibiotics by IV * Give antispasmodics * Maintain NPO status * Provide IV fluids

Answer B,D,E ✅Provide IV fluids IV fluids are necessary when the client is NPO to avoid dehydration. * ✅Maintain NPO status During an acute flare up of diverticulitis, the client should have IV fluids and be placed on bowel rest with IV antibiotics. * ✅Give antibiotics by IV IV antibiotics are necessary to reduce infection during a flare-up. Give the client an enema Enemas are contraindicated in acute diverticulitis but are allowed when the client is not having an acute flare. Give antispasmodics Antispasmodics may be given if the client is not having an acute flare-up, but are contraindicated during a flare-up.

A client presents to the clinic with a renal calculus. The nurse knows that which of the following components most commonly makes up a renal calculus? A. Bacteria B. Phosphate C. Calcium D. Uric acid

Answer C * ✅Calcium A urinary tract stone is most commonly made up of calcium oxalate. While they are made of calcium, eating fewer oxalate-rich foods, such as chocolate, potato chips, peanuts and beets, can prevent these stones from forming. Getting plenty of calcium can actually prevent stones from forming. * Uric acid This accounts for less than 5 percent of all renal calculi. This type of renal calculus is more common in men than women, and is caused from urine that is too acidic. * Bacteria Bacterial infection of the kidneys can lead to a rare type of stone called a struvite stone. This is not common. * Phosphate This is not a common component of renal calculi.

The nurse is caring for a client with a chief complaint of angina. Which lab panel does the nurse anticipate being ordered? Renal panel Thyroid panel Cardiac panel Liver panel

Answer C Thyroid panel This is not related to angina and will not help confirm nor rule out a cardiac condition. Liver panel This is not related to angina and will not help confirm nor rule out a cardiac condition. Renal panel This is not related to angina and will not help confirm nor rule out a cardiac condition. Cardiac panel Angina is a condition of the heart involving chest pain due to inadequate blood flow to the heart. The nurse would anticipate a cardiac panel to rule out a myocardial infarction.

The nurse is discharging a postpartum client and newborn baby. During the pregnancy and delivery, the client developed hemorrhoids. Which of the following statements by the client indicates the need for further teaching? * "I will get stool softeners on our way home from the hospital" * "I will take sitz-baths for pain relief" * "I will sit on a heating pad to decrease pain" * "I will increase my fiber intake

Answer C * ✅"I will sit on a heating pad to decrease pain" Using a heating pad will make the hemorrhoids and postpartum bleeding worse, so this statement indicates the need for further teaching. * "I will get stool softeners on our way home from the hospital" This statement indicates an understanding of postpartum hemorrhoid management. Stool softeners will decrease straining which is necessary in order to not worsen the hemorrhoids. * "I will increase my fiber intake" Fiber intake is important in order to bulk up the stools and allow them to pass more easily, so the client demonstrates understanding with this statement. * "I will take sitz-baths for pain relief" Since this is an appropriate action for the client to soothe the perineal area following birth and to manage hemorrhoids, it does not indicate a need for further teaching.

The nurse is caring for a client who has had an indwelling urinary catheter for 2 weeks. The nurse suspects that the client is developing a urinary tract infection. Which of the following assessment findings is inconsistent with this disease process? A.Confusion B.Urinary urgency C.Fruity breath D. Malodorous urine

Answer C * ✅Fruity breath This is NOT a finding associated with a urinary tract infection (UTI). Fruity breath is seen with hyperglycemia. * Confusion Confusion is commonly seen in older adults with a urinary tract infection. Since this finding IS consistent with a UTI, this answer is incorrect. * Malodorous urine This is a normal finding when a client has a UTI. * Urinary urgency This is a an expected finding when a client has a UTI, so this answer is incorrect.

A client presents to the emergency department with abdominal pain. The client states she has not had a bowel movement for one week. An X-ray confirms constipation, and the provider is concerned about a blockage if the client doesn't have a bowel movement soon. Which of the following medications pulls water into the colon to soften the stool? A. Metformin B. Methylprednisone C. Mannitol D. Meperidine

Answer C * ✅Mannitol This medication is a diuretic that uses oncotic pull to bring water into the GI tract, therefore softening the stool. * Meperidine This is an opioid analgesic that is used to treat moderate to severe pain. * Metformin This is an antidiabetic medication used to control blood glucose levels by increasing cellular insulin sensitivity. * Methylprednisone This is a steroid medication that is often used prior to giving a breathing treatment in a client with an asthma attack. It allows albuterol to pass through the bronchioles.

A nurse is caring for a client who is experiencing muscle weakness that gets better with rest. The client reports difficulty swallowing and controlling facial expressions. The nurse knows the client's antibodies are blocking and changing the signals sent between nerves and muscles. This client is suffering from which of the following? * Guillain Barre syndrome * Multiple sclerosis * Myasthenia gravis * Fibromyalgia

Answer C * ✅Myasthenia gravis Myasthenia gravis (MG) is caused by a disconnect between nerve signals and muscles, most often characterized by difficulty swallowing and difficulty controlling facial expressions as well as eye movement. The KEY words to MG is "weakness that gets better with rest". * Multiple sclerosis Multiple Sclerosis (MS) is caused by a deterioration of the myelin sheeth which leads to jumpy and spastic nerve impulses. * Fibromyalgia This has an unknown cause but is characterized by wide spread muscle pain and weakness. * Guillain Barre syndrome This is caused by auto-immune damage to nerves making communication between muscles and the brain to become interrupted. Symptoms do not necessarily improve with rest.

A nurse is caring for an adult client who has put out less than 15 mL per hour of urine for the last 6 hours. The nurse knows that this is which phase of acute kidney injury? A. Anuric B. Polyuric C. Oliguric D. Normouric

Answer C * ✅Oliguric The normal urine output is 30-50 mL/hr of urine for an average sized adult. The term "olig/o" refers to "little" or "less than" and "-uric" refers to urine. Therefore putting out very little urine or lower than normal amounts of urine is considered 'oliguria'. This is also known as the oliguric phase of a kidney injury. * Normouric "normo-" would indicate normal amounts of urine. The normal urine output is 30-50 mL/hr of urine for an average sized adult. This client is putting out much less than that. * Polyuric "poly" would indicate 'many' or 'much' - but this client is putting out LESS than average amounts of urine. Anuric The term "an-" would infer that there is zero or NO urine being produced.

A nurse is helping a client with colostomy irrigation. Which best describes effluent, which is part of this process? * The name of the applicator used to instill the fluid * The fluid that is instilled into the stoma * The stool that comes out of the colostomy * The type of skin barrier that is used to cover the stoma

Answer C * ✅The stool that comes out of the colostomy Colostomy irrigation involves instilling fluid into a client's stoma using a specialized piece of equipment. The stool and liquid that comes out of the stoma after the irrigation is called the effluent. The effluent should be collected in a container or passed into the toilet during the irrigation. * The fluid that is instilled into the stoma This is called the irrigant. * The name of the applicator used to instill the fluid This is done by administering an enema into the ostomy. * The type of skin barrier that is used to cover the stoma This is called an ostomy barrier.

The nurse is caring for a client who presents to the emergency department with angina. Which of the following is the first action the nurse will take? Administer 2 mg morphine IV Give 2L oxygen per nasal cannula Administer nitroglycerin 0.4 mg sublingual Administer 4 baby aspirin at 81 mg each

Answer C Administer nitroglycerin 0.4 mg sublingual The first action the nurse will take out of the four choices is to administer a 0.4 mg sublingual nitroglycerin tab. Give 2L oxygen per nasal cannula This is the third thing the nurse would do, provided that the client's oxygen level is below 94%. If the client's oxygen level is above 94%, they do not need oxygen. Administer 2 mg morphine IV This is the last action the nurse would take, and it is only given if chest pain is unrelieved by nitroglycerin and aspirin. Administer 4 baby aspirin at 81 mg each This is the second action the nurse will take.

A case management nurse is working in a long-term care facility. A new resident has been admitted with symptoms of Parkinson's disease. Which of the following activities best demonstrates that the case manager is promoting client independence? A. Allowing the client to have a private room B. Asking the client to keep track of his or her own medications C. Helping the client to come up with a daily schedule that the client wants to follow D. Allowing the client to stay up late and wander the halls at night if desired

Answer C ✅* Helping the client to come up with a daily schedule that the client wants to follow A case manager has many tasks when working to coordinate client care. A client in a long-term care facility may need help with upholding independence in certain activities, which will protect quality of life. The nurse can best help this client by coming up with a daily schedule that fits the clients needs AND that the client wants to follow, which promotes independence but is not overwhelming for the client. * Allowing the client to have a private room While this may be an appropriate action based on the severity of Parkinson's symptoms, it does not necessarily promote independence. * Asking the client to keep track of his or her own medications This is not an appropriate activity for this client due to their age and condition. If the client makes a mistake with medication, there could be serious consequences. If the client makes a mistake with a daily schedule, there is very little to no harm. * Allowing the client to stay up late and wander the halls at night if desired This does not promote client independence. It causes confusion for the client, and exhaustion during the day

The nurse is educating a client on the proper use of crutches. The client asks why he can't rest his body weight on the pads under his armpits. What is the best explanation by the nurse? * A. "If you put all your weight there it can cause a muscle strain and then you won't be able to use your crutches to walk anywhere" * B. "If you put your weight all in your underarm, you can bruise your underarms" * C. "If you put constant pressure on the nerve in your underarm you can cause paralysis in the arm" * D. "This is how you are supposed to do it. It is the best position for your body"

Answer C "✅If you put constant pressure on the nerve in your underarm you can cause paralysis in the arm" If a client supports their weight on their underarms instead of on their hands they are at risk for crutch palsy, which is paralysis below the point of the nerve that has been under pressure. "If you put your weight all in your underarm, you can bruise your underarms" Crutch palsy is the reason to avoid placing weight on the underarms, not bruising. "If you put all your weight there it can cause a muscle strain and then you won't be able to use your crutches to walk anywhere" This does not necessarily cause a muscle strain, but it will cause crutch palsy "This is how you are supposed to do it. It is the best position for your body" While this statement is true, it is not the best explanation for the client regarding why it's not best.

The nurse is discharging a client who will be using crutches for the first time. The nurse is utilizing the teach-back method to confirm the client's understanding of using crutches. Which of the following indicates that more teaching is necessary? A. He places the crutch approximately 6-10" diagonally in front of his foot B. When going down stairs, he advances the crutches first C. The client watches legs closely as he ambulates D. When going up stairs he moves the injured side last

Answer C ✅The client watches legs closely as he ambulates This is the incorrect way to walk with crutches. The client should look up and forward and not fixate on the lower body as he ambulates. Looking down will increase the risk of a fall. He places the crutch approximately 6-10" diagonally in front of his foot This demonstrates appropriate use of the crutches, so it does not indicate a need for further teaching. When going up stairs he moves the injured side last This demonstrates appropriate use of the crutches, so it does not indicate a need for further teaching. When going down stairs, he advances the crutches first This demonstrates appropriate use of the crutches, so it does not indicate a need for further teaching.

A nurse is assisting a client using a rollator walker to ambulate across the unit. Which indicates that the client is using this device correctly? Select all that apply. A. The client pushes the walker with one arm B. The client leans into the walker C. The client stays close to the walker D. The client shuffles his feet behind the walker E. The client keeps the legs of the walker on the ground when stepping

Answer C,D ✅The client stays close to the walker The client should stay close to the rollator walker, or four-wheeled walker. If a client pushes incorrectly or leans into a walker, they are at risk of a fall and would benefit from education regarding proper use of the rollator. ✅The client keeps the legs of the walker on the ground when stepping When using any type of walker, one must utilize the device correctly in order to gain the most benefit from it. The client should stay close to the rollator walker, or four-wheeled walker, and keep the legs of the walker on the ground, rather than lifting them. The client shuffles his feet behind the walker One of the benefits of using a rollator walker is the increased ability for the client to pick their feet up off the ground when ambulating. If the client shuffles their feet while using the walker, the nurse can attempt to instruct the client to pick up their feet. The client leans into the walker The appropriate client for a rollator walker is one who does not need to lean into the walker. Leaning is dangerous and could result in a fall. The client pushes the walker with one arm The client should use both hands to grip the walker with ambulation.

The nurse is caring for a client who has a kidney stone. Which of the following foods should the nurse teach the client to avoid? Select all that apply. A.Grapes B. Rice C. Milk D. Chocolate E. Potato chips

Answer C,D,E * ✅Milk Stones are mainly made out of phosphorus, calcium and vitamin D, purines, and proteins. The nurse should teach the client to avoid dairy products, sodium, oxalated products such as spinach, soda, teas, and chocolate as well as foods with vitamin D. * * ✅Potato chips Potato chips contain oxalate and therefore contribute to kidney stones. * * ✅Chocolate Chocolate contains oxalate which contributes to kidney stones. Rice This does not cause calculi. * Grapes These do not cause calculi.

A bedridden client requires cradle boots bilaterally to prevent foot drop while in bed. Which of the following describes how to apply cradle boots? Select all that apply. A. Apply the boots only when the client is lying prone B. Ensure that the client wears a pressure dressing around the ankle before applying the boot C. Place the foot into the bottom portion of the boot D. Maintain the foot in the flexed position E. Secure the boot snugly around the foot

Answer C,D,E ✅Maintain the foot in the flexed position When the boots are applied, the client's heel is placed in the boot and the foot is supported in the flexed position with a strap across the top to keep the foot in place. ✅Place the foot into the bottom portion of the boot The bottom portion of the boot must cradle the foot. ✅Secure the boot snugly around the foot Cradle boots may be placed on the client's feet to protect the skin and to keep the feet in alignment. This prevents foot drop. Apply the boots only when the client is lying prone Cradle boots can be worn for the client in the supine, side-lying, semi-Fowler's, and Fowler's positions. Ensure that the client wears a pressure dressing around the ankle before applying the boot A pressure dressing is not required for a cradle boot.

Which of the following is a benefit of using a four-wheel walker for client ambulation? Select all that apply. A. Allows for more leaning capacity B More stable than a regular walker C. Easier to move D. Can be used for any client E. Faster walking speed

Answer C,E ✅Faster walking speed A four-wheel walker is one that has wheels on all four legs of the walker, allowing for a faster walking speed. Since a walker with 4 wheels is less stable, the client must have enough strength to hold themselves up right in order to use this type of walker. ✅Easier to move A four-wheel walker is one that has wheels on all four legs of the walker, making it easy to move and allowing for a faster walking speed. This type of walker is not quite as stable as a walker without wheels and it can only be used for clients who do not need to lean on the walker for balance. Allows for more leaning capacity A walker with 4 wheels should not be used for leaning. More stable than a regular walker This walker is less stable than a walker with 2 wheels and two rubber contact points, or rubber contact points only. Can be used for any client A four-wheel walker should not be leaned on for support, and should not be used by clients who will be putting their weight on the walker. .

A nurse is caring for a client who is in end-stage renal disease and requires dialysis. Choose which alternative best describes the type of diet the nurse should recommend for this client. * A diet low in protein * A diet high in salt * A diet low in carbohydrates * A diet high in protein

Answer D * ✅A diet high in protein Many clients with kidney disease require low-protein diets because of the effects of protein on the kidneys. However, when a client is in end-stage renal disease and requires dialysis, a low-protein diet may not be necessary. In fact, the client may more likely require a high-protein diet at this stage because of the loss of nutrients from dialysis. * A diet low in carbohydrates In ESRD a balance of carbohydrates is important to make sure the body is getting the right amount of energy. * A diet high in salt To much salt causes fluid retention and becomes a problem for the person in ESRD. * A diet low in protein In end stage renal disease, protein intake and absorption is very important. Often, the client with ESRD is malnourished and increased protein intake helps.

The nurse is caring for a client who has chronic kidney disease. Which of the following medications would the nurse question? * Kayexalate * Calcium gluconate * Insulin * Spironolactone

Answer D * ✅Spironolactone This is a medication that should not be given to a client with kidney disease. Hyperkalemia is a major concern, and since spirolactone is a potassium-sparing diuretic it should NOT be used in clients with CKD. * Kayexalate Kayexalate eliminates potassium by binding to it in the intestinal tract. * Insulin Insulin helps transport potassium into cells, it causes the potassium level to decrease, which is a benefit to the client with kidney disease. * Calcium gluconate This drug increases the threshold of cardiac cell contraction in hyperkalemia, which is appropriate for a client with chronic kidney disease who is experiencing arrhythmias.

A nurse is providing dietary discharge teaching to a client who has been diagnosed with cirrhosis who has had an elevated ammonia level. Which of the following statements by the client demonstrate that more education is required? * "I really need to watch my salt intake." * "I'll go to the grocery store and stock up on nutritional shakes." * "I need to make sure I eat a high-calorie diet." * "I'll make sure to eat a high-protein diet."

Answer D * ✅ "I'll make sure to eat a high-protein diet." Ammonia is a byproduct of protein metabolism. Clients with cirrhosis and an elevated ammonia level should be on a low to normal protein diet. This client would require further education. * "I'll go to the grocery store and stock up on nutritional shakes." There is no reason why this client cannot drink these nutrition drinks. They just need to watch their overall protein intake. * "I need to make sure I eat a high-calorie diet." This is appropriate. Clients with liver disease are at high risk for malnutrition, so it is acceptable to ensure a high calorie diet, as long as it is not high in protein. * "I really need to watch my salt intake." Clients with liver disease are at risk for fluid buildup due to improper protein metabolism. Watching salt intake is appropriate to ensure they do not retain too much fluid for other reasons either. However, a full salt restriction is not necessary like it would be with heart or kidney disease.

A client is complaining of chest pain. After reviewing the client's lab values and medical history, the nurse knows which of the following conditions is likely contributing to the client's condition? Labs: K-6.7. Na 135 Mg 2.0 Hgb. 14 A. Aortic stenosis * B. COPD * C. Hyperthyroidism * D. Chronic kidney disease

Answer D * ✅Chronic kidney disease This client has significant hyperkalemia, which can lead to chest pain and lethal arrhythmias. Chronic kidney disease often causes clients to have much higher than normal potassium, which can contribute to this client's condition. * Hyperthyroidism This condition does not tend to cause hyperkalemia, which is likely what is causing this client's chest pain. * COPD This condition does not tend to cause hyperkalemia, which is likely what is causing this client's chest pain. * Aortic stenosis This condition does not tend to cause hyperkalemia, which is likely what is causing this client's chest pain.

A nurse is helping a 47-year-old client with getting up in the morning and performing activities of daily living. The client was diagnosed with myasthenia gravis 6 years ago. Which of the following would the nurse most likely expect the client to find difficult because of this diagnosis? A.Controlling bowel movements B.Sitting in a chair C. Listening to a radio program D. Eating breakfast

Answer D * ✅Eating breakfast Myasthenia gravis is an autoimmune disorder characterized by weakness of skeletal muscles and fatigue with exertion. The client would most likely have difficulties with performing activities of daily living that require exertion or use of muscles, including eating and grooming. It would be less difficult for a client to participate in more passive activities. * Controlling bowel movements Maintaining bowel continence does not require skeletal muscles to accomplish in the adult client. * Sitting in a chair Sitting in a chair is a passive activity that does not require skeletal muscles to accomplish. * Listening to a radio program Listening to a radio program does not require skeletal muscles to accomplish

An adult client presents to the emergency department with scabs all over their skin and missing teeth. The client has dilated pupils and is sweating despite being afebrile. Which substance does the nurse correctly suspect that this client is using? A. Heroin B. Alcohol C. Cocaine D. Methamphetamine

Answer D * ✅Methamphetamine Methamphetamine "meth" usage will cause dilated pupils. This client will become overheated easily and will sweat, have a dry mouth, and pick at their skin. Meth also suppresses hunger so many meth users will be very thin and malnourished. * Heroin Heroin users will generally appear to be sleepy with very small pupils. * Cocaine Persons using cocaine generally appear as overly energetic, with an increased heart rate and dilated pupils. * Alcohol A client who abuses alcohol will typically be slurring their speech and may present with injuries sustained while intoxicated.

The nurse is caring for a client with an ileostomy. During assessment of the client, the nurse notes that the pouch opening is 1/2 inch larger than the stoma site. Which of the following poses a risk for this client? * Stoma pain and burning * Pouch system detachment * Leakage and odor from the site * Peristomal skin breakdown

Answer D * ✅Peristomal skin breakdown The opening of the skin barrier and pouch should be just 1/8 inch larger than the stoma. If the opening is any larger than this, the fecal matter will irritate the surrounding skin and can cause skin breakdown. When applying the skin barrier, the nurse should ensure that the skin around the stoma is clean and dry. The stoma is measured, and an opening 1/8 inch larger is applied to the skin using a skin barrier paste. The bag is then attached to the barrier. * Pouch system detachment The pouch system is generally reliable and would not detach due to an incorrect opening size. The bag should be emptied when it is 1/3 to 1/2 full, and if this is managed appropriately and the pouch is correctly attached, it should stay in place. * Leakage and odor from the site An incorrect opening size would not result in leakage and odor. Rather, leakage and odor occurs if the bag is not properly pressed into the flange, or the bag is not sealed appropriately. * Stoma pain and burning The stoma itself does not contain pain receptors. The skin around the stoma can become painful if the opening is too large, but not the stoma itself.

A nurse is educating a 65-year-old client who has recently been diagnosed with Parkinson's disease. The client asks the nurse, "Will my daughter get Parkinson's when she is my age?" Which answer from the nurse is correct? A. There is no connection between family members with Parkinson's disease and development of the condition B. Parkinson's is caused by an environmental trigger, so she will only develop it if she has the same exposures as you did C. Yes, your daughter will most likely inherit the condition D. There are some genetic links to the disease, but she will not necessarily develop the condition

Answer D * ✅There are some genetic links to the disease, but she will not necessarily develop the condition Parkinson's disease is thought to be caused by a combination of environmental and genetic factors. Approximately 15% of people who with Parkinson's disease have a family history of the disorder. It is more common for the condition to develop in someone who has no family history of Parkinson's. In most cases of Parkinson's disease, genetic mutations are not present. Instead, deposits of proteins called Lewy bodies are found on dying dopamine-producing neurons. * Yes, your daughter will most likely inherit the condition The link between genetics and the disease is weak, and is not well understood. * There is no connection between family members with Parkinson's disease and development of the condition In some cases there seems to be a connection, but in the majority of cases there is not. * Parkinson's is caused by an environmental trigger, so she will only develop it if she has the same exposures as you did Environmental triggers are part of the equation in some cases, but not all.

A nurse is caring for a client who has developed Wernicke's encephalopathy as a result of chronic alcohol abuse. Based on the nurse's knowledge of this condition, the nurse understands that it most commonly occurs because of a deficiency in which of the following vitamins? * Calcium * Vitamin D * Riboflavin * Thiamine

Answer D * ✅Thiamine Wernicke's encephalopathy is a set of neurological symptoms that develops because of a deficiency in thiamine, also called B1. One of the most common causes of the condition is chronic alcohol abuse. The condition most often causes a classic group of symptoms that make its diagnosis easier to consider based on the client's history. Symptoms include ataxia, confusion, and weakness or paralysis of the extraocular muscles. * Riboflavin A deficiency of these vitamins do not cause Wernicke's encephalopathy. * Vitamin D A deficiency of these vitamins do not cause Wernicke's encephalopathy. * Calcium A deficiency of these vitamins do not cause Wernicke's encephalopathy.

A client is continuing to attempt to get out of bed after being informed to stay in bed while recovering from surgery. Which situation best describes the concept of least restrictive device use regarding restraints? * Placing the client in seclusion but not using physical restraints * Applying wrist restraints to keep a client in bed but removing them every 2 hours * Administering medication to control the client's behavior * Utilizing a bed alarm that alarms when the client gets up

Answer D * ✅Utilizing a bed alarm that alarms when the client gets up The concept of using the least restrictive device is implemented into restraint used to identify those situations that would reduce risk to the client but that may not constitute full restraints and restricted movement. The nurse should consider alternatives to standard restraints and find a situation that would keep the client safe and control disruptive behavior whenever possible. An example of this is using a chair or bed alarm to alert staff when a fall-risk client tries to get up without help. * Administering medication to control the client's behavior This is considered a chemical restraint and is not utilized if a non-restraint alternative is available. * Applying wrist restraints to keep a client in bed but removing them every 2 hours This is a physical restraint and should not be used to keep a client in bed, because there are other less restrictive methods to keep a client in bed, such as a bed alarm or sitter. * Placing the client in seclusion but not using physical restraints Seclusion is a type of restraint and can cause the client psychological harm. This should be avoided if possible.

A nurse is caring for a client who has angina. The nurse notes that the client has a prescription for nitroglycerin sublingual tablets. Based on the nurse's understanding of this medication, the nurse knows that this medication works by: Preventing blood clots in the lower extremities Increasing blood pressure to improve cardiac contractility Minimizing atherosclerotic plaques to improve blood flow Dilating blood vessels and increasing venous return

Answer D Dilating blood vessels and increasing venous return Nitroglycerin is a medication used for the treatment of angina when cardiac ischemia develops. Nitroglycerin works by dilating the blood vessels, including those of the coronary arteries, which can improve blood flow and control symptoms of angina. Preventing blood clots in the lower extremities Nitroglycerin dilates blood vessels, but does not have an anticoagulant effect. Minimizing atherosclerotic plaques to improve blood flow Nitroglycerin does not change the development of plaques. Increasing blood pressure to improve cardiac contractility Nitroglycerin is a vasodilator, not a vasoconstrictor

The nurse is caring for a client with Parkinson's disease. The nurse anticipates that which type of medication will be administered to manage the disease? A. SSRIs B. Dopamine antagonists C. Antipsychotic D. Dopaminergics

Answer D ✅ Dopaminergics Dopaminergic medications are used in the treatment of Parkinson's disease by stimulating dopamine receptors to increase the level of dopamine available in the central nervous system. This reduces symptoms of the disease and allows the client maximum function. * Dopamine antagonists A dopamine antagonist is an antidopaminergic, also known as a typical antipsychotic. This type of drug blocks dopamine receptors, which is the opposite effect of what is desired for treatment of Parkinson's. An example of a dopamine antagonist is haloperidol. * SSRIs A selective serotonin reuptake inhibitor is usually prescribed for depression. These are not used in the treatment of Parkinson's. * Antipsychotics An antipsychotic is an antidopaminergic drug, which achieves the opposite effect desired in Parkinson's.

A provider has ordered a serum creatinine test for a client who is being assessed for chronic kidney disease. The client asks the nurse about the test. Which of the following responses correctly explains serum creatinine? * This will tell us if your kidneys are damaged and you will need dialysis * This test tells us if you are releasing excess sugar in your urine * The provider ordered this test to check for an infection * We are testing to see if your kidneys are able to excrete waste through your urine

Answer D ✅* We are testing to see if your kidneys are able to excrete waste through your urine A creatinine test is a test of kidney function that measures the amount of creatinine in the bloodstream. Creatinine is a waste product of creatine, which is produced by the muscles for energy. Creatinine is removed from the body by the kidneys, and increased levels indicate that the kidneys are not able to excrete normal amounts of creatinine from the body. An increased serum creatinine is not seen until there is at least a 50% renal function loss. * This test tells us if you are releasing excess sugar in your urine This is not reflected in a serum creatinine level. * The provider ordered this test to check for an infection Serum creatinine levels are not indicative of an infection. * This will tell us if your kidneys are damaged and you will need dialysis Dialysis is not the first step once it is determined that kidneys are damaged. Dialysis is necessary once a client is in end stage renal disease, have lost 85-90% of kidney function and have a profoundly low glomerular filtration rate.

A nurse is caring for a postpartum client who has severe hemorrhoids that developed during pregnancy. What should the nurse advise the client to do to best manage her discomfort? * Apply a perineal pad soaked in arrowroot oil to prevent itching * Help the client to sit on a heating pad * Apply ice directly to the site of the hemorrhoids for 20 minutes, 3 times per day * Help the client to sit in a sitz bath to soak her perineum

Answer D * ✅Help the client to sit in a sitz bath to soak her perineum Hemorrhoids are a complication of pregnancy that often last into the postpartum period. They can cause intense itching and pain for the affected client. In this situation, the nurse can help the client to sit in a sitz bath, which is a small amount of water in a tub that is designed to cover the perineum, and soak for a period of time. Sitting in this type of bath can provide some relief from the discomfort of hemorrhoids. * Apply ice directly to the site of the hemorrhoids for 20 minutes, 3 times per day Ice should never be applied directly to the client's body, but can be used for hemorrhoids if it is placed into an ice pack. There should be a layer of cloth between the ice and the skin in order to prevent the skin from getting too cold. * Apply a perineal pad soaked in arrowroot oil to prevent itching A perineal pad should only be applied if clean and dry. * Help the client to sit on a heating pad Heat will worsen swelling. Ice is best, but only if applied in the proper covering.

An ostomy nurse is caring for a client who is scheduled for surgery for placement of a colostomy. The nurse explains that it is important to choose the most appropriate site for the stoma. Which of the following regarding stoma location is accurate for the nurse to include in teaching? * The stoma is more likely to become infected if it is not exactly placed in the proper location * Without proper placement, the stoma will be sunken below the skin level * Placement of the stoma in the right location will affect how you feel about having it * If the stoma is not in the right place, you may have trouble reaching it to take care of it

Answer D * ✅If the stoma is not in the right place, you may have trouble reaching it to take care of it One of the most important aspects to consider for the client who will have a stoma is to determine the correct location of the site. A stoma that is poorly positioned may be difficult for the client to reach, which means the client may have a harder time taking care of it. This could lead to a greater risk of skin complications and infection. * The stoma is more likely to become infected if it is not exactly placed in the proper location There is not an exact location, although the curve-under of a client's belly is off-limits because the client cannot see this area. * Without proper placement, the stoma will be sunken below the skin level A retracted stoma can be due to a lack of available intestine to make a proper stoma, or the client may have gained weight, resulting in a sunken stoma. Proper placement on the abdomen is not a reason for a retracted stoma. * Placement of the stoma in the right location will affect how you feel about having it The stoma location does not change the fact that the client now has a stoma, and may need to utilize coping mechanisms as he or she integrates the stoma into everyday life.

A nurse is working with a client who had bowel surgery with placement of a colostomy one month ago. The client has not had any complications following colostomy surgery so far. Which nutritional recommendation should the nurse give that is appropriate for a client in this stage of recovery? * Measure waist circumference once per week * Try new foods frequently to further improve output * Limit the amount of fluids consumed * Thoroughly chew food

Answer D * ✅Thoroughly chew food A client who has had a colostomy placed and who is not having complications can eat a normal diet and enjoy food as before the surgery. Although this client is still in a relatively early stage of recovery, the nurse should reinforce with the client to chew food thoroughly while eating and drink plenty of fluids, because blockages and dehydration are a common problem related to colostomies. * Try new foods frequently to further improve output With new food comes a risk of complications. It is okay for the client with a colostomy to try new foods occasionally, but frequently adding new foods to a diet that is already working well for the client is not necessarily warranted. * Measure waist circumference once per week This is not an aspect of nutritional management in clients with an ostomy. * Limit the amount of fluids consumed The client with an ostomy is at risk for dehydration and blockage, so the nurse should teach the client to stay hydrated

The client presents to the emergency department complaining of chest pain for the past 2 hours. The client is breathing, alert, and oriented. What is the first action the nurse should take? Call the provider Start an IV Get vital signs Get an EKG

Answer D Get an EKG The priority intervention for the nurse is to get an EKG started to verify whether the client is having a myocardial infarction. Vital signs and an IV start will occur immediately after the nurse gets the EKG, or the nurse will delegate to another nurse if one is available. the EKG is a priority because the provider will need to look see the EKG results as soon as they are available. Start an IV The IV is a huge priority, but getting the EKG started is the first action. The more quickly the team identifies a client having an MI, the better the outcome for the client. An IV start and blood draw would be shortly after. Call the provider When a client arrives with chest pain, an EKG is obtained, IV started, vital signs taken and oxygen administered if needed. All these actions can be done without orders from the provider. Get vital signs An EKG must be initiated, then IV start and vital signs quickly after.

A nurse is caring for a client with reduced mobility following hip surgery the day before. Which best describes how the nurse would help prevent skin breakdown from immobility? A. Massage bony prominences with emollient cream after giving the client a bath B. Turn and reposition the client every 4 hours D. Use padding and pillows under the heels and other bony prominences E. Help maintain skin integrity by teaching the client how to move up in bed

Answer D * ✅Use padding and pillows under the heels and other bony prominences Skin breakdown is a consequence of immobility. A client with immobility is at risk of pressure ulcers because of infrequent repositioning, which redistributes pressure to other parts of the body. The nurse can help to prevent skin breakdown by using padding under bony prominences to decrease the amount of pressure placed on these areas. Turn and reposition the client every 4 hours If a client is unable to reposition themselves, they should be repositioned by the nurse at least every 2 hours. Massage bony prominences with emollient cream after giving the client a bath Bony prominences should never be massaged. Massaging applies pressure to areas that have been exposed to too much pressure, so this is unhelpful. Help maintain skin integrity by teaching the client how to move up in bed On the day following hip surgery, a client's mobility is greatly reduced. The client will have hip mobility precautions, and cannot move themselves up in bed due to these precautions.

A nurse is caring for a client who has an order for a CT scan with contrast. Which of the following medication orders would require further clarification from the provider? * Glucophage * Regular insulin * Furosemide * Cholecalciferol

Answer: A * ✅Glucophage When given near IV contrast administration, Glucophage (metformin), can significantly increase the risk for contrast-induced nephropathy and may need to be held and/or additional pre-procedure medications may need to be given. This would require the nurse to call the provider to clarify and get an order to hold the medication. * Cholecalciferol Cholecalciferol is Vitamin D and has no impact on, nor is it impacted by a CT scan with contrast. This order doesn't need to be clarified. * Regular insulin Insulin administration is not impacted by a CT scan. The only thing the nurse would need to consider is timing of meals - ensuring that the client can eat before going to scan if they need to receive their insulin. * Furosemide Clients at risk for kidney issues may receive a fluid bolus and furosemide (Lasix) after the scan to help clear their kidneys, this would not require further clarification.

A nurse is caring for a client who is experiencing gradual paralysis that started from the toes and has moved up the legs. The nurse knows that the client's peripheral nervous system has been attacked causing disruption in communication of the nerves because this client is suffering from which of the following? * Guillain Barre syndrome * Fibromyalgia * Myasthenia gravis * Multiple sclerosis

Answer: A * ✅Guillain Barre syndrome Guillain Barre syndrome is caused by damage to the peripheral nervous system making communication between muscles and the brain become interrupted. The KEY phrase that indicates Guillain Barre syndrome is "paralysis that started from the toes and moved up the legs". Myasthenia gravis This is caused by a disconnect between the nerve and the muscle,and is progressive, but does not start peripherally and move up the legs. The first symptoms are usually seen in the eyes. * Multiple sclerosis This is caused by a deterioration of the myelin sheath, and is characterized by spasticity of muscles rather than paralysis. * Fibromyalgia This condition is characterized by widespread muscle pain and weakness, not paralysis.

The dietary plan for a client with diverticulitis includes which of the following? * High fiber, increased fluids * Low carb, high fiber * Low protein, high fat * High protein, restrict fluids

Answer: A ✅High fiber, increased fluids Clients with diverticulitis have pouches of fecal matter stuck in their diverticula. High fiber, increased fluids, and low fat consumption promote movement in the GI tract. * * Low carb, high fiber The best way to promote movement through the GI tract is through high fiber, low fat, and plenty of fluids. * High protein, restrict fluids The best way to promote movement through the GI tract is through high fiber, low fat, and plenty of fluids. * Low protein, high fat The best way to promote movement through the GI tract is through high fiber, low fat, and plenty of fluids.

An immobile client has developed an area of skin breakdown on the hip. The nurse recognizes that there are several factors that potentially contributed to this skin breakdown. Select all of the following extrinsic factors that would have contributed to skin breakdown in a client. A. Tissue perfusion B. Moisture C. Nutrition D. Shear E. Friction

Answer: B, D, E * Nutrition Nutrition is an intrinsic risk factor. ✅Moisture Moisture is an extrinsic factor. ✅Friction Friction is an extrinsic factor. ✅Shear Skin breakdown can occur as a response to various factors. Extrinsic factors are those in the environment that most likely contribute to a client's loss of skin integrity. Examples of extrinsic factors that contribute to skin breakdown include moisture on the client's skin and skin injury, such as through friction or shear forces. Intrinsic factors include things affecting the client from the inside, like tissue perfusion and nutrition status. Tissue perfusion Tissue perfusion is an intrinsic risk factor.

A nurse is performing a neurological exam on a client who is in the healthcare clinic. Which of the following is a component of assessing deep tendon reflexes? Select all that apply. A normal deep tendon reflex response is 4+ Reflexes are graded from 0 to 4+ * Distract the client briefly while testing Tap the tendon with a quick motion * Ask the client to clench the muscles in the affected area

Answer: B,C,D * ✅Tap the tendon with a quick motion Deep tendon reflexes are tested as part of a client's neurological assessment. To assess these reflexes, the nurse taps the tendon in a quick motion with a reflex hammer, distracting the client as needed so that the client will not try to control the response. * * * ✅Distract the client briefly while testing Distracting the client will help the nurse ensure an accurate response by preventing the client from concentrating on the reflexive movement. * * ✅Reflexes are graded from 0 to 4+ A normal deep tendon reflex response is a 2+, which corresponds to "brisk". A 0 would be no response, while a 4+ corresponds to "very brisk" or "hyperactive". * A normal deep tendon reflex response is 4+ 4+ corresponds to "very brisk" or "hyperactive", which is an abnormal finding Ask the client to clench the muscles in the affected area The client must have relaxed muscles in the area to be tested.

The client has been diagnosed with Bell's palsy. Which of the following is a priority nursing intervention? * Apply steroid cream to the face * Apply cold packs to the affected area * Encourage chewing on the affected side * Assist the client to protect the affected eye

Answer: D * ✅Assist the client to protect the affected eye The client with Bell's palsy has swelling around the facial nerve, affecting innervation to one side of the face. The client should be taught to protect the affected eye, because the client has a decreased ability to close this eye. This causes the client to be susceptible to dryness and injury from debris that could scratch the cornea and permanently affect vision. The nurse can teach the client to use eyedrops regularly to keep the eye moisturized, and manually close the eye or use an eye patch for protection. * Apply cold packs to the affected area Heat should be applied rather than cold. * Encourage chewing on the affected side The client should chew on the unaffected side. * Apply steroid cream to the face Steroids are given orally, not topically.

A client with diverticulosis had a colostomy placed yesterday. Which of the following assessment findings would be the MOST concerning to the nurse? * Pale-pink stoma * Purple stoma * Red stoma * Red skin around stoma

Answer: b * ✅Purple stoma This is the most concerning finding as it indicates severe ischemia and possibly even strangulation of the stoma. This needs to be addressed immediately. * Pale-pink stoma Pale pink is slightly concerning as there may be a decrease in blood flow to the area, but it is not as immediately concerning as the purple stoma. * Red stoma This is a desired finding with a stoma as it indicates the stoma is receiving a healthy blood flow. * Red skin around stoma Skin irritation and redness around the stoma needs to be addressed, but could be due to a sensitivity to the adhesive or barrier paste. It is not as urgent or concerning as a purple stoma.

The nurse is caring for a client who had an ileostomy placed yesterday. The client calls the nurse and frantically states, "My stoma looks really weird and I'm worried something is wrong with it." The nurse notes the stoma to be beefy red. Which response is most appropriate? * "The provider should have explained to you that this is normal for a stoma. You should only notify me if you really need me" * "Your stoma may be infected. I will contact the provider for orders right away" * "Thank you for notifying me so quickly. I will call the provider immediately to take a look" * "Your stoma looks great. A healthy stoma with good circulation should have a beefy red appearance"

Answer: d * ✅ "Your stoma looks great. A healthy stoma with good circulation should have a beefy red appearance" A stoma should be beefy red, which indicates appropriate circulation. * "Thank you for notifying me so quickly. I will call the provider immediately to take a look" There is no need to contact the provider, because this is a healthy color for a stoma. * "The provider should have explained to you that this is normal for a stoma. You should only notify me if you really need me" While this is technically true, it is a condescending way to speak to the client and does not reinforce a healthy nurse-client relationship. * "Your stoma may be infected. I will contact the provider for orders right away" Signs of an infected stoma include reddened skin around the stoma and a fever. The nurse should also be concerned if the stoma is pale as this indicates compromised circulation, but a beefy red stoma on post-operative day 1 is not a concern.

A client who had a total knee replacement has returned from the surgical procedure with a CPM machine in place for range-of-motion exercise. The client asks the nurse why he needs to wear the machine. Which of the following responses from the nurse is correct? A.You will need to wear this machine for the next six weeks B. This will continuously perform range-of-motion exercises when I set up the machine C. This is a machine that replaces rehabilitation exercises for you, so you do not have to do the work D. This machine is designed to provide pain control in addition to muscle flexibility

Answer:B ✅This will continuously perform range-of-motion exercises when I set up the machine A continuous passive motion (CPM) machine provides a passive range of motion for a client who has had medical treatment on a joint. A CPM machine provides the movements at a set rate that is prescribed by the provider. The client must still go through rehabilitation and strengthening exercises following surgery, but the CPM machine is in place to assist with range-of-motion exercises as needed. This is a machine that replaces rehabilitation exercises for you, so you do not have to do the work The client will still do rehabilitation and physical therapy to improve the outcome for the knee. This machine is designed to provide pain control in addition to muscle flexibility The machine does not provide pain relief. You will need to wear this machine for the next six weeks The machine is used intermittently as ordered but is not continuously in use.

An interdisciplinary team works in pulmonary rehabilitation to provide education and pulmonary support to respiratory care clients and their families. Part of the rehabilitation process is reviewing a log or journal with the client for recording exercises that the client performs at home. Which member of the team would most likely be responsible for reviewing this information with the client? A. Provider B. Pharmacist C. Dietitian D. Rehab nurse

Answer:D ✅Rehab nurse The pulmonary rehabilitation nurse may have many responsibilities for client care, including client and family education and explaining the methods of how exercise affects pulmonary function. Another aspect that the pulmonary rehab nurse may perform is teaching the client to record the exercise done at home by writing it down in a journal. Once the client records the information, the nurse can then see the client's exercise regimen and find out how he or she is responding to it. Provider The nurse is usually responsible for reviewing this type of information with the client. Dietitian The nurse is usually responsible for reviewing this type of information with the client. Pharmacist The nurse is usually responsible for reviewing this type of information with the client.

active immunity

Antibodies produced in body; long lasting • Natural Antibodies produced during an active infection • Examples: Chickenpox, mumps, measles • Artificial Vaccine of actual antigens • Examples: Mumps measles, rubella (MMR)

passive immunity

Antibodies produced outside the body; short acting • Natural Antibodies passed from mother to child through placenta and breast milk • Artificial Injected immune serum

Name the three hormones that influence water reabsorption in the body.

Antidiuretic hormone (ADH), aldosterone, and atrial natriuretic hormone (ANH)

high calorie diet

Appropriate for patients with malnutrition, burns and cancer Description -High protein, vitamin, and fat patient complaints -Malnourished

Second-level priority problems

Are next in urgency: They require prompt intervention to prevent further deterioration and may include a mental status change, acute pain, or abnormal laboratory values

Stuporous

Aroused only through pain, no verbal response, never fully awake Levels of Consciousness

Drug treatment of CAD/MI

Aspirin, coumadin, propanolol or metoprolol, nitroglycerin, oxygen Noninvasive surgery Atherectomy Blockage by a single, soft plaque may be reduced with the aid of a long, slender catheter inserted into a coronary artery to the plaque Heart Disease - Coronary Artery Disease

Which of the neuroglia cells help form the blood-brain barrier (BBB)? The Nervous system

Astrocytes: 1. provide supportive framework for blood vessels, neurons, and pia mater 2. limit normal permeability of capillary endothelium by secreting chemicals 3. induce blood brain barrier 4. surround synapses to regulate extracellular concentration of K+ 5. uptakes neurotransmitters 6. regulate CSF composition by regulating the levels of ions and gases

Anticoagulant therapy is appropriate for which conditions? (Select all that apply.) a. Atrial fibrillation b. Thrombocytopenia c. Myocardial infarction d. Presence of mechanical heart valve e. Aneurysm f. Leukemia

Atrial fibrillation, Myocardial infarction(alteplase (Activase) during treatment for acute myocardial infarction.), Presence of mechanical heart valve

When teaching a patient about insulin glargine (Lantus), which statement by the nurse about this drug is correct? "You can mix this insulin with NPH insulin to enhance its effects on glucose metabolism." "You cannot mix this insulin with regular insulin and thus will have to take two injections." "It is often combined with regular insulin to decrease the number of insulin injections per day." "The duration of action for this insulin is 8 to10 hours, so you will need to take it twice a day."

B. "You cannot mix this insulin with regular insulin and thus will have to take two injections."

When caring for a pregnant patient with gestational diabetes, the nurse should question a prescription for which drug? Insulin glargine (Lantus) Glipizide(Glucotrol) Insulin glulisine (Apidra) NPH insulin

B. Glipizide(Glucotrol)

Which is a rapid-acting insulin with an onset of action of less than 15 minutes? A.Insulin glargine (Lantus) B.Insulin aspart (NovoLog) C.Insulin detemir (Levemir) D.Regular insulin (Humulin R)

B. Insulin aspart (NovoLog

A client with diabetes mellitus complains of difficulty seeing. What would the nurse suspect as the causative factor? Lack of glucose in the retina The growth of new retina blood vessels or "neovascularization" Inadequate glucose supply to rods and cones Destructive effect of ketones on retinal metabolism

B. The growth of new retina blood vessels or "neovascularization"

Adolescents communication

Be totally honest with him, even if the information is unpleasant. The first consideration is one's attitude, which must be one of respect. Second, communication must be totally honest. An adolescent's intuition is highly tuned and can detect phoniness or the withholding of information. Always tell him or her the truth. The nurse should avoid periods of silence when communicating with adolescents as silence can be seen as threatening to them. The nurse should not tell an adolescent that everything will be confidential as that is not true. Something need to be reported by law or for the adolescent's well-being. Telling the adolescent not to worry, to go on a diet, or share your own experiences at that age is belittling the adolescent's feelings, providing unsolicited advice, or agreeing with her. Adolescents' increased body awareness and self-consciousness may cause eating disorders such as anorexia nervosa or bulimia, conditions in which the real or perceived body image does not favorably compare with an ideal image. The nurse should not belittle the adolescent's feelings, provide unsolicited advice, or agree with her.

Why is the liver considered an exocrine gland? The Digestive System

Because it secretes bile into ducts, the liver qualifies as an exocrine gland.

Nocturnal enuresis

Bedwetting

Name the substance formed in the liver that aids in the absorption of fats. The Digestive System

Bile

Thrombolytic Drugs: Adverse Effects

Bleeding,Internal Intracranial Superficial Nausea, vomiting, hypotension, anaphylactoid reactions Cardiac dysrhythmias; can be dangerous

The Pulmonary Circuit

Blood flows from right ventricle to pulmonary trunk through pulmonary valve, has three semilunar cusps Pumps blood to pulmonary circuit Carries blood to and from gas exchange surfaces of lungs Pulmonary veins Pulmonary arteries Conus arteriosus (superior end of right ventricle) leads to pulmonary trunk Pulmonary trunk divides into left and right pulmonary arteries Blood flows from right ventricle to pulmonary trunk through pulmonary valve

The Systemic Circuit

Blood leaves left ventricle through aortic valve into ascending aorta Ascending aorta turns (aortic arch) and becomes descending aorta Pumps blood to systemic circuit Carries blood to and from the body Blood alternates between pulmonary circuit and systemic circuit Systemic circulation Blood leaves left ventricle through aortic valve into ascending aorta Ascending aorta turns (aortic arch) and becomes descending aorta

Three cells of bone

Bone contains three types of cells: osteoblasts, osteocytes, and osteoclasts

Name the five main structures of the nephron.

Bowman capsule (glomerular capsule), proximal convoluted tubule, Henle loop (nephron loop), distal convoluted tubule, and collecting duct

Assuming the patient eats breakfast at 8:30 AM, lunch at noon, and dinner at 6:00 AM, he or she is at highest risk of hypoglycemia after an 8:00 AM dose of NPH insulin at what time? 10:00 AM 2:00 PM 5:00 PM 8:00 PM

C. 5:00 PM

The nurse will instruct the patient to treat hypoglycemia with which drug? Acarbose (Precose) Propranolol (Inderal) Glucagon (GlucaGen) Bumetanide (Bumex)

C. Glucagon (GlucaGen)

The health care provider prescribes one tube of glucose gel for the client with type 1 diabetes. The nurse recognizes that this is for treatment of which diabetes complication? Diabetic acidosis Hyperinsulin secretion Insulin-induced hypoglycemia Idiosyncratic reactions to insulin

C. Insulin-induced hypoglycemia

Which oral hypoglycemic drug has a quick onset and short duration of action, enabling the patient to take the medication 30 minutes before eating and skip the dose if he or she does not eat? Acarbose (Precose) Metformin (Glucophage) Repaglinide (Prandin) Pioglitazone (Actos)

C. Repaglinide (Prandin)

A 14-year-old adolescent with diabetes has been self-administering insulin twice a day. This morning the parents found their child lethargic and confused. After the adolescent's admission to the emergency department, laboratory testing reveals a hemoglobin A1c level of 10% and a blood glucose level of 200 mg/dL (11.1 mmol/L). What does the nurse suspect as the most likely cause of this client's condition? Hypoglycemia Somogyi effect Uncontrolled blood glucose level Noncompliance with the prescribed insulin regimen

C. Uncontrolled blood glucose level

Explain the difference between an anabolic process and a catabolic process. Nutrition and Metabolism

Catabolism breaks food molecules down into smaller molecular compounds and, in so doing, releases energy from them. Anabolism builds nutrient molecules up into larger molecular compounds and, in so doing, uses energy.

Name the three divisions of the large intestine. The Digestive System

Cecum, colon, and rectum

What are the diagnostic uses of cerebrospinal fluid samples? The Nervous System

Cerebrospinal fluid can be tested for the presence of blood cells, bacteria, or other abnormal characteristics that may indicate injury or infection, such as meningitis.

Name the four major pairs of plexuses found in the spinal cord. The Nervous System

Cervical plexus, brachial plexus, lumbar plexus, sacral plexus PERIPHERAL NERVOUS SYSTEM

Mania

Characterized by a state of extreme excitement

What are the three major secretory cells found in the gastric glands of the stomach? The Digestive System

Chief cells, parietal cells, and endocrine cells

What are coenzymes and how are they important in metabolism? Nutrition and Metabolism

Coenzymes are organic, nonprotein catalysts that often act as "molecule carriers." Many enzymes or coenzymes are totally useless without the appropriate vitamins to attach to them and thus give the shape that allows them to function properly.

Disseminated Intravascular Coagulation (DIC)

Complex, acquired disorder in which clotting and hemorrhage simultaneously occur In the homozygous protein C deficiency develop widespread intravascular coagulation, resulting in blindness due to retinal artery occlusion, stroke, and coagulopathy, rarely survive infancy. In the heterozygous form of the disease, patients survive into adult life but may develop multiple sys- temic thromboses, including pulmonary embolus or the BuddChiari syndrome. These complications are exacerbated by pregnancy or the oral contraceptive pill, which reduce circulating levels of protein C. Paradoxically, anticoagulants such as warfarin may increase the tendency of these procoagulant phenomena by inhibition of the formation of vitamin K-containing clotting factors, and hence further inhibition of protein C activity.

The nursing diagnosis: Related to

Condition or etiologic factor that appears to show some type of patterned relationship Ex. Acute confusion related to (r/t) elevated sodium level) CANNOT write a medical diagnosis ■DX-Specific problem ■OUTCOME- each action should have an outcome ■INTERVENTION- actions per nurse ■EVALUATION- can the patient accomplish the goal?

Identify four of the integrative functions of the cerebrum The Nervous System

Consciousness, use of language, emotions, memory CENTRAL NERVOUS SYSTEM

Stage I - The Inflammatory Process

Constriction of blood vessels, dilatation of small vessels, increased vessel permeability, increased leukocytes, swelling, and pain. Leukocytes begin to engulf the infection.

Contact Precautions

Contact can be direct or indirect: • Direct is skin-to-skin contact through touch, turning, or bathing. Meningitis (Bacterial) • Indirect contact is made by touching contaminated items, items within the patient's room." EX: Chickenpox/Herpes Zoster Virus (Varicella/Shingles) & Vancomycin-resistant Enterococcus "VRE" , MERSA

The Cuff

Cuff should be 20% wider than the diameter of the limb. Creating a False High Reading • Having a cuff that is too narrow • Having a cuff that is too loose • Deflating the cuff too slowly • Having the arm below the heart • Having the arm unsupported Creating a False Low High Reading • Having a cuff that is too wide • Having a cuff that is too tight • Deflating the cuff too quickly • Having the arm above the heart Creating a False Diastolic Reading • Deflating the cuff too slowly • Having a stethoscope that fits poorly in the examiner's ears • Inflating the cuff too slowly Creating a False Systolic Reading • Deflating the cuff too quickly"

The patient is prescribed 30 units of regular insulin and 70 units of insulin isophane suspension (NPH insulin) subcutaneously every morning. The nurse should provide which instruction to the patient for insulin administration? "Inject the needle at a 30-degree angle." "Rotate sites at least once or twice a week." "Use a 23- to 25-gauge syringe with a 1-inch needle to increase insulin absorption." "Draw up the regular insulin into the syringe first, followed by the cloudy NPH insulin."

D. "Draw up the regular insulin into the syringe first, followed by the cloudy NPH insulin."

The nurse is providing education to a patient for the prescription glipizide (Glucotrol). The nurse explains this medication is more effective when administered at which time? At bedtime In the morning 15 minutes postprandial 30 minutes before a meal

D. 30 minutes before a meal

The nurse is preparing a patient for a computed tomography scan using iodine contrast media. Which medication should the nurse question if prescribed one day before the scheduled procedure? Acarbose (Precose) Pioglitazone (Actos) Repaglinide (Prandin) Metformin (Glucophage)

D. Metformin (Glucophage)

Name the two types of teeth. The Digestive System

Deciduous teeth and permanent teeth

Sleep deprivation

Decrease in the amount and quality of sleep

AGING negative impact on skeletal

Decrease stamina & Muscle mass Decrease hormones collagen & increase bone loss Decrease ability of muscle use oxygen

Sigmoid Colostomy

Descending Colostomy Effluent Semisolid from descending colostomy. Firm from sigmoid colostomy. On discharge, there is an odor. Discharge is irritating if left in contact with skin around stoma. Frequency of output is unpredictable and varies with each person. Skin barrier option May be used for peristomal skin protection if pouch is worn Pouch option Pouch should be worn if person does not irrigate Type of pouch Drainable, closed end, or stoma cap Need for irrigation Yes, as instructed by enterostomal (ET) nurse or physician" .

body mechanics

Describes the safe use of body to maintain balance and proper body alignment. The coordinated effort of JOINTS + LIGAMENTS + TENDONS and muscles and the brain controlled by. cerebellum. Includes "center of gravity" posture, "line of gravity" balance, "support base" weight bearing, full-partial-none

low calorie diet

Description - Decreased fat, no whole milk, cream eggs, complex carbohydrates breads are another alternative; although they lack the phytochemicals, vitamins, and minerals of whole wheat bread, they do provide generous amounts of fiber added in place of some starch.

Tube feeding diet

Description :Formulas or liquid food Patient Complaints :Oral surgery, oral or esophageal cancers, inability to eat or swallow

high protein diet

Description :Meat, fish, milk, cheese, poultry, eggs Patient Complaints :Tissue repair, underweight Defined as more than 35% of total caloric intake from protein; Chronic consumption is generally associated with risk factors for heart disease and some types of cancer; Stresses kidneys through increased urea production (avoid for those with kidney problems); Causes increased need for fluid consumption; With low carb intake follows decreased glycogen sotres, inhibition of performance, and possible dehydration;

diabetic diet

Description Balance of protein, carbohydrates, fat Patient Complaints:Insulin-food imbalance

Incontinence Altered Urinary Patterns

Description Difficulty with control Assess for Infection, injury, distended bladder

Retention - Altered urine pattern

Description Holding on to urine Assess for Infection, injury, pain, distended bladder, medications, restlessness, surgical, complications

Polyuria

Description Increased urination Assess for Infection, injury, alcohol, diabetes, caffeine, diuretics, increased thirst, dehydration

Oliguria

Description Little urination Assess for Infection, injury, elevated BUN, dehydration, kidney, disease

Anuria

Description No urination Assess for Renal failure, dehydration, obstruction

Dysuria

Description Painful urination Assess for Infection, injury, frequency, blood

Urgency

Description Urgent and immediate need to void Assess for Infection"

Nocturia

Description Urinating at night Assess for Infection, injury, pregnancy, stress, intake

Residual - altered urinary

Description Urine remaining in bladder after voiding Assess for Infection, distention, pain, injury

Frequency

Description Voiding small amounts Assess for Infection, injury, pregnancy, stress, intake

urge urinary incontinence

Description involuntary passage of urine with strong urgency Causes- small badder capacity, bladder irritation, alcohol, caffeine Symptoms bladder spasms, urgency, and frequency

stress urinary incontinence

Description- intraabdominal pressure causes leakage Causes- coughing, laughing, obesity, pregnancy, weak muscles Symptoms- urgency and frequency

Reflex Urinary Incontinence

Description- involuntary and occurring at predictable intervals Causes- anesthesia, medication, spinal cord dysfunction Symptoms- lack of urge to void

total urinary incontinence

Description- uncontrolled and continuous loss or urine Causes- neuropathy, trauma, fistula between bladder and vagina Symptoms- constant flow, nocturia, unaware of incontinence

Function Urinary Incontinence

Description-Involuntary and unpredictable with intact urinary and nervous system Causes- changes in environment or cognitive deficits Symptoms- Urge to void that causes loss or urine

Low cholesterol diet

Description: Little meat or cheese Patient Complaints : Need to decrease fat intake no more than 300 mg/day of dietary cholesterol

low sodium diet

Description: No salt added during cooking Patient Complaints: Heart or renal disease no added salt or 1 to 2 g of sodium

Intermittent insomnia

Difficulty remaining asleep

Nutrition-Nursing Diagnoses by Functional Area

Domain 2: Nutrition Class 1: Ingestion Insufficient Breast Milk Ineffective Infant Feeding Pattern Imbalanced Nutrition: Less Than Body Requirements Imbalanced Nutrition: More Than Body Requirements Readiness for Enhanced Nutrition Risk for Imbalanced Nutrition: More Than Body Requirements Impaired Swallowing Class 2: Digestion Class 3: Absorption Class 4: Metabolism Risk for Unstable Blood Glucose Level Neonatal Jaundice Risk for Neonatal Jaundice Risk for Impaired Liver Function Class 5: Hydration Risk for Electrolyte Imbalance Readiness for Enhanced Fluid Balance Deficient Fluid Volume Excess Fluid Volume Risk for Deficient Fluid Volume Risk for Imbalanced Fluid Volume"

What are the two bundles of nerve fibers located in between each vertebra called? The Nervous System

Dorsal nerve root and ventral nerve root CENTRAL NERVOUS SYSTEM

Saline Enemas

Draws fluid into the bowel (9 mL of sodium to 1 L of water; hold for 15 min).

Identify the three divisions of the small intestine The Digestive System

Duodenum, jejunum, and ileum

What are the three meninges? The Nervous System

Dura mater, arachnoid mater, and pia mater

Cognitive/Sensory - Functional Assessment/DX

Educational level Learning needs Communication barriers (list) Memory loss (yes/no) Developmental age Reads English (yes/no) Other languages (list) Sensory Hearing and vision (no problems, impaired, devices) Pain (yes/no, how managed) Domain 5: Perception/Cognition Class 1: Attention Unilateral Neglect Class 2: Orientation Impaired Environmental Interpretation Syndrome Class 3: Sensation/Perception Class 4: Cognition Acute Confusion Chronic Confusion Risk for Acute Confusion Ineffective Impulse Control Deficient Knowledge Readiness for Enhanced Knowledge Impaired Memory Class 5: Communication Readiness for Enhanced Communication Impaired Verbal Communication"

Ileostomy

Effluent A continuous discharge that is soft and wet. The output is somewhat odorous and contains intestinal enzymes that are irritating to peristomal skin. Skin barrier option Highly desirable for peristomal skin protection Pouch option. Pouch necessary at all times Type of pouch. Drainable or closed-end for specific needs Need for irrigation:None .

Transverse Colostomy

Effluent Usually semiliquid or very soft. Occasionally, transverse colostomy discharge is firm. Output is usually malodorous and can irritate peristomal skin. Double-barreled colostomies have two openings. Loop colostomies have one opening but two tracks—the active (proximal), which discharges fecal matter, and the inactive (distal), which discharges mucus. Skin barrier option. Highly desirable for peristomal skin protection Pouch option Pouch necessary at all times Type of pouch Drainable or closed end for specific needs Need for irrigation None"

closed or direct questions.

Elicit hard facts. Call for short one- to two-word answers.

Activity And Exercise- Functional Assessment/DX

Energy level (high, normal, low) Usual exercise and activity patterns (recent changes) Needs assistance with (eating, bathing, dressing) Requirements (cane, walker, wheelchair, crutches) Domain 4: Activity/Rest Class 1: Sleep/Rest Insomnia Sleep Deprivation Readiness for Enhanced Sleep Disturbed Sleep Pattern Class 2: Activity/Exercise Risk for Disuse Syndrome Impaired Bed Mobility Impaired Physical Mobility Impaired Wheelchair Mobility Impaired Transfer Ability Impaired Walking Class 3: Energy Balance Disturbed Energy Field Fatigue Wandering Class 4: Cardiovascular/Pulmonary Responses Activity Intolerance Risk for Activity Intolerance Ineffective Breathing Pattern Decreased Cardiac Output Risk for Ineffective Gastrointestinal Perfusion Risk for Ineffective Renal Perfusion Impaired Spontaneous Ventilation Ineffective Peripheral Tissue Perfusion Risk for Decreased Cardiac Tissue Perfusion Risk for Ineffective Cerebral Tissue Perfusion Risk for Ineffective Peripheral Tissue Perfusion Dysfunctional Ventilatory Weaning Response Class 5: Self-Care Impaired Home Maintenance Readiness for Enhanced Self-Care Bathing Self-Care Deficit Dressing Self-Care Deficit Feeding Self-Care Deficit Toileting Self-Care Deficit Self-Neglect"

Stage II - The Inflammatory Process

Exudation with fluids and dead cells Serous Clear; part of the blood Purulent Thick; pus with leukocytes Sanguineous Bloody

Depression

Feeling of hopelessness or sadness or loss of interest

Fibrinolysis 'Thrombin time'

Fibrinolysis may be suspected on the basis of a reduced fibrinogen, prolonged thrombin time, or the presence of D-dimer or FDPs. Thrombin time is assayed by adding thrombin to plasma and observing the time to fibrin formation, normally 10-12 seconds. occurs as a result of digestion of fibrin by proteolytic enzymes. may be either enhanced (t-PA, streptokinase, urokinase), or inhibited (aprotinin, tranexamic acid, ε-aminocaproic acid).

The glomerular filtrate is generally composed of what substances? The Urinary System

Fluids, waste products, and electrolytes

What is the primary function of the colon? The Digestive System

Formation of feces and the reabsorption of water from the feces

The Right Ventricle

Free edges attach to chordae tendineae from papillary muscles of ventricle Prevent valve from opening backward "Support" Right atrioventricular (AV) valve Also called tricuspid valve Opening from right atrium to right ventricle Has three cusps Prevents backflow Trabeculae carneae Muscular ridges on internal surface of right (and left) ventricle

Incubation- Stages of Infection

From initial contact with infectious material to onset of symptoms

Prodrome phase

From nonspecific signs and symptoms to specific signs and symptoms (prodromal)

Extension

Function Increase angle Examples Straighten elbow or knee, chin straight, hands open, back, fingers, or toes straight Range of Motion (ROM)

Flexion

Function Decrease angle Examples Bend elbow or knee, chin down, make fist, bend at waist or wrist, lift leg, bend toes Range of Motion (ROM)

Abduction

Function Move away from midline Examples Legs or arms away from body, fingers spread apart

Supination

Function - Move joint up Examples -Palm upward, elbow outward" copyright.

Pronation

Function -Move joint down Examples- Palm downward, elbow inward

Inversion

Function -Turn joint inward Examples Foot or hand pointed toward the body

Eversion

Function -Turn joint outward Examples -Foot or hand pointed away from the body

Rotation

Function :Move around axis Examples: Circle of head, hand, foot, leg, arm, fingers, toes

Vitamin K in diet

Function: Aids in blood clotting Sources: Green leafy vegetables"

B12(cobalamin) in diet

Function: Aids muscles, nerves, heart, metabolism Sources: Organ meats, milk

Vitamin E in diet

Function: Antioxidant Sources: Peanuts, vegetable oils"

Magnesium in the diet

Function: Component of enzymes Sources:wheat bran grains, green veggies, nuts, chocolate, legumes

Vitamin D in the diet

Function: Enables body to use calcium and phosphorus Sources: Milk, margarine, fish, liver, eggs" .

Chloride In diet

Function: Formation of gastric juices Sources:Salt" .

A(retinol) in diet

Function: Helps eyes, skin, hair; fights infection Sources: Yellow fruits and vegetables, liver, kidneys, fish" .

C(ascorbic acid) in diet

Function: Maintains integrity of cells, repairs tissue Sources: Citrus fruits, tomatoes, green vegetables, potatoes" Scurvy, slow healing, poor bone growth

potassium in diet

Function: Maintains nerve function Sources: Meat, milk, vegetables

B1 (thiamine) in diet

Function: Maintains nerves, aids carbohydrate function Sources: Bread, cereal, beans, peas, por" .

B2(riboflavin) in diet

Function: Maintains skin, mouth, nerve functions Sources: Milk, cheese, eggs, cereal, dark green vegetables"

Sodium in diet

Function: Maintains water balance, nerve function Sources: Salt, cured meats -Processed foods (75% of US intake, huge source for US) -Table salt -Soft water (calcium replaced with sodium) -Soy sauce -Monosodiumglutamate (MSG)

Adduction

Function: Move toward midline Examples:Legs together, arms at side, fingers together

B3 (niacin) in diet

Function: Oxidation of proteins and carbohydrates Sources: Meat, fish, poultry, eggs, nuts, bread, cereal" .

iodine in diet

Function: Regulates thyroid Sources: Table salt, seafood

Iron in diet

Function: Renews hemoglobin Sources:Meat, eggs, liver, flour, yellow or green vegetables .

Zinc in diet

Function:Component of enzymes Sources:Meat Highly absorbed: meat (organ), dark poultry, seafood (oysters), and cheese Poorly absorbed/ contain phytate: cereal grains, legumes, peas, nuts

Phosphorus in diet

Function:Renews bones and teeth, maintains nerve function Sources: Cheese, oats, meat, milk, fish, poultry, nuts

Calcium in diet

Function:Renews bones and teeth, regulates heart and nerves Sources:Milk, green vegetables, cheese, salmon, legume .

Name the three divisions of the stomach. The digestive System

Fundus, body, and pylorus

What is another name for the digestive tract? Digestive System

Gastrointestinal tract or alimentary canal- mouth, pharynx, esophagus, stomach, small and large

Health Perceptions-Functional Assessment /DX

General health (good, fair, poor) Tobacco or alcohol use (how much, how long) Recreational or prescribed medications (list) Hygiene practices Domain 1: Health Promotion Class 1: Health Awareness Deficient Diversional Activity Sedentary Lifestyle Class 2: Health Management Deficient Community Health Risk-Prone Health Behavior Ineffective Health Maintenance Readiness for Enhanced Immunization Status Ineffective Protection Ineffective Self-Health Management Readiness for Enhanced Self-Health Management Ineffective Family Therapeutic Regimen Management

Generalized anxiety disorder

Generalized anxiety disorder is a pattern of excessive worrying and morbid fear about anticipated "disasters" in the job, personal relationships, health, or finances. With

Febrile seizures

Generally occur in infants and young children; they are most often generalized tonic-clonic seizures. They typically occur in children with a high fever, usually higher than 102°F.

Explain the effect of the hormone ghrelin on digestive function. The Digestive System

Ghrelin is secreted by the endocrine cells. Ghrelin stimulates the hypothalamus to increase appetite and gastrin, which influences digestive functions

Identify the three processes by which the nephron forms urine. The Urinary System

Glomerular filtration, tubular reabsorption, and tubular secretion

Which cranial nerves play an important role in the control of blood pressure? The Nervous System

Glossopharyngeal and vagus nerves

Which hormones act to raise the blood glucose level? Nutrition and Metabolism

Glucagon, epinephrine, growth hormone, ACTH, and glucocorticoids

HEART

Great veins and arteries at the base Pointed tip is apex Surrounded by pericardial sac Sits between two pleural cavities in the mediastinum Three Types of Blood Vessels Four Chambers of the Heart The Pericardium - Double lining of the pericardial cavity "Visceral & Parietal sac"

Summary of Isolation Precautions

Handwashing Should be done before and after working with all patients and after removing gloves; immediately if hands become contaminated with blood or other body fluids Gloves Should be worn whenever contact with body fluids is likely Mask and/or eye cover Should be worn when splashing of body fluids is likely Gown Should be worn when soiling of exposed skin or clothing is likely CPR Should be done with pocket masks or mechanical ventilation, avoiding mouth to mouth Needles Should not be recapped unless using the one-handed method and only if procedure indicates recapping is needed; needles should have safety guards CAUTION: Do not break needles; discard all sharp objects immediately. Private rooms Should be used whenever possible Spills Should be cleaned immediately with bleach and water (one part bleach to nine parts water) for FDA-approved cleaning agent Specimens Should be collected in leakproof, puncture-resistant container; outside of container must be free of contaminants Transporting patients Should be kept to a minimum when working with infected patients

Impaction -Altered Bowel Elimination Patterns

Hard, dry stool embedded in rectal folds; may have liquid stool passing around impaction. Causes: Poor bowel habits, immobility, inadequate food or fluids, or barium in rectum Remedies: Digitally remove impaction, increase fluids and fiber, increase exercise, and institute bowel program." .

modification risk factor- CAD/MI

Heart Disease - Coronary Artery Disease Risk factor modification Stop smoking High blood pressure treatment Dietary modification to lower cholesterol and promote weight loss Stress reduction Increased physical activity (where appropriate)

Coronary artery disease (CAD)

Heart Disease - One of the first symptoms of CAD is commonly angina pectoris Areas of partial or complete blockage of coronary circulation Cardiac muscle cells need a constant supply of oxygen and nutrients. Reduction "narrowing" in blood flow to heart muscle produces a corresponding reduction in cardiac performance Reduced circulatory supply, coronary ischemia, results from partial or complete blockage of coronary arteries Usual cause is formation of a fatty deposit, or atherosclerotic plaque, in the wall of a coronary vessel The plaque, or an associated thrombus (clot), then narrows the passageway and reduces blood flow Spasms in smooth muscles of vessel wall can further decrease or stop blood flow

What is the term for enlargements of the veins in the anal canal? The Digestive System

Hemorrhoids (piles) Enlarged veins near the anus

Bile enters the gallbladder by way of which two ducts? The Digestive System

Hepatic and cystic ducts

Identify the two points at which the colon bends on itself to form 90-degree angles. The Digestive System

Hepatic flexure (right colic flexure) and splenic flexure (left colic flexure)

Hepatitis D virus (HDV)

Hepatitis D can develop only in individuals who have active hepatitis B and in those who are carriers of hepatitis D.

cystic fibrosis

Hereditary metabolic disorders, such as Cystic fibrosis (CF) is caused by a mutation in the cystic fibro sis transmembrane conductance regulator (CFTR) gene located on chromosome 7. This gene encodes the synthesis of a protein that serves as a channel through which chloride enters and leaves some types of epithelial cells. A mutation in this gene alters the cell's ability to regulate the chloride transport (p. 246). Currently more than 1000 mutations can cause CF; several dozens of these account for 90% of the mutations in European Americans. However, the most common mutation, which accounts for 70% of the CF cases, is known as Delta F508. CF is an autosomal recessive disease. A carrier has one mutated CFTR gene. The person affected by CF has mutations on both copies of CFTR. Genetic testing is now used to identify both carriers of CF and neonates with the disease, as well as to detect fetal disease during pregnancy. The sweat chloride test (p. 875) is more easily performed and is a cheaper way to diagnose the disease in affected children. The use of genetic testing for CF is often limited to those with a family history of CF, partners of CF patients, and pregnant couples with a family history of CF. The main purpose of CF genetic testing is to identify carriers who could conceive a child with CF. It is important to recognize that not all patients who have a CFTR genetic mutation will develop the disease. Furthermore, because not all mutations that may cause CF can be detected, a negative test does not necessarily eliminate the possibility of being affected by the disease. Genetic testing can be performed on blood samples or on samples taken during chorionic villus sampling (CVS, p. 254) or during amniocentesis (p. 49)

Sitting position

High Fowler position Areas assessed- head, neck, back, posterior thorax, and lungs, breast, axillea, heart, vital signs, and upper extremities Rational- sitting upright provides full expansion of lungs and provides better visualization of symmetry of upper body parts. Limitations- physical weakened patient may be unable to sit. Examiner should use supine position with head of be elevated instead.

Which of the amino acids are classified as essential? Nutrition and Metabolism

Histidine, isoleucine, leucine, lysine, methionine, phenylalanine, threonine, tryptophan, tyrosine, and valine

Self-Perception -Functional Assessment/DX

How illness or wellness is affecting patient Body image or self-esteem concerns Domain 6: Self-Perception Class 1: Self-Concept Hopelessness Risk for Compromised Human Dignity Risk for Loneliness Disturbed Personal Identity Risk for Disturbed Personal Identity Readiness for Enhanced Self-Concept Class 2: Self-Esteem Chronic Low Self-Esteem Situational Low Self-Esteem Risk for Chronic Low Self-Esteem Risk for Situational Low Self-Esteem Class 3: Body Image Disturbed Body Image"

Hypocortisolism (Addison's Disease)

Hyperkalemia; hyponatremia; hypotension; hypoglycemia; hypoaldosteronism; weakness; fatigue; gastrointestinal disturbances; and skin hyperpigmentation

Hypercortisolism (Cushing's syndrome)

Hypokalemia; hypernatremia; hypertension; hyperglycemia; increased truncal, facial, cervical adipose tissue; muscle wasting; osteoporosis; collagen loss (weakened skin, purple striae); and immunosuppression

symptoms may indicate a serious bleeding problem? a. Hypertension b. Hypotension c. Decreased level of consciousness d. Increased pulse rate e. Restlessness

Hypotension, Decreased level of consciousness, Increased pulse rate, Restlessness monitored during thrombolytic therapy

Because glucose is too large physically and is hydrophilic in nature, how is it transported across the brush border membrane of an intestinal mucosa cell? The Digestive System

In a process called sodium cotransport or coupled transport, carriers bind both sodium ions and glucose molecules and passively transport these molecules together out of the GI lumen.

Angina pectoris

In its most common form, a temporary ischemia develops when the workload of the heart increases Although the individual may feel comfortable at rest, exertion or emotional stress can produce a sensation of pressure, chest constriction, and pain that may radiate from the sternal area to the arms, back, and neck

Incontinence -Altered Bowel Elimination Patterns

Inability to hold feces in rectum because of impairment of sphincter control. Causes Surgery, cancer, radiation treatment of rectum, paralysis, or aging Remedies Bowel training, regular meal times, regular elimination patterns"

Cultural Assessment

Include introductory information, such as: • Patient's name, unit, or room number • Admission date, admitting diagnosis • Proposed length of stay Information about the country can be important: • What is the cultural or ethnic affiliation? • In what country was the patient born? • How many years has he or she been in the United States? • What generation American is the patient? Assess language needs: • Does the patient need an interpreter (what language)? • Does the patient need a communication tool (language board)? • Could the patient use telephone language line (available through most local telephone companies)? Assess for cultural practices: • Are there special rituals that may need to be honored? • Are there special health practices that may need to be honored? • How will the illness affect cultural practices? • How will the illness affect cultural rituals? Assess for cultural supports: • Which cultural or ethnic supports may help your patient? • To whom does the patient turn for help? • How does the patient describe his or her family? • Who is the patient's main source of support? • Who is the patient's main source of hope?"

Bulk forming

Increases fluids and bulk in the intestines, which stimulates peristalsis. An increase in fluid is needed Example: Metamucil Types of Cathartics

Moistening (stool softeners)

Increases water in the bowel Example Colace Types of Cathartics

What are the various mechanisms used by the digestive system to bring essential nutrients into the internal environment so they are available to each cell of the body? The Digestive System

Ingestion, the process of taking food into the mouth, starting it own it's journey though the digestive tract digestion, a group of processes that break down complex nutrients into simple ones thus facilitating their absorption secretion, release of digestive juices ( containing enzymes, acids, bases, mucus, bile) absorption, movement of digestive process

Identify the hormones that are involved in the control of lipid metabolism. Nutrition and Metabolism

Insulin, growth hormone, ACTH, and glucocorticoids

Exercise-Benefits r/t system:

Integumentary- improved tone, color, turgor, resulting from improved circulation Musculoskeletal system- coordination, muscle efficiency, efficiency of nerve impulse transmission Psychological well-being- energy, vitality, positive health behaviors, improved sleep, appearance, self-care

Immobility- risk r/t systems

Integumentary- risk for skin breakdown, formation of pressure injuries Decreased turgor caused by fluid shifts, Increased decubitus ulcers caused by prolonged pressure. Increased skin atrophy caused by decreased nutrition Bowel Changes -Constipation caused by decreased peristalsis Poorer sphincter and abdominal muscle tone Cardiac Changes- Heart rate increase of one-half beat per day caused by increased sympathetic activity Decreased stroke volume and cardiac output caused by increased heart rate Hypotension caused by vasodilatation, leading to thrombosis or edem" Musculoskeletal- muscle wasting, size, tone, strength, stability, joint mobility, flexibility, risk of contracture, bone demineralization Psychological well-being - increase sense of powerlessness, risk of helplessness, decrease self-concepts, social interaction, altered sleep-wake patterns, depression "Respiratory Changes Less alveoli expansion caused by less sighing Increased mucus in lungs caused by less ability to clear them Decreased chest movement restricts lung expansion Stiff intercostal muscles caused by less stretching Shallow respirations leading to decreased capacity Increased secretions caused by supine position of lungs Less oxygen leading to more carbon dioxide, which results in acidosis Atelectasis caused by decreased blood flow Neurosensory Changes Decreased tactile sensation Increased restlessness, drowsiness, and irritability Increased confusion and disorientation caused by hypercalcemia Urinary Changes Poor emptying caused by positioning Urinary stasis leading to more calcium in kidneys, leading to increased renal calculi Urinary retention and distention caused by poor emptying Incontinence caused by poor muscle tone Inability to void caused by overstretching of the bladder Infection caused by stasis and alkalinity Urinary reflux caused by stasis, leading to infections"

Reasons for Urinary Catheters

Intermittent • Relieve bladder distention • Obtain a sterile specimen • Assessment of residual urine • Long-term management of patients with spinal cord injuries and disorders Short-Term Indwelling • After surgery • Prevention of urethral obstruction • Measurement of output in bedridden patients • Bladder irrigation Long-Term Indwelling • Severe urinary retention • Avoidance of skin rashes or infections"

Atrioventricular (AV) valves

Internal Anatomy and Organization Connect right atrium to right ventricle and left atrium to left ventricle Are folds of fibrous tissue that extend into openings between atria and ventricles Two pairs of one-way valves prevent backflow during contractionPermit blood flow in one direction From atria to ventricle Opening from right atrium to right ventricle Has three cusps Prevents backflow Also called tricuspid valve. Semilunar valves Pulmonary and aortic tricuspid valves Prevent backflow from pulmonary trunk and aorta into ventricles Have no muscular support Three cusps support like tripod Blood from left atrium passes to left ventricle through left atrioventricular (AV) valve A two-cusped bicuspidvalve or mitralvalve Blood pressure closes valve cusps during ventricular contraction Papillary muscles tense chordae tendineae to prevent valves from swinging into atria, support the right AV (tricuspid)valve

The Right Atrium

Internal Anatomy and Organization Right Coronary Artery Supplies blood to: Right atrium Portions of both ventricles Cells of sinoatrial (SA) and atrioventricular nodes Marginal arteries (surface of right ventricle) Posterior interventricular artery Superior vena cava Receives blood from head, neck, upper limbs, and chest Inferior vena cava Receives blood from trunk, viscera, and lower limbs Coronary sinus Cardiac veins return blood to coronary sinus & Coronary sinus opens into right atrium Foramen ovale Before birth, is an opening through interatrial septum Connects the two atria Seals off at birth, forming fossaovalis Pectinate muscles Contain prominent muscular ridges On anterior atrial wall and inner surfaces of right auricle

Infection

Invasion of tissues by a disease-causing microorganism(s)

Although both males and females require a large amount of iron during the spurt of growth in the teenage years, why does the iron requirement remain high only in premenopausal women during the rest of adulthood? Nutrition and Metabolism

Iron requirements remain high for women during their reproductive years because adult women must continually replace the iron lost in the menstrual flow.

Nutrients of Concern in Most vegetarian diets

Iron, zinc, calcium, vitamin D, omega-3 fatty acids, and iodine are nutrients of con- cern not because they cannot be obtained in sufficient quantities from plants but because they may not be adequately consumed, depending on an individual's food choices. Vitamin B12 is of concern because it does not occur naturally in plants. ■ Iron- sources Iron-fortified bread and cereals Baked potato with skin Kidney beans, black-eyed peas, and lentils Cooked soybeans Tofu Veggie "meats" Dried apricots, prunes, and raisins Cooking in a cast iron pan, especially with acidic foods such as tomatoes *note -Because of the lower bioavailability of iron from plants, it is recommended that vegetarians consume 1.8 times the normal iron intake. ■ Zinc- sources Whole grains Legumes Zinc-fortified cereals Soybean products Pumpkin seeds Nuts *note- Zinc from plants is not absorbed as well as zinc from meats. Vegetarians are urged to meet or exceed the RDA for zinc. ■ Calcium- sources Bok choy Broccoli Chinese/Napa cabbage Collard greens Kale Calcium-fortified fruit juice Calcium-set tofu Calcium-fortified soy milk, breakfast cereals *note- Spinach, beet greens, and Swiss chard are also high in calcium, but their oxalate content inter- feres with calcium absorption. Calcium supplements are recommended for people who do not meet their calcium requirement through food. ■ Vitamin D- sources Sunlight Fortified milk Fortified ready-to-eat cereals Fortified soy milk Fortified nondairy milk products *note- Supplements may be necessary depending on the quality of sun- light exposure and adequacy of food choices. ■ Omega-3 fatty acids- sources Fortified foods, such as breakfast cereals, soy milk, and yogurt. Sources of alpha-linolenic acid are Ground flaxseed flaxseed oil Walnuts and walnut oil Canola oil Soy *note- Diets that exclude fish, eggs, and sea vegetables do not contain a direct source of omega-3 fatty acids. The body can convert small amounts of alpha-linolenic acid into one of the omega-3 fatty acids (docosahexaenoic acid [DHA]). ■Iodine - sources Iodized salt; sea vegetables *note-Sea salt and kosher salt are gener- ally not iodized; iodine content of sea vegetables varies greatly ■Vitamin B12 Fortified soy milk, breakfast cereals, and veggie burgers *noteSeaweed, algae, spirulina, tempeh, miso, beer, and other fermented foods contain a form of vitamin B12 that the body cannot use. Supplemental vitamin B12 through food or pills is recommended for all people over the age of 50 years regardless of the type of diet they consume because absorption decreases with age.

soapsuds enema

Irritates and distends bowel (5 mL of soap to 1 L of water; hold for 15 min); use only Castile soaps

Sexuality- Functional Assessment/ DX

Last menstrual period, menopause, breast examination Testicular examination How illness may affect sexuality How hospitalization may affect sexuality Any questions, needs, or additional concerns Domain 8: Sexuality Class 1: Sexual Identity Class 2: Sexual Function Sexual Dysfunction Ineffective Sexuality Pattern Class 3: Reproduction Ineffective Childbearing Process Readiness for Enhanced Childbearing Process Risk for Ineffective Childbearing Process Risk for Disturbed Maternal-Fetal Dyad"

Identify five major descending or motor tracts of the spinal cord. The Nervous System

Lateral corticospinal tracts, anterior corticospinal tracts, lateral reticulospinal tracts, medial reticulospinal tracts, and rubrospinal tracts CENTRAL NERVOUS SYSTEM

Protein Foods Group- Lean Choices

Lean Choices (3 g fat/oz) Beef-Select or choice grades of trimmed round, sirloin, and flank steak; tenderloin; rib, chuck, and rump roast; T-bone, porterhouse, and cubed steak; ground round>>all varieties of ground beef (5.5-17 g fat/3 oz cooked). Pork-Fresh ham; canned, cured, or boiled ham; Canadian bacon; tenderloin, center loin chop Lamb- Roast, chop, or leg Veal- Lean chop, roast Poutry- skinless dark meat chicken or turkey, White meat with skin, Skinless domestic duck or goose Fish- Smoked herring, oysters, fresh or canned salmon, catfish, sardines, tuna canned in oil Game- Skinless goose, rabbit Cheese- 4.5% cottage cheese, Grated parmesan Cheese with 3 g of fat or less per ounce Those with 3 g of fat or less per ounce, such as turkey pastrami Other Hot dogs with 3 g of fat or less per Liver, heart ■ Choose a variety of protein foods, which include seafood, lean meat and poultry, eggs, beans and peas, soy products, and unsalted nuts and seeds. ■ Increase the amount and variety of seafood consumed by choosing seafood in place of some meat and poultry. ■ Replace protein foods that are higher in solid fats with choices that are lower in solid fats and calories and/or are sources of oils.

Gastric Ulcers- digestive facts

Located in the antrum of stomach Generally occurs in people ages 45 to 70 years Most common ulcer in people older than 65 More common in women Higher mortality rate than duodenal ulcers Less common than duodenal ulcers Risk factors are stress, drugs, alcohol, smoking, and gastritis Pain occurs 1 to 2 hours after eating Pain felt high in epigastrium Pain may be described as heartburn Pain relieved by food or liquids May cause weight loss High recurrence rate Risk of malignancy High risk of hemorrhage

Name the four prominent cerebral fissures. The Nervous System

Longitudinal fissure, central sulcus (fissure of Rolando), lateral fissure (fissure of Sylvius), parietooccipital fissure CENTRAL NERVOUS SYSTEM

Psychosis

Loss of reality

enoxaparin (Lovenox)

Low-molecular-weight heparins (LMWHs) ■Drug Class -anticoagulant ■Peak time - 3-5 hr ■ Duration - 12 hr ■Half/life - 4.5 hr Drugs affecting Hemostasis

The Nursing Diagnosis: Secondary to

MD- identify diseases; describe problems for which the physician directs the primary treatment; remains the same for as long as the disease is present ■DX- Medical diagnosis ■OUTCOME- consider the time needed to achieve goals ■INTERVENTION- action per patient ■EVALUATION- did the patient accomplish the goal?

Initial treatment for acute coronary syndrome:

MONA: * Morphine * Oxygen * Nitroglycerin * Aspirin (ASA) *note - this is only a mnemonic and not the correct order of administration - see rationale for details* RATIONALE . * Morphine: given ONLY if aspirin and nitroglycerin do not relieve chest pain. Initial dose is 2-4 mg IV. * Oxygen: helps for you to remember to check oxygenation for chest pain - if under 94% or if patient is short of breath give 2L NC initially. Administer oxygen only when clinically relevant. * Nitroglycerin: This is the initial medication given, along with aspirin. This medication dilates the blood vessels to help allow any blood flow that might be impeded. Give 0.4 mg sublingual tab, wait 5 minutes, if the chest pain is not relieved administer another dose. This can happen 3 times total. Monitor a patient's blood pressure, hold for a systolic BP of less than 90 mmHg. * Aspirin: given to thin the blood and decrease mortality risk. A total of 4 baby aspirin (81 mg each) can be given for a total of 324 mg, or a single 325 mg dose.

MRSA and VRE

MRSA Methicillin-resistant Staphylococcus aureus VRE Vancomycin-resistant Enterococcus These are two of the most difficult infections to treat. USE CONTACT PRECAUTIONS for a skin or body fluid, MRSA, or VRE infection. USE DROPLET PRECAUTIONS for a respiratory MRSA infection and for when MRSA is found in tracheal secretions. Always wash hands with chlorhexidine gluconate soap before entering and after leaving each patient's room A) Standard Precautions For MRSA Masks Necessary if patient's respiratory tract is colonized or has an active infection; must use when suctioning or when patient has a productive cough Gowns Necessary if in contact with secretions Gloves Necessary for all contact with items that may be contaminated Private room If possible or with other patients with MRSA and no other infections B)Standard Precautions For VRE Masks Necessary if contact with secretions is likely Gowns Necessary if contact with secretions is likely Gloves Necessary for all contact with items that may be contaminated Private room If possible or with patients who have VRE and no other infections

Medical asepsis (clean technique)

Measures that limit pathologic spread of microorganisms

Surgical asepsis

Measures to keep pathogenic organisms at a minimum during surgery

How does mechanical digestion differ from chemical digestion? Digestive System

Mechanical digestion involves physically breaking down ingested food material into smaller pieces. Chemical digestion completes the breakdown process by mixing digestive secretions resulting in the release of nutrient "end products," such as glucose and amino acids. Enzymes catalyze the chemical reactions that break chemical bonds and facilitate chemical digestion.

Name the three divisions of the brainstem. The Nervous System

Medulla oblongata, midbrain, pons

What is the mechanism of action of each drug? - metformin (Glucophage)

Metformin works primarily by inhibiting hepatic glucose production and increasing the sensitivity of peripheral tissue to insulin, thus lowering blood glucose levels. *discontinue restart it 48 hours after the tests when usingDYE.

Transmission

Method by which microorganisms travel from one host to another

Name the four divisions of the brain. The Nervous System

Midbrain, cerebellum, diencephalon, and cerebrum CENTRAL NERVOUS SYSTEM

SKIN deficiencies

Minerals deficiency--> calcium deficiency - cause osteomalacia in adults iron deficiency-a pale tongue &anemia vitamin C deficiency -causes scorbutic gums (swollen, ulcerated, and bleeding gums,). & scurvy riboflavin deficiency -Magenta tongue

Nursing Diagnosis

ND- focus on unhealthy responses to health and illness; describe the problems treated by nurses within the scope of nursing practice; may change from day to day as the patient's responses change. ■DX- list the diagnosis ■OUTCOME- list the goals ■INTERVENTION- list the intervention ■EVALUATION- n/a

Coping /Stress - Functional Assessment/DX

Needs (social services, financial counselor) May need (home care, nursing home) Coping mechanisms used by patient Domain 9: Coping/Stress Tolerance Class 1: Post-Trauma Responses Post-Trauma Syndrome Risk for Post-Trauma Syndrome Rape-Trauma Syndrome Relocation Stress Syndrome Risk for Relocation Stress Syndrome Class 2: Coping Responses Ineffective Activity Planning Risk for Ineffective Activity Planning Anxiety Defensive Coping Ineffective Coping Readiness for Enhanced Coping Ineffective Community Coping Readiness for Enhanced Community Coping Compromised Family Coping Disabled Family Coping Readiness for Enhanced Family Coping Death Anxiety Ineffective Denial Adult Failure to Thrive Fear Grieving Complicated Grieving Risk for Complicated Grieving Readiness for Enhanced Power Powerlessness Risk for Powerlessness Impaired Individual Resilience Readiness for Enhanced Resilience Risk for Compromised Resilience Chronic Sorrow Stress Overload Class 3: Neurobehavioral Stress Autonomic Dysreflexia Risk for Autonomic Dysreflexia Disorganized Infant Behavior Readiness for Enhanced Organized Infant Behavior Risk for Disorganized Infant Behavior Decreased Intracranial Adaptive Capacity"

Anxiety disorder

No mechanisms to block varying degrees of anxiety

Comatose

No response to pain; no reflexes or muscle tone" Levels of Consciousness

What are the two neurotransmitters that may be released at the axon terminals of autonomic neurons? The Nervous System

Norepinephrine and acetylcholine PERIPHERAL NERVOUS SYSTEM

Odor - Fecal Characteristics

Normal Aromatic Abnormal Pungent

Color- Fecal Characteristics

Normal Brown Abnormal: Clay or White Black or tarry Red Pale Green

Frequency - Fecal Characteristics

Normal 1-2 times a day Once ever three days Abnormal 5 times a day Once every six day

Consistency - Fecal Characteristics

Normal: Moist & Firm Abnormal: Hard Lose Watery Liquid

Shape - Fecal Characteristics

Normal: Cylindric Abnormal: Narrow ribbon like

3- or 5-lead EKG/12- lead ECG

Nursing Interventions Bedside EKG monitoring (3 or 5 Lead) RATIONALE Apply a 3- or 5-lead EKG monitor to determine the presence of any arrhythmias. Most common arrhythmias that cause angina include Atrial Fibrillation with Rapid Ventricular Response (AFib with RVR), Supraventricular Tachycardia (SVT), and Bradycardias. 12- lead ECG If initial 12-lead ECG indicates inferior MI, do a right-sided 12-lead ECG. RATIONALE To rule out the presence of Myocardial Infarction. It takes 5-15 minutes to determine if the chest pain will subside with nitroglycerin. By checking a 12-lead EKG, a possible STEMI can be ruled out immediately. If STEMI is present, patient should be taken to the Cath Lab STAT. Right sided 12 lead ECG shows the right side of the heart to assess for right ventricular ischemia.

Three most common Anti-Infective - Aminoglycosides?

Nursing Points General * Indications * Gram-negative infections 1. Pseudomonas spp. 2. Enterobacteriaceae family * Gram-positive cocci 1. Enterococcus spp. 2. Staphylococcus aureus 3. Bacterial endocarditis * Contraindications * Allergy * Pregnant women * Lactating women Assessment 1. Therapeutic drug monitoring * Maximize drug efficacy * Minimize risk for toxicity * Nephrotoxicity 1. Monitor renal function 2. Creatinine clearance (2x/weekly) 3. Decreased urine output 4. Fluid retention * Ototoxicity 1. Baseline audiogram 2. Audiologist 3. Hearing loss 4. Tinnitus 1. Adverse Effects * Nephrotoxicity 1. Proteinuria 2. Increased BUN 3. Increase serum creatinine level 4. Low urine output 5. Fluid retention * Ototoxicity 1. Hearing loss 2. 8th CN damage 3. Cochlear / vestibular damage 4. Dizziness 5. Tinnitus 6. Fullness in ear 7. Vertigo Therapeutic Management * Antibiotic Dosing * Minimum inhibitory concentration (MIC) 1. Lowest concentration of drug needed 2. Crucial for maximum bacterial death 3. 1- vs 3-day dosing * Time vs concentration killing 1. Amount of time above MIC 2. Increased concentration above MIC * Dosing based on concentration kill, not time * Peaks and trough 1. Peaks = Highest levels of med 2. Trough = Lowest levels of med 3. Trough > 2? Toxicity 4. Trough monitored q3 days * Drug Interactions * Nephrotoxic drugs * Loop diuretics * Intestinal flora * Warfarin Patient Education * If you are experiencing any of the following symptoms, you should call your provider as aminoglycosides cause kidney and ear injury and/or damage: * Nephrotoxicity 1. Protein in urine 2. Increased BUN 3. Increase serum creatinine level 4. Low urine output 5. Fluid retention * Ototoxicity 1. Hearing loss 2. Dizziness 3. Tinnitus 4. Fullness in ear 5. Vertigo

Ileus (Paralytic Ileus)

Occurs when the bowel has decreased motility. Causes Surgery, long-term narcotic use, or complete obstruction Remedies. Medical intervention for physical obstructions. Specific action depend on the cause of the ileus."

Obstruction

Occurs when the lumen of the bowel narrows or closes completely. Causes External compression can be caused by tumor; internal narrowing can be caused by impacted feces. Remedies Remove impaction or tumor."

Name the three stages of swallowing, or deglutition. The Digestive System

Oral stage (mouth to oropharynx), pharyngeal stage (oropharynx to esophagus), and esophageal stage (esophagus to stomach)

Osteoblasts

Originating from mesenchymal stem cells (MSCs), osteoblasts are the primary bone-producing cells, and are involved in many functions related to the skeletal system (see Table 38-1). Osteoblasts are responsive to parathyroid hormone (PTH) and produce osteocalcin when stimulated by 1,25-dihydroxy-vitamin D3.1 Osteoblasts are active on the outer surfaces of bones, where they form a single layer of cells. Osteoblasts initiate new bone formation by their synthesis of osteoid (nonmineralized bone matrix). Osteoblasts also mineralize newly formed bone matrix. Stimulation of new bone formation and orderly mineralization of bone matrix occur by concentrating some of the plasma proteins (growth factors) found in the bone matrix and by facilitating the deposit and exchange of calcium and other ions at the site. Enzymes, signaling proteins, and growth factors, including bone morphogenic proteins (BMPs) and other members of the transforming growth factor-beta (TGF-β) superfamily, are critical components of bone formation, maintenance, and remodeling

Myocardial infarction (MI), or heart attack

Part of the coronary circulation becomes blocked, and cardiac muscle cells die from lack of oxygen The death of affected tissue creates a nonfunctional area known as an infarct Heart attacks most commonly result from severe coronary artery disease (CAD) A crisis often develops as a result of thrombus formation at a plaque (the most common cause of an MI), a condition called coronary thrombosis A vessel already narrowed by plaque formation may also become blocked by a sudden spasm in the smooth muscles of the vascular wall Individuals having an MI experience intense pain, similar to that felt in angina, but persisting even at rest Pain does not always accompany a heart attack; therefore, the condition may go undiagnosed and may not be treated before a fatal MI occurs Damaged myocardial cells release enzymes into the circulation, and these elevated enzymes can be measured in diagnostic blood tests The enzymes include: Cardiac troponin T, Cardiac troponin I, A special form of creatinine phosphokinase, CK-MB

dorsal recumbent (supine)

Patient lies on back with legs slightly apart. Knees are slightly bent with the soles of the feet flat on the bed.

Supine (horizontal recumbent)

Patient lies on back with legs together and extended.

Reverse Trendelenburg

Patient lies on back with legs together. Bed is straight with head of bed higher than the foot.

Trendelenburg

Patient lies on back with legs together. Bed is straight with head of bed lower than the foot. Used in pelvic surgery.

Dorsal lithotomy

Patient lies on back with legs well apart. Knees are bent; stirrups are often used. Position is used to examine the bladder, vagina, rectum, or perineum.

Left lateral recumbency

Patient lies on left side with hips closer to the edge of the bed.

Left Sims'

Patient lies on left side with right knee bent against abdomen. Used in rectal examinations and giving enemas.

Knee-chest/Genupectoral position

Patient rests on knees and chest with head turned to the side. Position is used to examine the rectum or vagina.

side-lying (lateral) position

Patient's head is in straight line with spine. Use pillows to support head, arms, and upper leg.

Identify the various enzymes found in the intestinal juice. The Digestive System

Peptidases, sucrase, lactase, and maltase

Heart- wall

Pericardial cavity Parietal pericardium: Dense fibrous layer, Areolar tissue, Mesothelium, Atrioventricular (AV) valves Connect right atrium to right ventricle and left atrium to left ventricle Are folds of fibrous tissue that extend into openings between atria and ventricles Permit blood flow in one direction From atria to ventricles Structural Differences between the Left and Right Ventricles: Right ventricle wall is thinner, develops less pressure than left ventricle Right ventricle is pouch-shaped, left ventricle is round Epicardium (visceralpericardium): Mesothelium Areolar tissue Myocardium(cardiac muscle tissue): Connective tissues & Cardiac muscle cells contains Intercalated discs Interconnect cardiac muscle cells Linked by gap junctions Convey force of contraction Propagate action potentials Internal Anatomy and Organization Interatrial septum separates atria Interventricularseptum separates ventricles

Physical abuse

Physical abuse is defined as violent acts that result or could result in injury, pain, impairment, or disease.

Which of the divisions of the diencephalon serves as the body's biological clock by secreting the hormone melatonin? The Nervous System

Pineal gland

Coagulation Labs

Platelets 140k- 500k PT Prothrombin time 10-14 sec PTT Partial thromboplastin time 30-45 sec TT Thrombin time control +5 sec FPS Fibrinogen split products -reaction at >1:4 dil Fe dificiency Iron or Ferritin 0-20 ng/mL Reticulocyte count. 0.5-1.5% RBC

Preschoolers communication

Preschoolers are animistic; they imagine inanimate objects can come alive and have human characteristics. Thus, the nurse should not say that the blood pressure cuff will give her arm a hug, that it is asleep, or that give it a name as those are humanistic characteristics and the blood cuff could wake up or come alive. Therefore, health care providers should use short, simple sentences with a concrete explanation for any unfamiliar equipment that will be used on the child.

Constipation -Altered Bowel Elimination Patterns

Presence of large quantity of dry, hard feces that is difficult to expel; frequency of bowel movements is not a factor. Causes : Reabsorption of too much water in the lower bowel as a result of medication such as narcotics, ignoring the urge to defecate, immobility, chronic laxative abuse, low fluid intake, low fiber intake, aging, postoperative conditions, or pregnancy Remedies: Increase fluids, fiber cereals, fruits and vegetables, exercise, and avoid cheese"

Dirty

Presence of many microorganisms or pathogens; any soiled item

Generalized tonic-clonic seizure (grand mal)

Previously called grand mal. Tonic means stiffening, and clonic means rhythmic shaking. There is abnormal electrical activity affecting the whole brain (thus the term "generalized"). Causes Neurologic disease, cancer, head injury, fever, or pregnancy-induced hypertension Interventions Remain calm and turn on the call light. Ensure the patient's safety, lower the bed, and raise the side rails. Stay with the patient, time the seizure, and make sure the patient does not hit his or her head. Document seizure activity and actions taken." Excerpt From Clinical Companion for Fundamentals of Nursing: Just the Facts Patricia A. Potter This material may be protected by copyright.

Kyphoses

Primary curvatures. Seen in thoracic and sacral region, develop during the fetal period. Concave anteriorly.

Sleep- Functional Assessment/DX

Problems (falling asleep, early waking, hours per night, napping) Methods used to facilitate sleep Feelings on waking (fatigued, refreshed) Domain 4: Activity/Rest Class 1: Sleep/Rest Insomnia Sleep Deprivation Readiness for Enhanced Sleep Disturbed Sleep Pattern Class 2: Activity/Exercise Risk for Disuse Syndrome Impaired Bed Mobility Impaired Physical Mobility Impaired Wheelchair Mobility Impaired Transfer Ability Impaired Walking Class 3: Energy Balance Disturbed Energy Field Fatigue Wandering Class 4: Cardiovascular/Pulmonary Responses Activity Intolerance Risk for Activity Intolerance Ineffective Breathing Pattern Decreased Cardiac Output Risk for Ineffective Gastrointestinal Perfusion Risk for Ineffective Renal Perfusion Impaired Spontaneous Ventilation Ineffective Peripheral Tissue Perfusion Risk for Decreased Cardiac Tissue Perfusion Risk for Ineffective Cerebral Tissue Perfusion Risk for Ineffective Peripheral Tissue Perfusion Dysfunctional Ventilatory Weaning Response Class 5: Self-Care Impaired Home Maintenance Readiness for Enhanced Self-Care Bathing Self-Care Deficit Dressing Self-Care Deficit Feeding Self-Care Deficit Toileting Self-Care Deficit Self-Neglect"

Schizophrenia

Profoundly withdrawn from reality, often with bizarre behaviors

isotonic exercise

Promotes osteoblastic activity Requires little resistance Goes though ROM used with repetition Increases muscles and increases bones/tone Push-up, squat, Jog bicycling, Dance aerobics, swimming,

Assessment of Body Mechanics: Physiological Integrity

Proper mechanics, prevent injury, ensure safe patient handling, movement Balance & lifting equipment Joint mobility & Muscles Gait pattern & strength weight-bearing• level of cooperation• weight/height 0= Fully independent 1= Requires assistive devices 2= Requires assistance from another individual 3= Requires assistance from another individual + assistive devices 4= Fully dependent 43 to 67% Basic care and comfort Pharmacological and parenteral therapies Reduction of risk potential Physiological adaptation Pediatrics to gerontology

Antibodies (immunoglobulins) Functions:

Proteins within plasma that react with antigens. Antibody IgM First to respond; activates the complement system; stimulates ingestion by macrophage; principal antibody of the blood IgG Most prevalent antibody; major antibody of the tissues; produced after IgM; only antibody to cross placenta; antitoxin; antiviral IgA Principal antibody of the GI tract; found in tears, saliva, sweat, breast milk; protects epithelial lining IgD Only in minute concentrations; function unknown IgE For allergic reactions"

Psychosocial factors used for nutrition

Psychological Factors ■ Depression ■ Eating disorders ■ Psychosis Social Factors ■ Illiteracy ■ Language barriers ■ Limited knowledge of nutrition and food safety ■ Altered or impaired intake related to culture ■ Altered or impaired intake related to religion ■ Lack of caregiver or social support system ■ Social isolation ■ Lack of or inadequate cooking arrangements ■ Limited or low income ■ Limited access to transportation to obtain food ■ Advanced age (older than 80 years) ■ Lack of or extreme physical activity ■ Use of tobacco or recreational drugs ■ Limited use or knowledge of community resources

Torsades de pointes

Rate: 120 - 200 usually P wave: Obscured by ventricular waves QRS: Wide QRS - "Twisting of the Points" Conduction: Ventricular only Rhythm: Slightly irregular

Values And Beliefs - Functional Assessment

Religious or cultural affiliation Religious or cultural beliefs concerning health or illness Holiday or food restrictions while hospitalized Religious or cultural restrictions on medications or treatments Religious or cultural rituals needed while hospitalized Clergy or religious leader requested while hospitalized

Cardiac Arrest

Remain calm and turn on the call light. Begin CPR (follow standard guidelines) until more experienced staff arrives and takes over. Clear furniture from the room and ask family to move to waiting area. (Some facilities allow family to watch CPR activity.)"

What are the parts of a nephron? The Urinary System

Renal corpuscles Proximal convoluted tubule Henle loop Descending Limb Thick ascending limb Distal convoluted tubule Collecting duct

Stage III - The Inflammatory Process

Repair of tissues. Examples include: Regeneration Same tissues Stroma Connective tissues Parenchyma Functional part Fibrous Scar

Compulsions (OCD)

Repetitive ritualistic actions, such as handwashing, that a person feels driven to perform in an attempt to decrease anxiety and prevent a catastrophe (e.g. contamination [fear of germs], violence, perfectionism, and superstitions.

Immunity

Resistance to a disease associated with the presence of antibodies

Respiration And Circulation-Functional Assessment

Respiratory problems (shortness of breath) Smoking history Circulation problems (chest pain, edema, pacemaker)

Semicomatose

Responds only to pain but has gag and blink reflexes Levels of Consciousness

Heart -Four Chambers

Right atrium Collects blood from systemic circuit Right ventricle Pumps blood to pulmonary circuit Left atrium Collects blood from pulmonary circuit Left ventricle Pumps blood to systemic circuit

Rybelsus

Rybelsus (semaglutide) is used together with diet and exercise to improve blood sugar control in adults with type 2 diabetes mellitus. Rybelsus is usually given after other diabetes medicines have been tried without success. Rybelsus is not for treating type 1 diabetes.

Sedation Scale

S = Sleepy, but easy to arouse 1 = Awake and alert 2 = Slightly drowsy but easy to arouse 3 = Drowsy, drifts to sleep during conversation 4 = Somnolent, minimal or no response to physical stimulation"

The nurse is caring for a client with a hypertensive crisis. This situation becomes a medical emergency when the client shows which sign or symptom? Select all that apply. A. Chest pain B. Facial drooping C. Epistaxis D. Loss of consciousness E. Headache

SBP= <180 DBP= <120 ANS: A, E late sign (objective) patients will experience angina/chest pain frequent headaches

Identify the accessory organs of the digestive system. The digestive System

Salivary glands, tongue, teeth, liver, gallbladder, pancreas, and vermiform appendix

SMAST-G questionnaire

Screening aging adults - Use the SMAST-G questionnaire for older adults who report social or regular drinking of any amount of alcohol - Older adults have specific emotional responses and physical reactions to alcohol, and the 10 questions with yes/no responses address these factors

Laboratory diagnosing for malnutrition

Serum creatinine level >2.5 mg/dl defined high risk, majority of patients elderly, the prevalence of chronic renal problems is high albumin and prealbumin these proteins may help identify patients at high risk for morbidity, mortality, and malnutrition but are not valid criteria for assessing protein status because they become depleted from infl ammation and physiologic stress.

Roles/Relationships - Functional Assessment/DX

Significant other or emergency contacts Primary, secondary, or tertiary roles Role changes caused by illness or wellness Role conflicts caused by illness or wellness Domain 7: Role Relationships Class 1: Caregiving Roles Ineffective Breastfeeding Interrupted Breastfeeding Readiness for Enhanced Breastfeeding Caregiver Role Strain Risk for Caregiver Role Strain Impaired Parenting Readiness for Enhanced Parenting Risk for Impaired Parenting Class 2: Family Relationships Risk for Impaired Attachment Dysfunctional Family Processes Interrupted Family Processes Readiness for Enhanced Family Processes Class 3: Role Performance Ineffective Relationship Readiness for Enhanced Relationship Risk for Ineffective Relationship Parental Role Conflict Ineffective Role Performance Impaired Social Interaction"

"Heart Attack

Signs and symptoms Chest pain; shortness of breath; dyspnea; a squeezing, crushing, or heavy feeling in the chest; lightheadedness; pain in left arm or in the jaw; nausea Intervention Calm the patient and turn on the call light. Begin oxygen at 2 L if nearby. Remain calm and stay with the patient until help arrives. Document symptoms and actions taken."

Emollient

Softens and delays drying of stool Example: Liquid petrolatum Types of Cathartics

Partial seizure

Sometimes confused with petit mal. Only a part of the brain is affected.

Korotkoff Sounds

Sounds of Blood Pressure Phase I Systole (sharp thud) Phase II Systole (swishing sound) Phase III Systole (low thud or knocking) Phase IV Diastole (begins fading) Phase V Diastole (silence) Blood volume Amount of blood in the system Decreased blood volume Equals decreased pressure, meaning increased need for fluids Increased blood volume Equals increased pressure, meaning need for fewer fluids Cardiac output Stroke volume multiplied by heart rate Diastole Ventricular relaxation Pulse pressure Systole minus diastole (normal range is 25-50) Systole Ventricular contraction Viscosity Thickness of the blood Increased viscosity Equals increased pressure, meaning more work on the heart

assistive devices called adaptive devices.

Special equipment that helps a person who is ill or disabled to perform ADLS and walking Walker- pt hold handgrip step/walk Crutches- increase pt mobility 1st then weight to unaffected leg four fingers Cane- first stronger side should be equal distance between greater trochanter and floor

Identify seven various ways to classify the "knee jerk" or patellar reflex. The Nervous System

Spinal cord reflex segmental reflex ipsilateral reflex stretch reflex extensor reflex; tendon reflex deep reflex PERIPHERAL NERVOUS SYSTEM

A suppository

Stimulates bowel and softens stool" Types of Cathartics

Irritant

Stimulates peristalsis by irritating bowel mucosa and decreasing water absorption Example: Castor oil Types of Cathartics

cleansing enema

Stimulates peristalsis; irritates bowel by distention. (Use 1 L of fluid; have patient hold it as long as possible.) "Types of Enemas

Identify the three functions of the gallbladder. The Digestive System

Storage of bile, concentration of bile fivefold to tenfold, and ejection of the concentrated bile into the duodenum

Reservoir

Storage place for organisms to grow

comprehensive nutritional assessment

Subjective Global Assessment (SGA) Weight Change ■ Unintentional weight loss and the time period of loss Dietary Intake ■ Change from normal, duration, type of diet consumed Gastrointestinal Symptoms Lasting Longer than 2 Weeks ■ Nausea, vomiting, diarrhea, anorexia Functional Capacity ■ Normal or suboptimal; ambulatory or bedridden Disease and Its Relation to Nutritional Requirements ■ Primary diagnosis; severity of metabolic stress Physical Signs and Severity of Findings ■ Loss of subcutaneous fat (triceps, chest), muscle wasting (quadriceps, deltoids), ankle edema, sacral edema, ascites Clients considered to be at moderate or high risk for malnutrition through screening are usually referred to a dietitian

Subjective Data

Subjective data is what the person says about him or herself during history taking. Past history of skin disease, allergies, hives, psoriasis, or eczema? Change in pigmentation or color, size, shape, tenderness? Excessive dryness or moisture? Pruritus or skin itching? Excessive bruising? Rash or lesions? Medications: prescription and over-the-counter? Hair loss? Change in nails' shape, color, or brittleness? Environmental or occupational hazards? Self-care behaviors?

Mumps symptoms

Swelling and pain on both sides of face Sore throat Fever Aches and pains Very contagious for up to 7 days before symptoms show and 10 days after symptoms start Symptoms may last 10 days Transmission through airborne droplets and saliva Consideration Vaccination

tools that assess for alcohol use or problems

TWEAK, AUDIT, and SMAST-G

Orthostatic Or Postural Changes

Take blood pressure and pulse with patient lying down. Then have patient sit or stand for 1 minute. Retake blood pressure and pulse. Record both sets of numbers. If patient is orthostatic, pressure will decrease (20-30 mm Hg) and pulse will increase (5-25 beats per minute) when sitting or standing. Record and report any orthostasis."

Describe the components and responsibilities of the central nervous system (CNS). The Nervous system

The CNS comprises both the brain and spinal cord. It is the principal integrator of sensory input and motor output. Thus the CNS is capable of evaluating incoming information and formulating responses to changes that threaten the body's homeostatic balance.

Superficial Anatomy of the Heart

The atria have thin muscular walls which can expand and deflate. Atria: Thin-walled Expandable outer auricle (atrial appendage) Sulci: Coronary sulcus divides atria and ventricles Anterior interventricular sulcus and posterior interventricular sulcus Separate left and right ventricles Contain blood vessels of cardiac muscle

Why are the calyces considered the beginning of the "plumbing system" of the urinary system? The Urinary System

The calyces are considered the beginning of the "plumbing system" of the urinary system because it is the place where urine leaving the renal papilla is collected for transport out of the body.

What are the three general functions of the cerebellum The Nervous System

The cerebellum acts with the cerebral cortex to produce skilled movements by coordinating the activities of groups of muscles; it helps control posture; and it controls skeletal muscles to maintain equilibrium. CENTRAL NERVOUS SYSTEM

anticoagulant

The general term for a substance that prevents or delays coagulation of the blood ■Dalteparin ■Enoxaparin ■Heparin ■Warfarin coagulation modifier drugs/Drugs affecting Hemostasis

biomedical theory

The germ theory, which states that microscopic organisms such as bacteria and viruses are responsible for specific disease conditions

Explain what is meant when the kidneys are described as lying in a retroperitoneal position. The Urinary System

The kidneys lie in a retroperitoneal position, which means they are posterior to the parietal peritoneum, against the posterior wall of the abdomen.

postictal state seizure

The period after a generalized or partial seizure during which the person usually feels sleepy or confused.

Wernicke's aphasia

The person can hear sounds and words but cannot relate them to previous experiences. Speech is fluent, effortless, and well-articulated, but it has many paraphasias (word substitutions that are malformed or wrong) and neologisms (made-up words) and often lacks substantive words. Speech can be totally incomprehensible. Often, a great urge to speak is present. Repetition, reading, and writing also are impaired.

complex partial seizure

The person is conscious but impaired.

If you trace the axon inside the sympathetic chain ganglion, the preganglionic fiber may branch along what three paths? The Nervous system

The preganglionic fiber can synapse with a sympathetic postganglionic neuron. It can send ascending or descending branches through the sympathetic trunk to synapse with postganglionic neurons in other chain ganglia. Or, it can pass through one or more ganglia without synapsing.

hemostasis

The termination of bleeding by mechanical or chemical means coagulation modifier drugs

Ulnar pulse

The ulnar is on the opposite side of the wrist and feels less prominent. Palpate the ulnar pulse only when evaluating arterial insufficiency to the hand.

Protein Excess

There are no proven risks from eating an excess of protein. Data are conflicting as to whether high-protein diets increase the risk of osteoporosis or renal stones. Although a UL has not been established, this does not mean there is no potential for adverse effects from a high protein intake from food or supplements (National Research Council, 2005).

Heart blood vessels

Three Types of Blood Vessels: Supply to the Heart The Coronary Arteries= Coronary circulation Arteries-Carry blood away from heart a. Right Coronary Artery, Right atrium Portions of both ventricles Cells of sinoatrial (SA) and atrioventricular nodes Marginal arteries (surface of right ventricle) Posterior interventricular artery b.Left Coronary Artery, Supplies blood to: Left ventricle,Left atrium &Interventricular septum, Two Main Branches of Left Coronary Artery Circumflex artery Anterior interventricular artery Veins-Carry blood to heart. The Cardiac Veins: a.Great cardiac vein-Drains blood from area of anterior interventricular artery into coronary sinus b. Anterior cardiac veins- Empty into right atrium c.Posterior cardiac vein, middle cardiac vein, and small cardiac vein Empty- into great cardiac vein or coronary sinus Capillaries-Networks between arteries and veins

Anistreplase (Eminase)

Thrombolytic (clot-buster) ■ Drug class: Thrombolytic ■ Peak time: 45 min ■ Duration: 4-6 hr ■ Half-life: 70-120 min

Radial pulse

To locate pulses in the arm have the patient sit or lie down. Location along the radial side of the forearm at the wrist. Thin individuals have a groove lateral to the flexor tendon of the wrist. Feel with light palpation in the groove

AIDS (acquired immune deficiency syndrome)

Transmission through blood and body fluids, sexual contact, sharing IV needles, contaminated blood, and from mother to fetus Considerations Education regarding mode of transmission, avoidance of sexual contact with infected persons, use of latex condoms, proper blood screening of all transfusable products, and proper handling of needles and other contaminated material

Tetanus (Lockjaw)

Transmission through direct contact of wounds with infected soil or feces Considerations Public education regarding mode of transmission, vaccination

Hepatitis A And Hepatitis E

Transmission through direct contact with water, food, or feces Considerations Handwashing before touching food, proper water and sewage treatment, reporting of cases, immunoglobulin vaccination when traveling to high-risk areas, proper disposal of contaminants

Polio (Poliomyelitis) virus

Transmission through oral or fecal contact Consideration Vaccination

Chickenpox/Herpes Zoster Virus (Varicella/Shingles)

Transmission through respiratory droplets or by direct contact with open lesions Considerations Contact isolation, avoid direct contact with lesions, and administration of varicella zoster immune globulin. Caregivers should be chickenpox immune. New vaccines are available.

Syphillis

Transmission through sexual contact, direct contact with lesions, and blood transfusions Considerations Public education regarding transmission, prenatal screening, and prenatal follow-up; use of latex condoms; blood screening

Gonorrhea

Transmission through vaginal secretions, semen, sexual contact Considerations Public education regarding mode of transmission; use of latex condoms. Some strains are antibiotic resistant.

Coronary artery bypass graft (CABG)

Treatment of CAD and myocardial infarction Coronary artery bypass graft (CABG) is a more invasive option in treating angina. It involves bypassing constricted arteries using venous grafts obtained during cardiac surgery in order to provide adequate blood flow to cardiac tissues. Because of long-term patency rates (90% after 10 years), the internal mammary artery (IMA) is the most common artery used for a CABG. In a coronary artery bypass graft, a small section is removed from either a small artery or a peripheral vein and is used to create a detour around the obstructed portion of a coronary artery The procedures are named according to the number of vessels repaired, so we speak of single, double, triple, or quadruple coronary bypasses Heart Disease - Coronary Artery Disease

Balloon angioplasty

Treatment of CAD and myocardial infarction Noninvasive surgery Balloon angioplasty The tip of the catheter contains an inflatable balloon Once in position, the balloon is inflated, pressing the plaque against the vessel walls Because plaques commonly redevelop after angioplasty, a fine tubular wire mesh called a stent may be inserted into the vessel, holding it open Heart Disease - Coronary Artery Disease

prothrombin time (PT) & International Normalized Ratio (INR)

Two tests used to monitor the effects of drug therapy with warfarin sodium coagulation modifier drugs

Nutrition - Functional Assessment/ DX

Type of diet (list) Enjoys snacks (yes/no, what type) Fluid intake (types of fluids) Fluid restriction (yes/no) Skin (normal, dry, rash) Teeth (own, dentures, bridge) Weight (recent gain or loss) Domain 2: Nutrition Class 1: Ingestion Insufficient Breast Milk Ineffective Infant Feeding Pattern Imbalanced Nutrition: Less Than Body Requirements Imbalanced Nutrition: More Than Body Requirements Readiness for Enhanced Nutrition Risk for Imbalanced Nutrition: More Than Body Requirements Impaired Swallowing Class 2: Digestion Class 3: Absorption Class 4: Metabolism Risk for Unstable Blood Glucose Level Neonatal Jaundice Risk for Neonatal Jaundice Risk for Impaired Liver Function Class 5: Hydration Risk for Electrolyte Imbalance Readiness for Enhanced Fluid Balance Deficient Fluid Volume Excess Fluid Volume Risk for Deficient Fluid Volume Risk for Imbalanced Fluid Volume"

aluminum

Type of test Blood Normal findings All ages: 0-6ng/mL Dialysis patients of all ages: <60ng/mL Test explanation and related physiology Under normal physiologic conditions, the usual daily dietar y intake of aluminum (5 to 10 mg) is completely excreted by the kidneys. Patients in renal failure (RF) lose the ability to clear aluminum and are at risk for aluminum toxicity. Aluminum-laden dialysis water and aluminum-based phosphate binder gels designed to decrease phosphate accu- mulation increase the incidence of aluminum toxicity in RF patients. Furthermore, the dialysis process is not highly effec- tive at eliminating aluminum. If aluminum accumulates, it binds to albumin and is rapidly distributed throughout the body. Aluminum overload leads to accumulation of aluminum in the brain and bone. Brain depo- sition has been implicated as a cause of dialysis dementia. In bone, aluminum replaces calcium and disrupts normal osteoid formation. Serum aluminum concentrations are likely to be increased above the reference range in patients with metallic joint pros- thesis. Serum concentrations >10ng/mL in a patient with an aluminum-based implant suggest significant prosthesis wear. Chromium and other metals can be detected using similar labo- ratory techniques. Interfering factors • Special evacuated blood collection tubes are required for alu- minum testing. • Most of the common evacuated blood collection devices have rubber stoppers that are comprised of aluminum- silicate. Simple puncture of the rubber stopper for blood collection is sufficient to contaminate the specimen with aluminum. • Gadolinium- or iodine-containing contrast media that have been administered within 96 hours can alter test results for heavy metals, including aluminum. Procedure and patient care • Fasting: no • Blood tube commonly used: royal blue or tan • If the blood sample is sent to a central diagnostic laboratory, results will be available in 7 to 10 days. Abnormal findings Increased levels Aluminum toxicity

amino acid profiles (Amino acid screen)

Type of test Blood; urine Normal findings Normal values vary for different amino acids. Test explanation and related physiology Amino acids are "building blocks" of proteins, hormones, nucleic acids, and pigments. They can act as neurotransmitters, enzymes, and coenzymes. There are eight essential amino acids that must be provided to the body by the diet. The body can make the others. The essential amino acids must be transported across the gut and renal tubular lining cells. The metabolism of the essential amino acids is critical to the production of other amino acids, proteins, carbohydrates, and lipids. Amino acid lev- els can thereby be affected by defects in renal tubule or gastroin- testinal (GI) transport of amino acids. When there is a defect in the metabolism or transport of any one of these amino acids, excesses of their precursors or deficien- cies of their "end product" amino acid are evident in the blood and/or urine. There are more than 90 diseases described that are associated with abnormal amino acid function. Clinical manifestations of these diseases may be precluded if diagnosis is early, and appropriate dietary replacement of miss- ing amino acids is provided. Usually, urine testing for specific amino acids is used to screen for some of these errors in amino acid metabolism and transport. Blood testing is very accurate. Federal law now requires hospitals to test all newborns for inborn errors in metabolism including amino acids. Testing is required for errors in amino acid metabolism such as phenylketonuria (PKU), maple syrup urine disease (MSUD), and homocystinuria (see newborn metabolic screening, p. 657). Testing for more rare disorders may include testing for tyrosinemia and argininosuc- cinic aciduria. A few drops of blood are obtained from the heel of a new- born to fill a few circles on filter paper (Guthrie card) labeled with names of infant, parent, hospital, and primary physician. The sample is usually obtained on the second or third day of life, after protein-containing feedings (i.e., breast milk or formula) have started. After a presumptive diagnosis is made, amino acid levels can be determined by chromatographic methods on blood or amniotic fluid. The genetic defects for many of these diseases are becoming amino acid profiles 45 more defined, allowing for even earlier diagnosis to be made in utero.

Elimination- Functional Assessment/DX

Upper GI (nausea, vomiting, dysphagia, discomfort) Bowels (frequency, consistency, last bowel movement, ostomy) Bladder (incontinence, dysuria, urgency, frequency, nocturia, hematuria) Domain 3: Elimination And Exchange Class 1: Urinary Function Functional Urinary Incontinence Overflow Urinary Incontinence Reflex Urinary Incontinence Stress Urinary Incontinence Urge Urinary Incontinence Risk for Urge Urinary Incontinence Impaired Urinary Elimination Readiness for Enhanced Urinary Elimination Urinary Retention Class 2: Gastrointestinal Function Constipation Perceived Constipation Risk for Constipation Diarrhea Dysfunctional Gastrointestinal Motility Risk for Dysfunctional Gastrointestinal Motility Bowel Incontinence Class 3: Integumentary Function Excerpt From Clinical Companion for Fundamentals of Nursing: Just the Facts Patricia A. Potter This material may be protected by copyright.

whooping cough (pertussis)

Upper respiratory infection caused by the Bordetella pertussis bacteria. May cause permanent disability in infants and even death. Transmission through airborne droplets and nasal discharge Considerations Vaccination, wearing of masks when near infected patients, reporting of all cases"

Identify the accessory organs of the urinary system that are involved in the transport of the urine from the kidneys to the outside of the body.

Ureters, urinary bladder, and urethra

Name the body systems that excrete unneeded substances as part of their functions.

Urinary (nitrogenous wastes), integumentary (sweat), respiratory (carbon dioxide), and digestive (feces)

Airborne Precautions

Used for airborne infectious agents of 5 micrometers or smaller

Carminative -Types of Enemas

Used to expel flatus

Passive ROM

Used to move the client Joints indicated for temporary/permanent loss of mobility, sensation, or consciousness As FULl a range as possible Improves/maintain MOBILITY and help prevent CONTRACTURE.

Vegetarian Diets

Vegetarian sources of protein include legumes, nuts, nut butters, and soy products such as soy milk, tofu, tempeh, and veggie burgers. ■Eat a variety of foods including whole grains, vegetables, fruits, legumes, nuts, seeds, and if desired, dairy products and eggs. Consider meatless versions of familiar favor- ites, such as vegetable pizza, vegetable lasagna, vegetable stir-fry, vegetable lo mein, and vegetable kabobs. ■Experiment with meat substitutes made from vegetables. For variety, try soy sausage patties or links, veggie burgers, or soy hot dogs. ■Eat enough calories. Adequate calories are necessary to avoid using amino acids for energy, which could lead to a shortage of amino acids for protein synthesis. ■Consume a rich source of vitamin C at every meal. Eating a good source of vitamin C at every meal helps to maximize iron absorption from plants. Try orange and citrus fruits, tomatoes, kiwi, red and green peppers, broccoli, Brussels sprouts, cantaloupe, and strawberries. ■Include two servings daily of fats that supply omega-3 fats. A serving is 1 tsp flaxseed oil, 1 tbsp canola or soybean oil, 1 tbsp ground flaxseed, or 1⁄4 cup walnuts. Nuts and seeds may be used as substitutes from the Fat group. ■Don't go overboard on high-fat cheese as a meat substitute. Full-fat cheese has more saturated fat and calories than many meats. (e.g. cheddar 33.3g per 100 g; blue cheese/Gouda 28.7g per 100 g; parmesan 27.8g per 100 g; feta cheese 21.3g per 100 g ■Experiment with ethnic cuisines. Many Asian, Middle Eastern, and Indian restaurants offer a variety of meatless dishes. ■Supplement nutrients that are lacking from food. For vegans, this means vitamin B12 (unless reliable fortified foods are consumed) and perhaps vitamin D. The adequacy of calcium, iron, and zinc intake is evaluated on an individual basis. should be planned to provide adequate protein, essential fatty acids, calcium, iron, zinc, vitamin B12, and vitamin C

Protein Foods- Very Lean Choices

Very Lean Choices (0-1 g fat/oz) >>Ground turkey breast and ground chicken breast are made only from white meat and are much lower in fat (0.5 g/3 oz cooked) than all varieties of ground beef (5.5-17 g fat/3 oz cooked). Poultry Skinless white meat chicken or turkey Skinless Cornish hen Fish Cod, flounder, haddock, halibut, trout, smoked salmon, fresh or canned-in-water tuna Shellfish Clams, crab, lobster, scallops, shrimp, imitation shellfish Game Skinless duck or pheasant, venison, buffalo, ostrich Cheese Fat-free or low-fat cottage cheese Fat-free cheese Processed meats Those with 1 g of fat or less per ounce, such as turkey ham Other Egg whites, egg substitutes Kidney Sausage with 1 g of fat or less per ounce Legumes, lentils (cooked) ■ Choose a variety of protein foods, which include seafood, lean meat and poultry, eggs, beans and peas, soy products, and unsalted nuts and seeds. ■ Increase the amount and variety of seafood consumed by choosing seafood in place of some meat and poultry. ■ Replace protein foods that are higher in solid fats with choices that are lower in solid fats and calories and/or are sources of oils.

Fat-soluble vitamins

Vitamin A Vitamin D Vitamin E Vitamin K <--deficiency--> ■Vitamin A deficiency causes Bitot spots and visual problems. ■vitamin D deficiency - Osteomalacia 'adults' &Rickets is a condition in the youth

Saline Laxatives

When salt is in the bowel, the water will remain in the bowel as well. (Avoid use in patients with impaired renal function.) Example: Milk of magnesia (MOM), Epsom salts Types of Cathartics

Doppler devices

When unable to palpate a pulse, devices are used to augment pulse or blood pressure measurements.

subculture

Within cultures, groups of people share a subculture different beliefs, values, and attitudes. Differences occur because of ethnicity, religion, education, occupation, age, and gender than the majority of the larger culture.

Word salad (schizophasia)

Word salad is an incoherent mixture of words, phrases, and sentences.

non polar covalent bond

a covalent bond in which the bonding electrons are shared equally by the bonded atoms, resulting in a balanced distribution of electrical charge

full weight-bearing (FWB)

a doctor's order stating that a person has the ability to support full body weight on both legs and has no weight-bearing limitations

Partial weight bearing (PWB)

a doctor's order stating that a person is able to support some body weight on one or both legs

Glasgow Coma Scale (GCS)

a scale used to assess the consciousness of a patient upon physical examination, typically in patients with neurological concerns or complaints Best eye opening response, record "C" if eyes closed by swelling Best motor response to stimuli, record best upper limb response,

Culture Shock

a term used to describe the state of disorientation or inability to respond to the behavior of a different cultural group because of its sudden strangeness, unfamiliarity, and incompatibility with the individual's perceptions and expectations. Stages: Honeymoon, crisis, recovery, adjustment, reverse

isometric exercise

activity that uses muscle tension to improve muscular strength, tone, muscle mass with little or no movement through ROM. Promotes osteoblastic activity. Ideal patient: with decreased activity tolerance, immobilized. Contracting muscles against resistance without JOINT movement, and maintain BALANCE, will increase CENTER OR GRAVITY. Requires little time only comfortable position and proper body alignment. Stationary Hip lifts, wall sits, planks, Footboard and yoga

inappropriate affect

affect is an affect clearly discordant with the content of the person's speech.

Bulimia

an eating disorder characterized by episodes of 'binge eating'-overeating, usually of high-calorie foods, followed by self-induced vomiting., laxative use, fasting, or excessive exercise

An emergency database is: A) used to collect data rapidly and is often compiled concurrently with life-saving measures. B) used for a limited or short-term problem usually consisting of one problem, one cue complex, or one body system. C) used to evaluate the cause of disease. D) used to monitor progress of short-term or chronic health concerns.

ans: a An emergency database is an urgent, rapid collection of data often compiled concurrently while lifesaving measures are being performed. patient with sudden and severe shortness of breath.

The nurse is caring for a client who is newly diagnosed with multiple sclerosis. Which of the following is appropriate teaching for this client? * "Here is a list of foods containing rye, barley, and oats. You must not consume these if you want to avoid flare ups" * "Make sure you're getting good fluid intake, around 2L/day" * "If you are compliant with your treatment regimen, there is a chance this will resolve completely" * "You may noticed you start to walk by shuffling, or move your hands like you're rolling a pill. That is normal"

answer B ✅"Make sure you're getting good fluid intake, around 2L/day" Multiple sclerosis (MS) can cause incontinence, and this is difficult to cope with. Many clients with MS will drink less to avoid episodes of incontinence, but this leads to dehydration and constipation. The client with MS must be taught to continue adequate fluid intake. * "If you are compliant with your treatment regimen, there is a chance this will resolve completely" MS is a chronic, progressive condition that cannot be cured. Nursing concepts for MS focus on supportive care. * "Here is a list of foods containing rye, barley, and oats. You must not consume these if you want to avoid flare ups" Avoiding rye, oats and barley is for those with celiac disease, not MS. * "You may noticed you start to walk by shuffling, or move your hands like you're rolling a pill. That is normal" A shuffling gait and pill-rolling is common with Parkinson's, not MS.

protamine sulfate

antidote for heparin 1-2 mg protamine is necessary to neutralize 1 mg (100 IU) heparin. Protamine has several adverse effects; these include hypotension and flushing, which are probably related to histamine release and which can be minimized by giving protamine very slowly. There is a high incidence of anaphylactic and anaphylactoid reactions to protamine; these are most com- mon in patients who have fish allergy and in diabetics receiving protamine-based insulin. Protamine activates complement and releases platelet thromboxane A2, which may lead to severe pul- monary vasoconstriction and bronchoconstriction.

Diabetes

are a random serum glucose measurement and a urine dipstick test. Subsequent laboratory tests could include a fasting glucose test, a glycosylated hemoglobin analysis, a fasting lipid profile, an electrocardiogram, and an electromyogram. Repeat BP (152/100mm Hg) is a hypertensive value. Random glucose value (262 mg/dl), fasting glucose value (171mg/dl), and high quantities of glycosylated hemoglobin satisfy/exceed the criteria for a diagnosis of diabetes

Third-level priority problems

are important to the patient's health but can be addressed after more urgent problems. Examples include lack of knowledge or family coping

cultural taboo

are practices that are to be avoided, such as receiving blood products, eating pork, and consuming caffeine.

Hallucinations

are sensory perceptions for which no external stimuli exist. They may strike any sense: visual, auditory, tactile, olfactory, or gustatory.

Open-ended questions

are used for gathering narrative information. Open-ended questions allow for self-expression, build and enhance rapport, and obtain narrative information.

A food diary

asks individuals to write down everything consumed for a certain period of time. Having the patient complete a food diary for 3 days, including 2 weekdays and 1 weekend day individual with erratic eating patterns ,

Percussion

assesses through the use of palpable vibrations and audible sounds yields a sound that depicts the location, size, and density of the underlying organ. Turgor, texture, and consistency are assessed with palpation.


संबंधित स्टडी सेट्स

Life Insurance: Life Policy Riders

View Set

Intro to Archaeology QUIZ 1 (ANTH 107)

View Set

Chapter 57 Nutrition, Exercise, and Healthy Living

View Set

lesson 4- Writing Vision and Mission Statements

View Set

Managing People and work - QUIZ 11

View Set

Simulation Lab 4.2: Module 04 Use Remote Desktop Configure Firewall

View Set